You are on page 1of 244

Inicet

November 2021
Index
1. Anatomy 09

2. Physiology 31
3. Biochemistry 42
4. Pathology 60
5. Microbiology
86
6. Pharmacology 102
7. Forensic Medicine 117
8. ENT 127
9. Ophthalmology 136
10. SPM 145
11. Anaesthesia 154
12. Dermatology 158
13. Psychiatry 168
14. Radiology 170
15. Medicine 178
16. Surgery 195
17. Orthopaedics 207
18. Paediatrics 216

19. Obstetrics and Gynaecology 223


Anatomy
1. Which of the following parts of the scapula is palpable in the infraclavicular fossa?
(A) C
(B) A
(C) D
(D) B
The given figure is that of Scapula.

:
Scapula is a bone of the upper limb.
Coracoid process is palpable in the
infraclavicular fossa.

Parts of scapula
Coracoid Process
It is a separate bone in reptiles, but is attached to scapula in humans.
Thus it represents an atavistic epiphysis.
" Coroacoid process provides attachment to three muscles:
Short head of biceps
: Coracobrachialis
Pectoralis Minor

: Upper border of scapula has suprascapular notch, which is converted into a foramen by
superior transverse (suprascapular) ligament.
Suprascapular artery passes above the ligament and suprascapular nerve passes below the

:
ligament through the foramen.
The superior angle of scapula lies over the second rib and the inferior angle lies over the seventh
rib.

Sprengel’s deformity of Scapula


Failure of migration of scapula
from neck region to adult
position during intrauterine
life.

2. From which of the following does the marked structure develop?


(A) Subcardinal vein
(B) Supracardinal vein
(C) Common cardinal vein
(D) Anterior cardinal vein
Venous drainage of chest wall is
done by intercostal veins.
Intercostal Veins

Posterior Intercostal Vein


u
Right Posterior intercostal Veins
-

2nd, 3rd, 4th - Forms Right Superior Intercostal Vein


Drains Into

Arch of Azygous Vein


-

5, 6, 7, 8, 9, 10, 11 - Forms Azygos Vein

Drains into
Superior Vena Cava

-
1st drains into Right Brachiocephalic Vein.
u
Left Posterior Intercostal Veins
-
1st drains into Right Supracardinal Vein
-
2nd, 3rd, 4th - Forms Left Suprior Intercostal Vein
Drains into

Left Brachiocephalic Vein

-
5th, 6th, 7th, 8th - Forms Accessory Hemiazygos Vein
Drains into

Azygos Vein at T7
-
9th, 10th, 11th - Forms Hemiazygos Vein
Drains into
Azygos vein at T8
Anterior Intercostal vein
1st - 6th Drains into Internal Thoracic Vein.
: 7th, 8th and 9th drains into Musculophrenic vein.

Embryology
Azygos Vein - Right Supracardinal Vein
: Superior Vena Cava - Right Anterior Cardinal Vein + Right Common Cardinal Vein
Inferior Vena Cava - Sacrocardinal Vein + Supracardinal Vein + Subcardinal Vein +
: Hepatocardiac Channel
Portal Vein - Left Vitelline Vein + Dorsal Anastomosis + Right Vitelline Vein +
Proximal Ventral Anastomosis

3. Which nerve fibre pass from the marked structure?


(A) CN III
(B) CN IV
(C) CN V
(D) CN VI

The image given in the question points to the dorsal aspect of the brain stem.
: CN IV (Trochlear Nerve) is the only cranial nerve that exits dorsally from the brainstem.
Origin of Cranial Nerve
CN 1 (Olfactory Nerve) : Arises from olfactory bulb.

÷;
CN 2 (Optic Nerve) : Arises from Optic Chiasma
CN 3 (Occulomotor Nerve) : Arises from Ventral Mid Brain
CN 4 (Trochlear Nerve) : Arises from Dorsal Mid Brain
CN 5 (Trigeminal Nerve) : Arises from Ventral Pons
CN 6 (Abducent Nerve) : Arises from ponto - pyramidal junction.
CN 7 (Facial Nerve) : Arises from ponto - olivary junction.
CN 8 (Vestibulocochlear Nerve) : Lateral Pontomedullary Junction
CN 9, 10, 11 : Arise from medulla, lateral to olive.
s
CN 12 : Arises from medulla between pyramid and olive.

4. A patient presents with base of skull fracture involving the junction between temporal and
occipital bones. He was not able to speak properly after the trauma. Which of the marked
structures would you test to confirm the associated syndrome?
(A) B
(B) D
(C) A
(D) C
In this question, the fracture

: involves the junction between


temporal bone and occipital bone.
This junction contains the
jugular foramen which transmits
the 9th, 10th & 11th cranial
nerve.
Temporal
Bone

Jugular
Foramen

Occipital Bone
Vernet Syndrome
Injury to the jugular foramen structures can result in Vernet Syndrome which results in :

÷Dysphonia/Hoarseness (CN X)
Dysphagia (CN IX)
Drooping of soft palate (CN X)
Deviation of Uvula to same side (CN X)
:Loss of sensation of posterior 1/3rd of tongue (CN IX)
Loss of gag reflex (CN IX)
Sternocleidomastoid & Trapezius muscle paresis (CN XI)
: Since the clinical history is suggestive of dysarthria, we have to test the vocal
function of the patient to reach a diagnosis.
5. Third part of vertebral artery is related to which of the following?
(A) C
(B) A
(C) D
(D) B
Vertebral Artery

I
Vertebral artery is one of the principal arteries that supply the brain.
It is the first and largest branch of the first part of Subclavian Artery.
It has 4 parts :
I - Cervical Part : Extends from origin to foramen tranversarium of C6 vertebra.
II - Vertebral Part : Lies within the foramen transversarium of upper six cervical vertebra.
III - Suboccipital Part : Extends from foramen tranversarium of C1 Vertebra to foramen
magnum of skull.
-
IV - Intracranial Part : Extends from foramen magnum to lower border of pons.
In the cranial cavity, the vertebral arteries from both sides unite to from the basilar artery.

:
In the image of the C1 vertebra shown (Atlas), the 3rd part of vertebral artery moves from
foramen transversarium to foramen magnum reaching the posterior arch of the vertebra.

Subclavian Steal Syndrome


Obstruction of subclavian artery proximal to
origin of vertebral artery resulting in retrograde
flow of blood from opposite vertebral artery to
the subclavian artery of the affected side.
-
This causes stealing of blood of the brain.
6. The most specific complication that can occur if a central line is inserted below this bone:
(A) Chylothorax
(B) Arterial puncture
(C) Empyema
(D) Thrombophlebitis

The given image shows the left clavicle.

:
The left clavicle is closely related with cervical part of the thoracic duct which drains
lymph from the whole of the body except the right upper.
Central line when placed here can injure the thoracic duct resulting in chylothorax
(accumulation of chyle in lung cavity).

Thoracic Duct
Thoracic duct is the largest lymphatic vessel which drains lymph from whole of body except the
a) Right side of head and neck
b) Right side of chest wall
c) Right Lung
d) Right side of heart
e) Right surface of liver
It arises from the upper end of cisterna chyli on the posterior abdominal wall at the T12 vertebra
:level till the junction of left internal jugular vein and left subclavian vein at the root of neck.
Lymph in thoracic duct is milky white in appearance and is called chyle.
Central Line
Use

:
Parenteral nutrition, in cases of difficult peripheral venous access, haemodialysys,
transvenous cardiac pacing, pulmonary artery catheterisation.
Types
Subclavian, Internal Jugular, Femoral Vein
Insertion
: Done using Seldinger technique under USG guidance.
A blunt guidewire is passed through the needle, then the needle is removed.
: A dilating device may be passed over the guidewire to expand the tract.
Finally, the central line itself is then passed over the guidewire, which is then removed.
: All the lumens of the line are aspirated (to ensure that they are all positioned inside the
vein) and flushed with either saline or heparin.
A chest X-ray may be performed afterwards to confirm that the line is positioned inside
: the superior vena cava and no pneumothorax was caused inadvertently.
Complications
Pneumothorax, Vascular Injury, Infection, Venous Air Embolism, Chylothorax
Central line equipments
In order of typical usage:
1. Syringe with local anesthetic
: 2. Scalpel
3. Sterile gel for ultrasound guidance
: 4. Introducer needle (here 18 Ga) on syringe with saline
to detect backflow of blood upon vein penetration.
5. Guide wire
: 6. Tissue dilator
7. Indwelling catheter (here 16 Ga)
: 8. Additional fasteners, and
corresponding surgical thread
9. Dressing

:
7. Inferior thyroid artery does not supply:
(A) Thyroid
(B) Parathyroid
(C) Oesophagus
(D) Thymus
Inferior Thyroid Artery
Course
Inferior thyroid artery arises from the thyrocervical trunk which arises from the 1st part of
subclavian artery.
^
It passes upward in front of vertebral artery and longus colli muscle and moves medially
behind the carotid sheath contents to reach the lower border of thyroid gland where it divides
into two branches to supply the posteroinferior part of the gland and anastomose with the
superior thyroid artery.
Branches of Inferior Thyroid Artery
Inferior Laryngeal Artery
^

Tracheal Branches

÷
Oesophageal Branches
Ascending Cervical Artery : Anastomose with vertebral, pharyngeal and occipital artery.
Pharyngeal Branches
Branches to Thyroid Gland : Supplies thyroid and parathyroid.

Clinical Application
^
Inferior thyroid artery is closely related to the recurrent laryngeal nerve, hence it can be
injured during thyroidectomy.
^

Inferior thyroid artery also supplies the parathyroid glands, hence during thyroid
surgery, ITA must be ligated as distal as possible to avoid devascularisation of
parathyroid leading to hypoparathyroidism.

8. Fibres from mammillary body ends in which area?


(A) Thalamic nuclei
(B) Hippocampus
(C) Basal ganglia
(D) Amygdala
Mammillary Body
Mammillary bodies are a pair of small round bodies which are a part of the
: diencephalon and forms a part of limbic system.
They are located at the anterior end of the fornix.
They are important for processing of memories.
: Damage to mammillary bodies can result in impaired memory as seen in Wernicke
Korsakoff Syndrome.

The limbic system is responsible for mediating emotion, behaviour, long term
: memory and olfaction.
It is composed of the above depicted parts.
Papez Circuit
'
The functions of limbic system are regulated by the Papez circuit.

The fornix (association fibres) connect the hippocampus to the mammillary bodies.
: The mammillary bodies are connected to the anterior thalami nuclei.

9. A patient presented to the OPD with the following deformity of hand. Which of the following nerve
is most likely injured?
(A) Median
(B) Radial
(C) Ulnar
(D) Axillary

Ulnar Nerve supplies the intrinsic muscles of hand, the flexor carpi ulnaris and medial half of
: flexor digitorum profundus.
Median Nerve supplies the muscle of forearm (including flexor digitorum superficialis, lateral
part of flexor digitorum profundus) and some muscles of hand (flexor pollicis brevis, Abductor
pollicis brevis, Opponens pollicis, Lumbricals 1 & 2).
-
Paralysis of the median nerve results in ape thumb deformity because of atrophy of thenar
eminence.
Ape Hand Deformity
Ape hand deformity is a deformity in humans who cannot move the thumb away from the
:rest of the hand.
It is an inability to abduct the thumb.
Abduction of the thumb refers to the specific capacity to orient the thumb perpendicularly

:
to the ventral (palmar) surface of the hand.

The image in question shows a partial claw hand affecting the medial 2 fingers only.
Hence this is more likely an ulnar nerve injury as only the medial fingers are affected and there is

:
no ape thumb.

Radial Nerve
Muscles innervated
Triceps
: Extensor carpi radialis and ulnaris
Supinator
: Extensor pollicis
Motor functions
: Extension at all arm, wrist and proximal
finger joints below the shoulder.
Forearm supination.
: Thumb abduction in plane of palm.
Axillary Nerve
Muscles innervated

: Deltoid and teres minor


Motor functions
Abduction of arm at shoulder beyond first 15°
a
Sensory
Skin over the shoulder

Ulnar Nerve
-

Muscles innervated
Flexor carpi ulnaris
-

Flexor digitorum profundus


:
Adductor pollicis
Small digital muscles
-

Motor functions
Finger adduction and abduction
-

other than thumb.


Thumb adduction, flexion of
:
digits 4 & 5.
Wrist flexion and adduction.
-

Sensory
Skin over medial surface of the
-

hand through the superficial


branch.
Median Nerve
-

Muscles Innervated
Flexor carpi radialis
-

Palmaris longus
-

Pronator quadratus
:
Pronator teres
Digital flexors
-

Motor functions
Thumb flexion and opposition
-

Flexion of digits 2 & 3


: Wrist flexion and abduction
Forearm pronation.
-

Sensory
Skin over anterolateral
-

surface of hand.

Tests for Ulnar Nerve Injury

Card Test
Inability to hold a card or paper between the fingers due to loss of adduction by the
palmar interossei.

Egawa Test
The patient is asked to move the middle finger sideways with palm placed flat on the table.
: This is a test for the dorsal interossei of middle finger.
Book Test : Froment’s Sign
-
The patient is asked to hold a book between the thumb and other fingers.
-
In ulnar nerve injury, the first two muscles are paralysed and now the patient has to depend upon
-
Flexor Pollicis Longus (FPL) which flexes the thumb prominently.
Tests for Median Nerve Injury
Pen Test (Abductor Pollicis Brevis)
Abduction of thumb - A pen is kept at a level higher than the thumb and the patient
is asked to touch the tip of the pen.

Pointing Index/ Oschner Clasp Sign

Kiloh - Nevin Sign/ Weak OK Sign


To assess flexor digitorum profundus and flexor pollicis longus innervation.
: Ask the patient to make an okay sign by touching the tips of the thumb and index
finger together.
e
With weakness in these muscles, the distal phalanges cannot flex and instead of
fingertips touching, the volar surfaces of each distal phalanx make contact.
10. A patient with history of fall from height was managed with a cast. After removal of cast, he had
difficulty in walking properly. He was not able to lift his leg and it kept dragging while walking.
Which of the following structures could have been injured?
(A) Gluteus medius
(B) Gluteus maximus
(C) Pyriformis
(D) Quadratus femoris

Gluteal Muscles and their Actions


Gluteus Maximus : Extension and Lateral Rotation of Hip.
:
Gluteus Minimus and Medius : Abduction and medial rotation of hip.
Prevents sinking of opposite hip while walking.
Piriformis, Obturator Internus,
: Quadratics femoris,
Lateral Rotation of Hip
Superior and Inferior Gamelli

:
Nerve Supply
Superior Gluteal Nerve - Gluteus minimus, medius
Inferior Gluteal Nerve - Gluteus maximus
: Nerve to quadratus femoris - Quadratus femoris, Inferior Gemelli
Nerve to obturator Internus - Obturator Internus, Superior Gemelli
: Nerve to Piriformis - Piriformis

In the clinical question given, the patient complaints of dragging of leg while walking.
: This is referred to as Trendelenburg sign and is seen in paralysis of Gluteus
minimus and Gluteus medius due to Superior gluteal Nerve injury.
-
There is sinking of hip in the non - affected side resulting in dragging of leg.
11. Identify the pelvic diaphragm in the following image
(A) B
A
(B) C
B
(C) A
(D) D C

Pelvic Diaphragm
<
Pelvic diaphragm is a muscular bed which gives support to the pelvic viscera and closes
the pelvic outlet.
It is formed by the levator ani and coccygeus.
Levator Ani - Pubococcygeus (Pubovaginalis, Puboanalis, Pubococcygeal Proper)
Puborectalis
Iliococcygeus
Nerve Supply is by pudendal nerve (Root : S2, S3, S4)
The pelvic diaphragm is a part of the perineal body.
Perineal body has 3 strata
Superficial : Bulbospongiosus + Transverse Supericial Perinei +
: External anal sphincter + Ischiocavernous
Intermediate : Transverse perinei profundus + External Anal Sphincter (deep part)
Deep : Levator ANI Muscle (Pubococcygeus Part)

:
Ischiorectal Fossa
Boundaries
Anterior : Transverse Perinei Superficialis
: Posterior : Sacrotuberous Ligament
Medial : Levator ANI Muscles, Fascia Over Levator ANI
: Lateral : Obturator Internus, Fascia over Obturator Internus & Alcock’s canal
Contents
: Inferior Rectus Vessels, Nerve
Internal Pudendal vessels
: Pudendal Nerve
Posterior Scrotal vessels and Nerve (In Males)
: Posterior Labial nerve and vessels (In Females)
Alcock’s Canal (Pudendal Canal)
Seen in lateral wall of ischiorectal fossa.
: Contains internal pudendal vessels and pudendal nerve.

12. Which of the following structures is marked wrong?


(A) Cilia
(B) Basement membrane A
Doodle

(C) Goblet cells


(D) Muscle layer

'
The given image is of C
-708bar
B
armament
Respiratory epithelium.
D
00008000008bar

Respiratory Epithelium
It is a type of ciliated columnar epithelial lining most of the epithelial tract.
: It functions as a barrier to potential pathogens and foreign particles by secreting mucus and
its mucociliary clearance.

a
Layers of Respiratory Epithelium

13. The marked cell in the picture given below is inhibitory to which of the following?
(A) Golgi cell
(B) Granule cell
(C) Basket cell
(D) Deep nuclear cell

c-
Given image is the histological
image of cerebellum.

Internal Structure of Cerebellum


Outer molecular layer contains

: dendrites of purkinje cells, basket


cells and Stella the cells.
Middle Purkinje layer contains
Purkinje cells.
Inner granular layer contains axons

: of purkinje cells, Golgi cells and


granule cells.
The purkinje cells carry the output from
cerebella cortex to the cerebella nuclei.
Inputs to cerebellum
Climbing fibres (from inferior olivary nucleus) and Mossy fibres.
: Climbing fibres directly go to Purkinje cells.
Mossy fibres from branches which join granule cells and Golgi cells to form Cerebelloglomeruli.
: Only excitatory cell in Cerebellum : Granular Cell
Rest all cells are inhibitory.
: Cerebellar Nuclei : Dentate, Emboliform, Globose, Fastigial
Physiology
14. An anion is having the following concentrations : 100 mEq/l in ECF and 10 mEq/l in ICF.
How much is the equilibrium potential for this ion?
(A) +10 mV
(B) -10 mV
(C) +61 mV
(D) -61 mV
Nernst Equation
Used to calculate equilibrium potential of single ion.
: Ex = +/- 61 log [inside concentration]
[outside concentration]
Ex = 61 log [10] = 61 log [10 -1 ] = -61 mV

:
[100]

Goldmann’s equation is used to calculate equilibrium potential for multiple ions.


Gibbs Donnan Effect
Protein anion have negative charge and can’t move across
the membrane which affects the distribution of
permanent ions (Na+, K+).

15. All of the following will shift the O2 dissociation curve to the right except
(A) Increase in H+
(B) Increase in pCO2
(C) Increase in HbF
(D) Increase in 2,3 - BPG

Oxygen Dissociation Curve

Oxygen dissociation curve is the curve between O2 saturation and pO2.


It is sigmoid shaped because of the theory of cooperativity.
: First molecule of O2 binds with difficulty to Hb, the rest 3 molecules bind easily.
Factors causing left shift of ODC
Haldane Effect
: Decrease in CO2
Decrease in H+
: Hypothermia
Decrease in 2,3 - BPG
: Foetal Hb
Factors causing right shift of ODC
Bohr Effect
: Decrease in O2
Increase in CO2
: Increase in H+
Hyperthermia
: Increase in 2,3 -BPG
High altitude excercise

:
16. All of the following will increase gastric acid secretion except :
(A) Histamine
(B) Serotonin
(C) Acetyl Choline
(D) Gastrin

Mechanism of HCL release

:
There are 3 types of receptors on parietal cells : H2, M3, CCK - B12.
G cells present in the wall of stomach produces Gastrin which act as a ligand for CCK - B12
receptor and it also stimulates Enterochromaffin cells to secrete histamine.
Histamine acts on H2 receptor and activates it.
The vagus nerve releases ACh to the M3 receptors activating them.
: Activated H2, M3 and CCK - B12 receptor stimulate H+ pump of parietal cells causing HCL
release.
D cells release somatostatin (antagonist of Gastrin).
: H.pylori destroys D - cells, hence somatostatin production is decreased resulting in high
gastrin levels leading to increase in HCL production and Gastric Ulcer.
Factors that increase Gastrin
Distension of stomach
: Gastrin Releasing Peptide (GRP or Bombesin)
Amino acids or peptides

:
Factors that decrease Gastrin
Gastric Acid
Somatostatin
: Secretin
Gastric Inhibitory Peptide

:
17. All of the following have Na-I symporter (NIS), except:
(A) Placenta
(B) Pituitary
(C) Thyroid
(D) Salivary gland
Sodium - Iodide Co - transporter (Sodium - Iodide Symporter/NIS)
A transmembrane glycoprotein encoded by SLC545 gene.
It is seen in thyroid, stomach, salivary gland, lactating mammary glands, placenta, ovaries.
: It is involved in the first step of thyroxine hormone synthesis (Iodide Trapping)

Steps of Thyroid hormone Synthesis


i) Iodide trapping
ii) diffusion of Iodide to apex of cell
iii) Iodide transport to colloid
iv) Oxidation of Iodide to Iodine and Incorporation (Iodination) into tyrosine
residue (MIT,DIT).
v) DIT + DIT = T4, MIT + DIT = T3 (Conjugation)
vi) Uptake of thyroglobulin into follicular cells and fusion with lysosomes, proteolysis and
release of T3 and T4 into circulation.
18. Calculate the following frequency of the skeletal muscle from the given data: Latent
period - 10msec, Contraction period - 40msec, Relaxation period - 50msec.
(A) 20/sec
(B) 25/sec
(C) 35/sec
(D) 50/sec
Tetany

Tetany is defined as the sustained contraction of a muscle due to increased frequency of stimuli.
Tetanising frequency of a muscle = 1
(Tf) Contraction period of muscle (s)
Tf = 1 = 25 s
40ms

A
LP = Latent Period
CP = Contraction Period
RP = Relaxation Period

1
Tf =
CP(s)

Stimulus

A B C D
LP CP RP

19. In the PCT of kidney, luminal concentrations of all of the following will decrease, except:
(A) Chloride
(B) Bicarbonate
(C) Glucose
(D) Amino acids

Parts of Nephron and its function

Glomerulus and Bowman’s Capsule


Glomerulus is formed by afferent and efferent arterioles and acts as the filter of
nephron through which plasma is filtered into the Bowman’s capsule.
PCT
Site of mass absorption.
:
70% Na+, Cl-, K+ absorbed here.
100% glucose, Amino acids absorbed.
: Urea (50%) and HCO3 (80%) also absorbed.
Has brush bordered epithelium to increase area of absorption.
: Fluid in PCT is isotonic in nature.

Loop of Henle
Site of counter - current mechanism.
: Takes part in concentration and dilution of urine.
Descending Loop of Henle is permeable only to water (via Aquaporin - 1), hence it is
: known as concentrating segment/countercurrent multiplier).
Thick ascending limb is permeable to solutes (has Na+_K+_2Cl- Cotransport and
is called diluting segment.

Distal Convoluted Tubule


Site for Na+, Ca2+, and Cl- reabsorption.
: Has Na+_Cl- cotransporter and transient receptor potential Vannilloid Channel 5 for
Ca2+ transport at the apical end.
: Na+_Ca2+ exchanger is present at the basolateral end.

Collecting Duct
Site for action of ADH, Aldosterone, Atrialnatriuretic Peptide.
: Aquaporin - 2 channels present here for water reabsorption.

20. At 47 degree Celsius atmospheric temperature, heat loss from body will occur mainly by:
(A) Conduction
(B) Convection
(C) Radiation
(D) Evaporation
Heat Loss from the body
More than 50% of heat loss from body occurs by radiation under normal
: circumstances. (When body temperature > Atmospheric Temperature)
Only about 5% of heat loss occurs through conduction.
In the given question, the atmospheric temperature is 47°C which is much more
: than the body temperature.
In such cases, there wont be heat loss from the body and heat is gained by the
body as it is at a lower temperature.
So the body can lose heat only by evaporation through sweating in this case.
: Regulation of body temperature is done by anterior and posterior nuclei of hypothalamus.
In a hot environment, anterior nuclei initiates sweating, thirst and vasodilatation.
: In a cold environment, posterior nuclei initiates shivering, vasoconstriction and increases
frequency of urination to conserve heat.

21. A patient was brought to the emergency room with hypoxemia. The alveolar-arterial gradient (A-a
gradient) was found to be normal. Which of the following is the likely cause?
(A) Right to left shunt
(B) Hypoventilation
(C) V/Q mismatch
(D) Hyaline membrane disease

Alveolar Arterial Gradient


Alveolar - arterial O2 gradient (A-a gradient) is the difference in pO2 between alveoli and
: blood in arteries.
The alveoli contain high amount of O2 (same as atmospheric O2) and low levels of CO2.
The deoxygenated blood brought to the lungs contains low O2 and high CO2.
: As a result, the O2 in alveoli diffuse into the blood to make it oxygenated.
This A - a gradient can be affected by various factors.
: Eg : High A - a gradient is seen in respiratory membrane pathology.

Among the given options,


i) Right to Left Shunt - Results in bypassing of blood from venous to arterial side

: without oxygenation.
As a result, the arterial O2 decreases and A - a gradient increases.
ii) V/Q mismatch - Here the lung is ventilated properly but is not perfumed well.
As a result, arterial blood doesn’t get oxygenated properly and hence
A - a gradient is increased.
iii) Hyaline Membrane Disease - The process of diffusion of gases is affected here, so

: oxygenation of blood doesn’t take place properly and so A - a gradient is increased.


In hypoventilation, there is no defect in oxygenation of blood. Hence A -a gradient remains normal.
But because of decreased ventilation, there is decrease in O2 content in alveoli and arterial blood
resulting in hypothermia.
22. MET equivalent of 1500ml of oxygen consumption per minute for healthy 70kg adult male
with basal oxygen consumption is 250ml/min.
(A) 1/6
(B) 6
(C) 4
(D) 1/4
Metabolic Equivalent
1 Metabolic Equivalent is defined as the amount of oxygen consumed while at rest and is equal to
:3.5 ml/kg body weight/min.
For a 70 kg adult, 1 MET + 3.5 X 70 = 245 ml/min (250 ml/min) as given in question.
When O2 consumption has increased to 1500 ml/min.
: MET Equivalent = 1500 = 6 MET
250

23. Which of the following sensation is detected by Pacinian Corpuscle?


(A) Pressure
(B) Touch
(C) Temperature
(D) Pain
Touch Receptors

Touch Receptors are of 4 types :


Meissners’s Corpuscles
: Merkel Cells
Pacinian Corpuscles
: Ruffini’s Endings
Meissner’s Corpuscles Pacinian Corpuscles
9
Superficial in location (dermo - epidermal junction). 5
Present deep in dermis.
5
Most abundant. 5
Detects high frequency pressure.
Detects fine touch, low frequency vibration.

:
Merkel Cells
Superficial.
Ruffini’s Endings
-

Abundant in Joint spaces.


g
Projecting into epidermis. -
Detects Pressure Sensation.
Sensitive to sharp edges (Braille Reading System).

:
Meissner’s Corpuscles and Pacinian corpuscles are rapidly adapting receptors.
They adapt rapidly to given stimulus.

24.Maximum airflow resistance is seen in:


(A) Respiratory bronchioles
(B) Bronchus
(C) Alveolar duct
(D) Terminal bronchioles
Resistance in airways

:
Resistance is maximum with medium
sized airways.
Gen 0 to 2nd - Large Sized
Gen 3rd to 5th - Medium Sized
Gen 6th to 23rd - Small Sized
-

Resistance depends on the radius of airways and the type of flow in the airways.
Resistance is inversely proportional to r 4.
: Resistance is directly proportional to Turbulence.
Turbulence decreases as the airways decrease in size.
: In large sized airways, both turbulence and radius is high.
In small sized airways, turbulence and radius is low.
: But in medium sized airways, turbulence is present and radius is smaller compared to large
airways hence resistance is more.

25. Myocardial contractility was found to have increased in a person. What will be its effect on
pressure - volume curve of heart?
(A) Decrease in slope of end diastolic pressure volume curve
(B) Increase in slope of end systolic pressure curve
(C) Left shift of end systolic pressure volume curve without change in slope.
(D) Left shift of end diastolic pressure volume curve without change in slope.

Pressure - Volume Loop of Cardiac Cycle

" " " " " " * " "" """ " "^
"

i
,
y
i

÷
i - -
End diastolic Pressure volume
-
r
- - -
-
- -
-
-
Relation
-

(Stope)
- -
-
-
-
-
-

Normal Pressure - Volume loop


Of Cardiac Cycle
Pressure volume loop is based on the pressure changes and volume changes in the ventricles.
: The whole cardiac cycle is represented in this loop.

Ejection fraction can be calculated from this loop.
Ejection Fraction
EF = Stroke Volume X 100
End Diastolic Volume
= End Diastolic Volume - End Systolic Volume
X 100
End Diastolic Volume

Change in P - V loop in Valvular Heart Diseases

Contractility X Slope of
of Heart End - Systolic
Pressure Curve

Wiggers’ Diagram

In case of increased contractility, the end systolic pressure volume relation (slope) will increase.
: In case of decreased contractility, the end systolic pressure volume relation (slope) will decrease
and loop will shift to right.
Cardiac Cycle

Total Duration : 0.8s


Ventricular Systole
0.3 s
:Isovolumetric Contraction
Rapid Ejection Phase
Slow Ejection Phase

:
Ventricular Diastole
0.5 s
Protodiastole
: Isovolumetric Relaxation
Rapid Filling Phase
: Slow Filling Phase
Active Filling Phase Cardiac Cycle
Biochemistry
26.Wobble hypothesis of genetic code is an explanation for?
(A) Degeneracy
(B) Unambiguity
(C) Universality
(D) Non punctuated nature of code

Genetic Code
Genetic Code is a sequence of 3 nucleotide bases that code for a particular amino acid.

Properties of genetic code


/

Triplet Code
3 nucleotide bases are required to code for an AA.
/

Since there are 4 bases present (A,U,G,C) 64 unique codons are possible.
: 3 are stop codons, 61 codons together translate 20 AA.
/

Degeneracy
Multiple codons can code for same AA.
: Wobble hypothesis
Crick (1996) proposed the 'wobble hypothesis' to explain the degeneracy of the genetic code.
Except for tryptophan and methionine, more than one codons direct the synthesis of one
amino acid.
There are 61codons that synthesis amino acids,therefore, there must be 61 tRNAs each
having different anticodons.
But the total number of tRNAs is less than 61.
'

Unambiguos
Translation will be fixed every time.

i
C1 C2 C3 C4 C5
- - -

5’ - AUG - CAG - CAA - GCC - ACC - 3’


C1X
Non - overlapping
Non - punctuated
/

Universal : Same in all eukaryotes


27. Most reliable method for HbA1c estimation
(A) Ion exchange chromatography
(B) Agarose gel electrophoresis
(C) Affinity chromatography
(D) Gas chromatography

Gold Standard method for HbA1c is High Performance Liquid Chromatography.


: Other methods of HbA1c estimation :
- Capillary electrophoresis
- Enzymatic Digestion
- Ion Exchange Chromatography
Among the given options, Ion exchange chromatography is most suited.

:
28. Which of the following compounds is not glucogenic?
(A) Oxaloacetate
(B) Pyruvate
(C) Lactate
(D) Acetyl CoA

Acetyl CoA is purely ketogenic.


Pyruvate is both gkucogenic and ketogenic.
: Oxaloacetate, Citrate, Succinylcholine CoA, Fumarate are glucogenic.

Non - Carbohydrate Substrates for Gluconeogenesis


Glucogenic Amino Acids (Alanine) : Cahill Cycle
:
Glycerol
Lactate (Cori’s Cycle)
: Propionate CoA
Site of Gluconeogenesis
Liver, Kidney
: Organelle : Both Cytoplasm and Mitochondria

Key enzymes for Gluconeogenesis


Glucose - 6 - Phosphate
:
Fructose - 1,6 - bisphosphate
Pyruvate Carboxylase
:
Phosphoenol Pyruvate Carboxylase

Gluconeogenesis

Ketone Body Synthesis


Site : Liver
: Organelle : Mitochondria
g

Rate Limiting Enzyme : HMG CoA Synthase


Ketone Synthesis

29. In which of the following deficiencies there is collagen maturation defect?


(A) Beri beri
(B) Vitamin A deficiency
(C) Scurvy
(D) Rickets
Vitamin deficiency (scurvy) can cause bleeding gums, petechiae and Ecchymosis.

:
Vitamin C
Water soluble.
Not synthesised in humans (because of lack of L - Gulonolactone Oxidase).

:
Functions
Acts as coenzyme for hydroxylases.
s
Therefore deficiency can lead to defective hydroxylation of proline and lysine
resulting in defective collagen, thereby causing bleeding manifestation.
s

Needed by ferroreductase to convert Fe3+ to Fe2+ which is absorbed from


intestine.
s

Hence deficiency can cause Iron deficiency anaemia.


Clinical Manifestations of Deficiency
Haemarthrosis
: Scourbutic Rosary
Pseudoparalysis
: Bleeding Gums,
Petechiae

:
Other Vitamin deficiencies
Vitamin D - Rickets
Vitamin A - Night blindness, Decreased Collagen Synthesis, Xerophthalmia,
: - Increased susceptibility to infections
Vitamin B1 (Thiamine) - Beriberi
Vitamin B2 (Riboflavin) - Cheilitis, Stomatitis, Glossitis, Corneal Neovascularisation
: Vitamin B3 (Niacin) - Pellagra (Diarrhea, Dementia, Dermatitis, Death)
Vitamin B6 (Pyridoxine) - Peripheral Neuropathy, Neonatal Seizures
: Vitamin K - Bleeding Manifestations, Defective Wound Healing
- Recurrent Oral Ulcers

30. Which is most appropriate for the quantitative analysis of viral genomes?
(A) Real-time PCR
(B) RT-PCR
(C) RT-PCR followed by real time PCR
(D) Conventional PCR with restriction fragment length pair technique
Steps of PCR
(i) Denaturation
(ii) Annealing
(iii) Extension
(iv) Analysis of amplified specimen using electrophoresis
DNA polymerase enzyme use for extension is Taq polymerase isolated from Thermus Aquaticus.
: No : of amplified specimens = 2 n(n = no: of cycles)
Discovered by Kary B Mullis.

:
Steps of RT - PCR (Reverse transcriptase PCR)
Used to detect/quantify any kinds of RNA.
Here we use Tth polymerase derived from Thermus thermophilus which has two enzyme
:activity : Reverse transcriptase + DNA polymerase.
It is used to estimate viral load.
Real Time PCR
-
Also known as quantitative PCR.
Starting material is DNA.
: Used for simultaneous amplification + detection/quantification of amplicon.
Here specific dyes/probes are used to detect the quantity of amplified material.
: Eg : Ethidium Bromie, SYBR (Green Dye), Taq man probe, Molecular beacon, FRET probe.
Multiplex PCR
In this type, a single reaction mixture has multiple targets which are amplified.
: It is less time consuming but non - specific.
It makes use of restriction fragment length pair technique.

:
Nested PCR
Product os first set of primers contain target for 2nd set of primers.

In the given question, the target to be amplified is a viral genome.


Hence RT - PCR is required for amplification and real time PCR for further quantification.
31. An unconscious patient is brought to the casualty with history of not having food for more than 2
days. Which of the following tests will be positive for this patient?

(i) (ii) (iii)

(A) i, ii, iii


(B) ii, iii
(C) only ii
(D) only iii
In the given question, patient presents with history of starvation for more than 2 days which can
result in production of ketone bodies.

Stages of fasting
>
Early Fasting ( 4 - 16 hours) - Glycogenolysis
>
Fasting ( 16 - 48 hours) - Gluconeogenesis, Fatty Acid Oxidation
>
Starvation (2 - 5 days) - Ketone body synthesis
>
Prolonged Starvation - Muscle Proteolysis (Cachexia)
Test for Ketone Bodies

Rothera’s Test
Positive test gives a purple ring.
: Positive in case of acetone and acetoacetate.
Negative in case of Beta hydroxy Butyrate.

:
Gerhardt’s Test - Positive only in acetoacetate.
Ketostix - Dip stick test to detect ketone bodies.

Test for Carbohydrates


Molisch Test

I
Positive for all types of carbohydrates.
Purple violet ring is due to formation of furfural compounds.

Iodine Test
-

Detects starch in specimen.


-

Gives blue black colour.


Benedict’s Test
Detects reducing sugars only and gives an idea about the concentration of reducing

:
sugars through colour.

Colour Inference
Blue Solution No sugar
Green Solution 0 - 0.5 mg%
Green Precipitate 0.5 - 1.0 mg%
Yellow Precipitate 1.0 - 1.5 mg %
Orange Precipitate 1.5 - 2.0 mg%
Brick Red Precipitate > 2.0 mg%

Barfoed’s Test Seliwanoff Test


Detects reducing monosaccharides. Detects ketogroup (Aldehyde
-

Group in Monosachharides)

Test for Proteins


Heat Coagulation Test
: Cloudy precipitate formed on heating proteins.

32. Thiamine deficiency leads to lactic acidosis. Which enzyme action is inhibited in this case?
(A) Pyruvate dehydrogenase
(B) Pyruvate carboxylase
(C) Pyruvate kinase
(D) Phosphoenolpyruvate carboxykinase

Thiamine (Vitamin B1) acts as a coenzyme for various enzymes.


Active form of thiamine is Thiamine Pyrophosphate (TPP).
Enzymes requiring TPP
Pyruvate Dehydrogenase
: Alpha ketoglutarate dehydrogenase
Branched Chain ketoacid dehydrogenase
: Transketolase

Pyruvate Dehydrogenase
Pyruvate dehydrogenase is a multienzyme complex made of
: Pyruvate Dehydrogenase
Dihydrolipoamide transacetylase
: Dihydrolipoamide dehydrogenase
Coenzymes of PDH : TPP, Lipoamide, FAD, NAD, CoA
: Pyruvate PDH
Acetyl CoA
TPP

+
NAD NADH
NADH formed is used for ATP generation through ETC.
: If deficiency of PDH is present, lactic acid is formed instead of Acetyl CoA.
Pyruvate -
Lactate (Lactic Acidosis)
LDH

33. Assimilation of which of the following requires maximum energy expenditure?


(A) Poly unsaturated fatty acid
(B) Mono unsaturated fatty acid
(C) Proteins
(D) Carbohydrates
Assimilation of food
Assimilation of food refers to the thermic effect of food (TEF)
: Also referred to as the Specific Dynamic Action (SDA) or Dietary Induced Thermogenesis (DIT).
It is the amount of energy expenditure above the basal metabolic rate required for digestion,
: absorption, and disposal of ingested nutrients.
It’s magnitude depends on the composition of food consumed.
Carbohydrates : 5% - 15% of the energy consumed.
: Protein : 20% - 30%
c'

Fats : 5% - 15%
c'

Raw celery and grapefruit has a negative caloric balance, i.e., it requires more energy to digest than
recovered from the food itself.
Factors affecting thermic effect of food
Total caloric content of meals and the macronutrient composition.
: Marginally increased by both aerobic training and anaerobic weight training of sufficient
duration and intensity.
Meal frequency has little to no effect on TEF.

:
34. True about mitochondrial DNA:
(i) Higher chances of mutations as compared to nuclear genome
(ii) Single stranded circular DNA
(iii) Mitochondrial codons are slightly different from nuclear DNA codons
(iv) Mitochondrial DNA codes for all the ETC complex proteins
(v) It has around 16000 base pairs

(A) i, iii, v
(B) i, ii, iv
(C) ii, iii, v
(D) ii, iv, v
Mitochondrial DNA
Mitochondrial DNA constitutes 1% of cellular DNA.
It is double stranded, circular and has about 16,569 base pairs.
: It encodes for 37 structural genes :
2 rRNAs (16 SrRNA, 12 SrRNA)
: 22 mitochondrial tRNAs
13 proteins of ETC (19% of total)
:
7 subunits of Complex I
Cyt b of Complex III
:
3 Subunits of Complex IV
2 subunits of ATP Synthase
:Mutation rate is very high because :
No introns.
: No protective histones.
No effective repair enzymes.
: Exposure to oxygen free radicals generated by oxidative phosphorylation.
9
It follows a non - Mendelian type of inheritance (maternal inheritance).
35. Which of the following lipids is not an amphipathic compound
(A) Glycophospholipid
(B) Sphingophomyelin
(C) Triacyclglycerol
(D) Phosphoglycerol
Amphipathic Compounds Non - Polar Compounds
h

Phospholipids Cholesterol Esters


h

Cholesterol Triacyl Glycerols


h

: Apolipoproteins

Triacyl Glycerol (Neutral Fat) is the storage form of lipids in humans.

Compound Lipids
Fatty Acid + Glycerol + Other Groups
PO4 - Phospholipid
Other SO4 - Sulfolipid
Groups C - Glycolipid
Protein - Lipoprotein

36. CRISPR/Cas9 gene knockdown technology is used in which of the following DNA repair
(A) Base excision repair
(B) Nucleotide excision repair
(C) Non homologous end joining repair
(D) Base excision repair

CRISPR/Cas9

CRISPR/Cas9 gene is a defence mechanism in bacteria against phases.


: It codes for the enzyme Endonuclease Cas 9.
This enzyme is used in cleaving mutation sites in Human DNA (double stranded breaks).
: Hence it helps in eliminating the problematic gene and replace it with the desired gene.
Once the problematic gene is eliminated,

:
We can join the double stranded DNA breaks end to end (Non - homologous end
joining) : GENE Knockouts.
OR
We can insert a donor DNA (Homology - directed DNA repair) : GENE Knockins

37. Which of the following statements is not true about suicidal inhibition?
(A) The inhibitor binds to the active site where it is modified by the enzyme to produce a reactive group
that reacts irreversibly to form a stable inhibitor-enzyme complex.
(B) It is an irreversible form of enzyme inhibition that occurs when an enzyme binds to a substrate
analogue and forms an irreversible complex with it through a covalent bond.
(C) Inhibitor binds to all sites resulting in irreversible inhibition.
(D) Inhibitors are unreactive until within the enzyme’s active site.

Suicide Inhibition
It is a mechanism based inhibition.
It is an irreversible form of enzyme inhibition (‘suicidal’).
: The inhibitor forms an irreversible complex with the enzyme through a covalent bond.
The inhibitors are inactive till they bind to the active site of the enzyme.
: Eg : Allopurinol Inhibiting Xanthine Oxidase
Aspirin Inhibiting Cyclooxygenase
Difluoromethylornithine inhibiting Ornithine Decarboxylase.
Inactive
Compound Potent

O O
(Inhibitor) Inhibitor
Using mechanism of action (Irreversibly
E E
of Enzyme Bind to
Active Site Enzyme)

Other Types of Inhibition


Competitive Inhibition

I I
Inhibitor and Substrates are structural analogues.
Both bind reversibly to the same site on the enzyme.
During this inhibition, Km increases but Vmax remains constant.

-
Eg : Succinate dehydrogenase and Malonate
7-

HMG CoA reductase and Statins


7-

Vit L Epoxide Reductase and Dicoumarol


7-

DHF Reductase and Methotrexate


7-

Non - Competitive Inhibition


Here substrate and inhibitors are not structural analogues.
: Inhibitor binds to separate site on the enzyme.
Vmax decreases, but Km remains constant.
:
Eg : Cyanide and Cyt C Oxidase
: Disulfiram and Aldehyde Dehydrogenase
Fluoride and Enolase
: Arsenide and Alpha ketoglutarate Dehydrogenase
Uncompetitive Inhibition

O_0 E S E S

Inhibitor Inhibitor

Inhibitor can’t bind with Inhibitor Binds to


free enzyme Enzyme - Substrate complex

Km and Vmax, both are different.


:
Eg : Placental Alkaline Phosphatase and Phenyl Alanine)
38. Choose the correct statements for a patient with advanced liver disease:
(i) Hyperammonemia
(ii) Low ATP production
(iii) Conversion of alpha-keto glutarate to glutamate and then to glutamine is increased.
(iv) Alpha-keto glutarate undergoes oxidative decarboxylation.

(A) All are correct


(B) i & ii
(C) ii & iii
(D) i, ii & iii
In case of advanced liver disease / liver failure, conversion of NH3 to Urea (Urea Cycle)
: doesn’t take place in the liver.
As a result, NH3 accumulates in the body, especially in the neutrons of brain where they
bind all the H+ ions to form NH4+.
Because of H+ ion depletion, there is decreased ATP production via the ETC.
: Alpha ketoglutarate is the scavenger of NH3.
It binds to form Glutamate.
: Glutamate binds with NH3 to form Glutamine (carrier of NH3).
Glutamate
Alpha Ketoglutarate + NH3 Glutamate + NH3 Glutamine
Synthase
: This causes depletion of Alpha - ketoglutarate which is a part of TCA cycle and hence it
affects ATP production resulting in low energy.
39. True statement about intracellular regulation of iron metabolism:
(A) Transferrin receptor 1 translation is decreased by binding to iron regulatory protein to 3’ UTR
(B) Transferrin receptor 1 translation is increased by binding to iron regulatory protein to 5’ UTR
(C) Ferritin translation is decreased by binding to iron regulatory protein to 5’ UTR
(D) Ferritin translation is increased by binding to iron regulatory protein to 5’ UTR

Iron Metabolism
C
Approximately 2 mg of iron is absorbed daily in the duodenum and proximal jejunum.
The human body controls iron levels by regulating iron absorption.
: Iron is usually present in Ferric form in diet (Fe3+) which is reduced to Fe2+ for absorption (done
by membrane bound Ferric reductase duodenal Cytochrome B on Apical brush border membrane).
Fe2+ ions are transported into the cells by DMT - 1 where it is stored as Ferritin.
: To reach the systemic circulation, Ferroportin 1 is used.
Fe2+ in circulation is oxidised to Fe3+ by Hephaestin.
: Fe3+ is transported through Transferrin.

Intracellular Regulation of Iron Metabolism

Iron is transported in the circulation bound to transferrin (TFR1).


: It can also exist in non - transferrin bound form, especially when serum Iron levels are high and
transferrin is saturated.
Transferrin receptor is expressed ubiquitously on the cell surface and takes up transferrin bound

: iron through receptor mediated endocytosis.


Iron is taken to the cytoplasm from the endosome via DMT - 1.
b
The levels of iron metabolism related proteins (Ferritin, TFR -1, DMT -1, etc) are regulated
at the post transcriptional level via the iron responsive element/iron responsive protein (IRE/
IRP) system.

In case of Iron excess, IRP IRP binds to IRE in the 3’ untranslated


binds to 5’untranslated region regions (UTR)during conditions of
(UTR), blockingtranslation iron deficiency and stabilise the
and decreasing iron uptake. mRNA, enhancing translation of the
proteins and increasing Iron uptake.
Pathology
40. Which one of the following is a characteristic finding seen in Langerhans cell histiocytosis?
(A) Granulomas
(B) Eosinophils
(C) Birbeck granules
(D) Histiocytes
Langerhans cell histiocytosis

Langerhan’s cell histiocytosis is a rare multisystem disorder characterised by clonal

:
proliferation of Langerhan’s Cell.

Langerhan’s Cell
It is an antigen presenting cell which arise from the bone marrow.
It is located in the stratum spinosum layer of skin.
: It contains Birbeck granules (Tennis Racquet shaped)
IHC Marker : s100, CD1a, HLA - DR, CD 207 (Langerin) - Most Spcific Marker

: Langerhan’s Cell Histiocytosis has different presentations.


It is associated with BRAF (V600E) Mutation.

:
(Like colon adenocarcinoma, melanoma, Hairy cell leukaemia,
Papillary Thyroid Cancer, Pilocytic astrocytoma)

Various presentations of LCH

Letterer - Sieve Disease


Hand - Schuller - Christian Disease
: Eosinophilic Granuloma
Letterer - Sieve Disease
Seen at 2 years of age.
: Characterised by lyric bone lesions, liver involvement, skin lesions.
Bone lesions are seen in skull, spine and mouth.

:
T
<
Multifocal, Lytic,
Geographical lesions
with bevelled margins
Vertebra Plana

Hepatosplenomegaly and Lymphadenopathy present.

Hand - Schuller - Christian Disease


Seen in children < 5 years of age.
Eosinophilic Granuloma
a
Floating Teeth

Skin lesions include Seborrhic dermatitis, Erythematous Pustular lesions.

Seen in age < 8 years.


c-
<

Calvarial bone defects a

Presents as solitary bone lesion.


Diabetes Insipidus
÷
Exophthalmous

41. The following image shows the cut section of the liver of a patient who died due to heart failure.
Based on the specimen which of the following is true?
(A) Chronic venous congestion of Liver; dark area viable and grey area necrosed.
(B) Chronic venous congestion of Liver; dark area necrosed and grey area viable.
(C) Amyloidosis of liver.
(D) Hepatocellular carcinoma.
The given image in the question shows the nutmeg appearance of liver typically seen in
: chronic venous congestion of liver.
It is seen during right heart failure resulting in congestion of blood in the systemic organs.
The hepatocytes are arranged into hepatic lobules in which the portal vein, hepatic artery

:
and bile duct take centre position as central vein.

The liver receives blood supply through portal vein and hepatic artery and blood from liver
is drained by hepatic veins into the inferior vena cava.
Because of chonic venous congestion, there is back pressure on the hepatic veins (central

: veins) resulting in necrosis of the central region which appears dark in colour (centrilobular
necrosis and periportal fibrosis).
The periportal region remians viable and is grey in colour.
42. Clear cell carcinoma of ovary is seen in
(A) Epithelial tumor
(B) Metastasis
(C) Germ cell tumor
(D) Granulosa cell tumor
Classification of Ovarian Tumours

Epithelial tumors Germ cell tumors Sex cord-stromal tumors


1

Serous ,

Teratoma Granulosa-stromal cell tumors


-

Mucinous Mature Granulosa cell tumor


-

l l

Endometrioid Immature Fibrothecoma


-

l
l

Clear cell l

Dysgerminoma Sclerosing stromal tumor


-

Brenner l

Endodermal sinus tumor Sertoli-stromal tumors


-

Undifferentiated n

Embryonal cell carcinoma Sertoli-Leydig cell tumor


-

Choriocarcinoma Steroid cell tumors


-

Other tumors
-

Metastatic tumours
43. Splenomegaly is not a feature of
(A) Lymphoma
(B) Aplastic anemia
(C) Hairy cell leukemia
(D) Chronic myeloid leukemia
Causes of Splenomegaly Causes of Massive Splenomegaly
Haematological
Red Cell Membrane defects Thalassemia Major
Haemoglobinopathies
Autoimmune Haemolytic
Anaemias
Rheumatologic

Rheumatoid Arthritis
Systemic Lupus Erythematosis
Sarcoidosis
Infectious
Viral Visceral Leishmaniasis
Bacterial Hyperreactive Malarial
Mycobacterial Splenomegaly Syndrome
Fungal Mycobaterium avium complex
Parasitic
Congestive
Hepatic Cirrhosis
Venous Thromboses
(Hepatic, Portal, Splenic)
Infiltrative
Lymphomas Lymphomas
Myeloproliferative Neoplasms Myeloproliferative Neoplasms
Metastatic Cancer Gaucher’s Disease
Amyloidosis
Gaucher’s Disease
Niemann - Pick Disease
Glycogen Storage Diseases
Haemophagocytic Syndrome
Langerhan’s Cell Histiocytosis
1
Splenomegaly is usually seen in infections, blood disorders (RBC defects, leukemias,
lymphomas), storage disorders and in congestive conditions (liver cirrhosis, heart failure,
venous thrombosis).
Aplastic Anaemia is a condition in which there is complete absence of both intra and extra
: medullary hemopoiesis. Hence there is no splenomegaly in this case.
Other options :
Lymphoma and leukemia presents with massive splenomegaly.
: Hairy cell leukaemia has bone marrow infiltration resulting in
extra medullary haemopoiesis presenting as splenomegaly.

44. The peripheral blood examination of a patient showed lymphocytosis with smudge cells. Which
is the next best investigation to make a definitive diagnosis?
(A) FISH
(B) Flow cytometry
(C) Bone marrow biopsy
(D) Cytogenetics

The given clinical history is suggestive of Chronic Lymphocytic Lymphoma.

Chronic Lymphocytic Lymphoma


Also known as Small Lymphocytic Lymphoma.
: It is the most common type of chronic leukemia in elderly.
Typically seen in 6th or 7th decade.
: Clinically presents with lymphadenopathy, hepatosplenomegaly, anaemia (autoimmune
haemolytic anemia)
Cytogenetics : Deletion of Chr 13q (MC); Deletion of Chr 11q, 17p.
: Classification of CLL is by Rai and Binet staging system.
CLL can transform into diffuse large B - cell lymphoma : Richter’s Syndrome

:
Bad Prognostic Markers
Raised Serum β2 - microglobulin, CD - 23, LDH
Presence of NOTCH - 1 mutation, 17p deletion, 11q deletion
: Increased expression of ZAP - 70.
Lab diagnosis
CBC - Hb Normal, TLC inrceased, Platelet Normal or decreased.
: 3
(Diagnostic Criteria : Absolute Lymphocytosis > 5000/mm )
Peripheral Smear : Increased number of small lymphocytes,
smudge cells/ basket cells / parachute cells.

I
Lymph Node biopsy
Normal architecture is replaced by large number of small cells with inconspicuos

÷
nucleoli, proliferation centres containing activated lymphoid cells are present between
them.
Flow Cytometry
CD19+, CD20+
CD5+ Helps to differentiate from mantle cell lymphoma
CD23+ which has CD5+ but CD23-
Tdt - Implies that the involved cells are not lymphoblasts.
: CD 34-
Flow cytometry helps us arrive at a definitive diagnosis and rule out
important differential diagnosis.

45. A child presents to the OPD with proptosis and restricted extra ocular movements. A provisional
diagnosis of chloroma is made. Which is next best suited investigation for evaluation?
(A) Platelet count
(B) Peripheral smear
(C) Leukocyte count
(D) Hb estimation
Chloroma

Also known as Myeloblastoma/Granulocytic Sarcoma.


: It is an extramedullary myeloblast proliferation commonly associated with
AML - M2 (MC FAB subtype of AML).
Most Common Site : Eye (can present anywhere in the body)
: Presents with proptosis, restricted eye movements, diplopia.
It is comprised of myeloblasts wich are MPO+ (reason for green colour of tumour).
:
Diagnosis
Peripheral Smear - Shows blast cells, Arbiskov Cells (monocytes seen in myeloblastoma)
Biopsy
: IHC Markers : CD43, CD45, CD117, MPO
Flow Cytometry

:
Treatment
Chemotherapy
Surgical/ Local management only if indication arises.

:
46. A 50yr old factory worker presents to the OPD with cough & dyspnea. On evaluation he was
found to have lung fibrosis and evidence of pleural thickening. Biopsy of lung showed the
following finding. Which of the following is most likely?
(A) Silicosis
(B) Asbestosis
(C) Anthracosis
(D) Byssinosis

The given clinical history is suggestive of Asbestosis.


Asbestosis

Asbestosis an occupational lung disease especially seen in shipping industry.


Occurs because of chronic exposure to Asbestos.
: Two types of Asbestos fibres are implicated
Serpentine
: Amphibole - More pathogenic
Patient complaints of cough, dyspnea.
: It usually affects the lower lobes of lung and presents as
Pleural plaque (Most Common Lesion)
: Pleural Nodules
Lung Cancer
: Malignant Mesothelioma (Most specific finding of asbestosis, needs about 25 years of exposure)
Markers of malignant mesothelioma : Calretinin, CK 5/6

:
H&E demostrates asbestos/ferruginous bodies (dumbbell shaped bodies)

Other Occupational Lung disorders

Silicosis
Miner’s/ Grinder’s Disease
Most common occupational lung disease.
: Increased risk of TB & Lung cancer.
Chest X Ray shows egg shell calcification, fibrosis of lung.

:
Coal worker’s Pneumoconiosis
Affects upper lobes
Presents as coal macules, coal nodules or fibrosis of lung.
: Caplan Syndrome : Rheumatoid Arthritis + Coal Worker’s Pneumoconiosis

47. A 60yr old patient presented to the OPD with history of dyspnea on exertion for the past 1
month. He has history of severe mitral stenosis. His diseased valve was replaced with a prosthetic
valve. The resected valve was biopsied. Identify the characteristic finding in the biopsy and the
disease involved.
(A) Aschoff nodules; Rheumatic Heart disease
(B) Fungal granuloma; Infective Endocarditis
(C) Naked granuloma; Sarcoidosis
(D) Granuloma; Tuberculosis

The given clinical history is suggestive of rheumatic heart disease.


Rheumatic fever and Rheumatic Heart Disease
It is an immunologically mediated multi - system disease.
: Type 2 hypersensitivity reaction which usually occurs 2-3 weeks after a
Beta haemolytic Streptococcal infection (Strains 1,3,5,18).
This is due to molecular mimicry of Streptococcal Antigen with glycoprotein in heart and joints.
: Most Common Valve affected : Mitral Valve
Least Common affected : Pulmonary Valve
: In acute rheumatic fever, patient presents with mitral regurgitation.
[
In chronic rheumatic fever, patient develops mitral stenosis.
[

Diagnosis is by Revised Jones Criteria.

The Modified Jones Criteria (2015)


Low Risk Populations
Major Criteria Minor Criteria
1. Carditis 1. Polyarthalgia
Clinical and/or subclinical 2. Fever (238.5°C)
2. Arthritis 3. ESR ≥ 60 mm/h and/or CRP 23.0 mg/dL.
Polyarthritis only 4. Prolonged PR interval after accounting for age variability.
3. Chorea
4. Subcutaneous nodules
5. Erythema marginatum
Moderate risk to High Risk Populations
Major criteria Minor criteria
1. Carditis 1. Monoarthralgia
Clinical and/or subclinical 2. Fever (≥38°C)
2. Arthritis 3. ESR ≥30 mm/h and/or CRP ≥3.0 mg/dL
Monoarthritis or polyarthritis 4. Prolonged PR interval after accounting for age variability.
Polyarthralgia
3. Chorea
4. Subcutaneous nodules
5. Erythema marginatum

(For all patient populations with evidence of preceding Group A streptococcal infection.)
Diagnosis of initial ARF: 2 major manifestations or 1 major plus 2 minor manifestations.
Morphology of heart in Rheumatic Heart Disease
1
Aschoff Bodies
Pathognomic for RHD.
: Consists of lymphocytes, plasma cells,giant cells, fibrinoid necrosis, collagen and cells with
wavy, slender, ribbon - like nucleus (Caterpillar cells/Antischkow Cells)

Bread and butter pericarditis


: McCallum plaques/ Subendocardial jets
Fish mouth stenosis/Button hole stenosis in chronic cases
: Small, warty, verrucous vegetation along the lines of closure of valve leaflets
(sterile vegetations)

Treatment
Bed Rest
: Penicillin X 10 days
Aspirin/Steroids
: Heart Surgery in case of valvular disease

Secondary Prophylaxis

AHA Recommendations for Duration of Secondary Prophylaxis


Category of Patient Duration of Prophylaxis
RF without carditis For 5 years after the last attack
or 21 years of age (whichever is longer).
RF with carditis but no For 10 years after the last
residual valvular disease attack, or 21 years of age (whichever is longer).
RF with persistent valvular For 10 years after the last
disease, evident clinically attack, or 40 years of age (whichever is longer).
or on echocardiography Sometimes lifelong prophylaxis.
Recommended Antibiotic Regimen for Secondary Prophylaxis
Antibiotic Dose Route Frequency
First Line
Benzathine 1,200,000 units (>=20kg) Deep Inramuscular Every 28 days
Benzylpenicillin G (BPG) 600,000 units (<20kg) Injetion Every 21 days for
Selected groups
Second Line (if IM route is not possible or consistently denied)
Phenoxymethylpenicillin 250 mg Oral Twice a day
(Penicillin V)
Following Documented Penicillin Allergy
Erythromycin 250 mg Oral Twice a day

For children weighing less than 10 kg, a dose of 600,000 units is still generally recommended
: but seek paediatric advice for careful planning of the regimen of secondary prophylaxis,
People on 28-day regirens can be recalled from 21 days to help ensure that injections are
given by day 28.
BPG given every 21 days may be considered for

:
a) patients who have breakthrough ARF despite complete adherence to a 28-day regimen, or
b) are at high risk of adverse consequences if ARF occurs (have severe RHD or a history of
heart valve surgery).

48. Patient presents with low grade fever and cough for the past 2 weeks. Chest X-ray showed
evidence of caseous necrosis in the apex of lung. What is the underlying mechanism of this disease?
(A) Type IV hypersensitivity - epitheloid cells, giant cells, macrophages
(B) Enzymatic degradation
(C) Fibrinoid necrosis
(D) Sudden cut of blood supply
The given clinical history (low grade fever, cough for 2 weeks) is suggestive of TB infection of lung.

TB Lung
Caused by inhalation of Mycobacterium Tuberculosis.
95% cases remain asymptomatic (Latent TB).
: 5% becomes symptomatic.
The inhaled organism activate cell mediated immunity resulting in the formation of an epitheloid
: granuloma with central caseous necrosis.
Pulmonary alveolar macrophages present the organism to a T cell resulting in a TH1 response
and release of Interferon γ and phagolysosome maturation.
I
Granuloma is composed of slipper shaped cells called epitheliod cells, multinucleated giant cells
called Langhan’s Giant cell with a central area of necrosis.
Initial infection results in the formation of a Ghon’s complex.
Ghon’s focus + Draining LNS Ghon’s Complex Ranke Complex
Calcify
(Ghon’ Focus : Caseous granuloma appearing as small foci of consolidation in lung)
Progression of TB
When TB affects any site other than lung, it is termed as extrapulmonary TB.
: Most Common Site : Lymph Nodes (especially cervical)

Diagnosis
Sputum Microscopy
: CBNAAT

Treatment : Anti -Tubercular Treatment


49. FISH is done for which of the following
(A) Her2/neu 1+
(B) Her2/neu 2+
(C) Her2/neu 3+
(D) all of the above
Her2/neu
Her2/neu is a membrane receptor that belongs to the human epidermal growth factor

!receptor family.
Amplification/ over expression is associated with increased risk of breast cancer.
Other receptor associated with breast cancers are ER and PR which are nuclear receptors.

Molecular Classification of Breast Cancer


Allred Scoring System
I
Allred scoring system is used to grade ER & PR positive cancers.
1
ER, PR receptors if positive stain the nucleus.
Her 2/neu receptors if positive, stain the plasma membrane.
: Allred Score = Proportion Score + Intensity Score

Proportion Score Intensity Score


0 : No cells positive 0 - Negative
1 : <1% cells positive 1 - Mild
2 : 1 - 10% cells positive 2 - Moderate/Intermediate
3 : 11 - 33% cells positive 3 - Strong
4 : 34 - 67% cells positive
5 : 67 - 100% cells positive

Her 2/neu is graded as :


0
Her 2/neu negative
1
2 Equivocal result - FISH test is required to finalise
3 Her 2/neu positive
ER, PR, Her2/neu receptor statuses are the most important prognostic factors of breast cancer in
the presence of metastasis.
50. The following karyogram shows a defect in a particular chromosome. Which mutation is the
arrow mark pointing to?
(A) Acrocentric
(B) Metacentric
(C) Submetacentric
(D) Telocentric
Classification of chromosomes
2
Depending on position of centromere, chromosomes are classified as
Metacentric : Both p and q are of equal size (p = q).
: Eg : Chromosome 1
Submetacentric : The q arm is slightly longer than p arm (p < q).
Eg : Chromosome X
Acrocentric : The p arm is very small compared to q arm (p <<< q).

: Eg : Chromosomes 13, 14, 15, 21, 22, 23, Y


Telocentric : The p arm is completely absent. This type does not exist in humans.

Karyotyping
The study of chromosomes is known as karyotyping.
: Karyogram : Arrangement of chromosomes in descending order of length, with sex
chromosomes in the end.

The chromosomes are arrested in the metaphase of cell cycle using colchicine and stained.
: They are viewed under a light microscope of 5 mb resolution.
In the given question, the arrow points to chromosome number 13 in the karyogram.
Chromosome 13 is normally acrocentric, but in the given image the centromere is pointed at
the centre showing a metacentric translocation defect.
51. A 15yr boy presents to the OPD with complaints of swellings in neck. On examination cervical
lymphadenopathy was identified. A biopsy of the lymph nodes was done and HPE showed the following
picture. Identify the cancer and the characteristic finding.
(A) Hodgkin’s with R-S cells; HIV infection
(B) Hodgkin’s with R-S cells; EBV infection
(C) Non-Hodgkin’s with giant cells
(D) Non-Hodgkin’s with R-S cells

Hodgkin’s Lymphoma

1
Hodgkin’s Lymphoma is a type of lymphoma in which cancer originates from lymphocytes
associated with multinucleated Reed - Stenberg cells (RS Cells) which are present in the
lymph nodes.
It has a bimodal age distribution (seen in 15 - 20 years and in elderly).
: Patients usually present with painless cervical node enlargement, fever (Pel Ebstein fever),
night sweats and weight loss.
The lymph node involvement is contiguous in nature.
: It can be associated with EBV infection (activation of NF - kB pathway by latent membrane
protein -1 of EBV).
Most common Paraneoplastic Syndrome : Amyloidosis (AA Type)
: Most Common HL in the world : Nodular Sclerosis HL (usually not associated with EBV).
Most comon HL in India : Mixed Cellularity HL (commonly associated with EBV infection)
:
HL with worst prognosis : Lymphocyte depleted HL
HL with best prognosis : NLPHL

:
WHO classification of Hodgkin’s Lymphoma
Classical Non - Classical
C

Nodular Sclerois
e

Nodular lymphocyte predominant


e

Mixed Cellularity
e
Hodgkin’s Lymphoma
e

Lymphocyte Rich
e
RS cells are CD15-, CD30-, CD20+,
e

Lymphocyte depleted Bcl 6+, CD45+, EMA +


: RS cells are CD15+,
CD30+, PAX5+
Reed Stenberg Cells (RS Cells)
s
Large binuclease cells
Prominent nucleoli present (Owl’s Eye Appearance)
: RS cells are also seen in :
Infectious Mononucleosis
: Non - Hodgkin’s Lymphoma
Solid Tisuue Cancers
: But they lack an inflammatory background
which distinguises them from Hodgkin’s Lymphoma

Variants of RS cells
Lacunar variant : Seen in nodular sclerosis Hodgkin’s Lymphoma
: Lymphohistiocytic/Popcorn variant : Seen in Nodula Lymphocyte predominant HL
I
Mononuclear Variant : Single Nuclei present

Treatment of HL : Adriamycin, Bleomycin, Vinblastine, Dacarbazine regimen


Ann - Arbor Staging of HL

(A) or (B) designates absence/presence of B symptoms respectively.


B symptoms : Fever, Night Sweats, Weight Loss
52. A boy presents to the OPD with history of fever, night sweats and neck swellings. Biopsy of the
swellings showed a starry sky appearance. Identify the possible gene mutation responsible for the
given condition.
(A) RB
(B) TP53
(C) c-myc
(D) BCR-ABL
The given clinical history is suggestive of Burkitt’s Lymphoma.
Burkitt’s Lymphoma
n
Most common type of NHL in children < 15 years.
n
It is a cancer of B lymphocytes present in germinal centre.
n
It can arise from genetic defects : t (8:14), t (2:8), t (8:22)
n
t (8:14) - Resulting in increased expression of c - myc (oncogene) causing increased
cell proliferation.
n
It is also commonly associated with EBV infection (activation of NK - kB pathway).

Variants of Burkitt’s Lymphoma


Endemic (African Variant) : Mostly in children, Associated with EBV
: Most Common Site : Jaw, Mandible
Sporadic : Mostly in adults, can be associated with EBV
Most Common SIte : GIT (ileo - caecum, peritoneum)
9
HIV associated : Seen in HIV patients, can occur anywhere in body

Diagnosis
Lymph Node Biopsy : Starry Sky appearance

: Stars : Macrophages

Sky : Lymphoid Cells

Tumour Markers : CD19+, CD20+, Bcl - 6+


Bone Marrow Aspirase : Large cells with royal blue cytoplasm containing cytoplasmic vacuoles.

53. A patient develops anaphylaxis following a bee sting. Identify the antibody mediating this
response.
(A) IgM
(B) IgA
(C) IgD
(D) IgE
The given history is suggestive of a Type I hypersensitivity reaction to a bee sting.
It is mediated by Ig E antibody.

Type I Hypersensitivity Reaction (Anaphylaxis)

Examples
-

Food Allergy
-

Pollen Allergy
-

Allergic Rhinitis
-

Allergic Dermatitis
-

Hay fever
-

Anaphylaxis
-

Atopy, etc.

Pathogenesis
r

On first exposure mast cells get sensitised to the antigen.


r

On subsequent exposure, the mast cells undergo immediate degranulation.


r

They release preformed inflammatory mediators initially (histamine) which cause vasodilation,
bronchospasm and increased vascular permeability - Early Phase Reaction.
r

They release formed mediators later (Leukotrienes, Cytokines, Chemokines) within 2 - 24 hours
which cause further tissue destruction - Late Phase Reaction.
r

IgE is the antibody that mediates Type I hypersensitivity reaction.


r

Cytokines involved include IL 4, IL 5.


54. Which of the following is the most radio resistant phase of the cell cycle?
(A) G1
(B) S
(C) G2
(D) M
Cell Cycle
Cell cycle is composed of 4 major phases :
G1 : Gap Phase I (Resting Phase)
: S : Synthesis Phase (DNA replication occurs here)
G2 : Gap Phase (Resting Phase)
: M : Mitosis (Cell division takes place)
Most radioresistant phase of cell cycle : S Phase
: Most radiosensitive phase of cell cycle : G2/M transition phase.
Regulation of cell cycle
Cell cycle is regulated by cyclins and cyclin dependent kinases (CDK).
: G1 - S transition is regulated by [Cyclin D + CDK4] and [Cyclin E + CDK2].
G2 - M transition is regulated by [Cyclin A + CDK2] and [Cyclin B + CDK1].
: Rb gene (Retinoblastoma gene) on Chromosome 13q 14 acts as the governor of cell cycle
and regulated the G1 - S transition.
p53 gene is responsible for arrest of cell cycle in G1 phase to provide time for DNA
: repair.
If not reparable it triggers apoptosis.

55. Identify the incorrect statement regarding frozen section.


(A) used for diagnosis of metastasis
(B) can be used during sentinel lymph node biopsy
(C) provides definitive diagnosis in all cancers
(D) helps to check for negative tumor margins
Frozen Section (Cryosection)
It is a pathological laboratory procedure done to perform rapid microscopic analysis of a specien.
: Mainly used in oncological surgery.
It can be used for diagnosis in many cases, but quality of slides produced is lower compared to

:
Uses
formalin fixed paraffin embedded tissue processing (gives more accurate diagnosis).

Intraoperative assessment of resectal specimens to check whether resection margins are


clear of the tumour.
Sentinel lymph node biopsy to check for presence of tumour in dissected lymph nodes.
: To check for suspected metastasis.
56. A 62yr old patient is admitted with heart failure and is scheduled for heart transplant. His RFT
is deranged and Hb-6gm%. The physician ordered 2 units of whole blood and was transfused. 4 hours
later he is having severe respiratory distress and fever. He is hypoxemic and MAP is elevated.
Which are the best investigations in this case?
(i) Chest x-ray
(ii) BNP levels
(iii) Neutrophil count
(iv) CD34

(A) i, ii
(B) ii, iii
(C) ii, iv
(D) I, iv
The given clinical history is suggestive of Transfusion Associated Circulatory Overload (TACO).

Transfusion Associated Circulatory Overload (TACO)


It is a transfusion reaction resulting in signs/symptoms of excess fluid in the circulatory
: system (hypervolemia) within 12 hours after transfusion.
Patient presents with dyspnea, hypoxemia, generalised edema, hypertension and tachycardia.
Risk factors for TACO : Renal failure, Heart failure, Liver failure, Old age

:
TACO Classification
Non - Severe : No permanent damage would arise if treatment was not given.
Severe : Treatment is needed to avoid permanent damage.
: Life Threatening : ICU care needed.
Death.

:
Diagnosis
Patient diagnosed with TACO should have atleast one of the following characteristics within
12 hours after the transfusion.
Acute/worsening respiratory distress (tachypnea, dyspnea, cyanosis, hypoxemia) in the
:
absence of other causes.
Evidence of acute or worsening pulmonary oedema (senn in chest imaging).
Along with :
Elevations in Brain - Natriuretic Peptide (BNP)
: Evidence of CVS system changes (tachycardia, hypertension, JVP elevation,
Peripheral oedema)
: Evidence of fluid overload.

TRALI - Transfusion-related acute lung injury


Differences between TRALI and TACO
TRALI TACO
Blood pressure Low-normal Normal-high
Body temperature Normal-elevated Normal
CXR No vascular congestion Vascular congestion, Pleural effusion
BNP Low (< 250 pg/ml) High
PAOP Low-normal High
Ejection fraction Normal function Abnormal function
Response to diuretics Inconsistent Improved
Edema fluid Transudate Exudate

57. Which of the following following is true for TACO?


(i) Chest x-ray showing congestion
(ii) Fever
(iii) Dyspnea
(iv) Hypotension
(v) BNP <200pg/ml

(A) i & iii


(B) i, iii, v
(C) ii, iv, v
(D) ii, iii, iv

Transfusion Associated Circulatory Overload (TACO)


s

It is a transfusion reaction resulting in signs/symptoms of excess fluid in the circulatory


system (hypervolemia) within 12 hours after transfusion.
Patient presents with dyspnea, hypoxemia, generalised edema, hypertension and tachycardia.
: Risk factors for TACO : Renal failure, Heart failure, Liver failure, Old age

TACO Classification
Non - Severe : No permanent damage would arise if treatment was not given.
: Severe : Treatment is needed to avoid permanent damage.
Life Threatening : ICU care needed.
: Death.
Diagnosis
M

Patient diagnosed with TACO should have atleast one of the following characteristics within
12 hours after the transfusion.
Acute/worsening respiratory distress (tachypnea, dyspnea, cyanosis, hypoxemia) in the
:absence of other causes.
Evidence of acute or worsening pulmonary oedema (senn in chest imaging).
Along with :
Elevations in Brain - Natriuretic Peptide (BNP)
: Evidence of CVS system changes (tachycardia, hypertension, JVP elevation,
Peripheral oedema)
: Evidence of fluid overload.

TRALI - Transfusion-related acute lung injury

Differences between TRALI and TACO


TRALI TACO
Blood pressure Low-normal Normal-high
Body temperature Normal-elevated Normal
CXR No vascular congestion Vascular congestion, Pleural effusion
BNP Low (< 250 pg/ml) High
PAOP Low-normal High
Ejection fraction Normal function Abnormal function
Response to diuretics Inconsistent Improved
Edema fluid Transudate Exudate

58. Which of the cells is affected in neonate suffering from respiratory distress syndrome?
(A) A C
A B
(B) B
(C) C
(D) D

D
Respiratory Distress Syndrome
It is seen commonly in pre - term neonates and babies born to diabetic mothers.
: There is decreased production/lack of surfactant production by Type 2 pneumocytes present
in alveoli of lung.
As a result there is collapse of alveoli resulting in respiratory distress (tachypnea, chest

: refractions, grunting and cyanosis).


Chest X - Ray findings
White out/Ground glass appearance of lung.
Air bronchogram.

:
Management
Prevention
Administration of antenatal steroid in pregnant women who are at risk of early
:
delivery (<34 weeks of gestation).
Dexamethasone : 6mg X 4 doses (12 hours apart)
Betamethasone : 12 mg X 2 doses (24 hours apart)
iSurfactant Therapy
Administered intratracheally.
1

Synthetic surfactants used (Lucinactant).


: It is either given prophylactically (in preterm < 28 weeks) or as rescue therapy after
development of RDS.
59. Oil red O stain is used for which specimen
(A) Frozen section
(B) Formaldehyde fixed
(C) Alcohol fixed
(D) Glutaraldehyde fixed
Oil red o stain
Oil red O stain is used in the staining of fats (triglycerides & lipids).
: Among all the options given only frozen section preserves the fat content.
Alcohol and aldehyde fixatives dissolve the fat present in the specimen and hence can’t be

:
stained by Oil red O stain.

Important Stains
Lipids/Fat Oil Red O, Sudan Black
Iron/Haemosiderin Perl’s Prussian Blue
Calcium Von Kossa, Alzarine Red S
Melanin Masson Fontana
Elastin Van Geison
H.Pylori Warthin’s Starry Silver
Cryptococcus India Ink
Mucin Alcian Blue, Mucicarmine
Mast Cell Toludine Blue
Collagen Masson’s Trichrome
Reticulin Silver Stain
Amyloid Congo Red
Copper Rhodamine, Rubeanic Acid, Orcein
Glycogen PAS

60. A 75yr old man presents to the OPD with a left sided testicular mass and ataxia. Which of the
following is most likely?
(A) Hodgkins lymphoma
(B) Non seminomatous GCT
(C) Non Hodgkins lymphoma
(D) Glioblastoma
According to the given history the patient is most likely to have Non - Hodgkin’s Lymphoma.
Glioblastoma is seen in the brain and it rarely metastasises to the testes.
: Non - seminomatous Germ Cell Tumour can present with testicular swelling but
ataxia is less likely.
4

Hodgkin’s lymphoma and Non - Hodgkin’s Lymphoma both can have paraneoplastic
syndrome symptoms.
Cerebellar ataxia and nephrotic syndrome are common in patients of Hodgkin’s and
: Non - Hodgkin’s Lymphoma.
Hodgkin’s Lymphoma usually presents with cervical lymphadenopathy, it rarely involves
: extra nodal tissues.
Non - Hodgkin’s Lymphoma can involve extra nodal tissues (testes, GIT, brain, etc) and
hence it is the most likely answer for this question.
Microbiology
61. Which of the following is not a test for pasteurisation of milk
(A) Phosphatase test
(B) Iodine test
(C) Methylene blue
(D) Standard plate
Pasteurisation
Pasteurisation is a technique used for the disinfection of milk.
Methods of pasteurisation
Holder Method : Heat the milk at 66° for 30 mins and cool it.
: Flash Method
HTST Method : Heat the milk at 72°C for 15 - 20 seconds and rapidly cool to <13°C
:
(High Temperature and Short Time Method)
UHT Method : Heat the milk to 125°C and cool in few seconds.
(Ultra High Temperature Method)

Efficacy of Pasteurisation
Efficacy of Pasteurisation can be checked by :
Phosphatase test : Enzyme destroyed by heat, so it is absent in pasteurised milk.
: Standard plate/Colony count : It wil be <30000 colonies/ml in pasteurised milk.
Coliform Test : E.coli in milk indicates foecal contamination, it is zero in pasteurised milk.

:
62. Match the following for Japanese encephalitis
(a) Culex mosquito (i) Accidental host
(b) Man (ii) Reservoir host
(c) Pig (iii) Amplifier host
(d) Ardeid birds (iv) Vector
(v) Paratenic host
(A) a-iv, b-i, c-ii, d-iii
(B) a-iv, b-i, c-iii, d-ii
(C) a-iv, b-ii, c-iii, d-i
(D) a-ii, b-iv, c-iii, d-i

Japanese Encephalitis

Caused by Group B Arbovirus belonging to Flaviviridae.


It is a zoonotic disease and man is an accidental host.
:
Vector : Culex mosquito (Culex vishnui, Culex tritaeniorhynchus).
Ardeid birds are the reservoir/maintenance host for this virus.
: Pigs act as the amplifer host in which the virus multiplies.
Man is an accidental host and has a dead end infection (can’t transmit to other humans).

:
Clinical features

L
Prodromal Phase

:
Lasts for 1 - 7 days.
Fever with coryza present.
Acute Encephalitic stage
Lasts for 3 - 6 days.
: Convulsions, focal neurological deficit, raised intracranial tension.
L

Post encephalitic stage


Sequelae of infection.
:
Residual neurological deficits present.

Diagnosis is by IgM ELISA.


: Treatment : IV fluids, antipyretics, mannitol/glycerol for raised ICT.
Prevention
JE Vaccination
:
Strains used : SA - 14 - 14 - 2 (live)
Beijing and Nakayama strain (killed)
Schedule : First dose at 9 months.
: Second dose at 16 - 24 months
Given as 0.5ml s/c in left arm.
63. Match the following

(a) Hymenolepsis nana (i)

(b) Ascaris (ii)

(c) Hook worm (iii)

(d) Taenia (iv)

(A) a-iii, b-i, c-iv, d-ii


(B) a-ii, b-iii, c-iv, d-i
(C) a-i, b-ii, c-iv, d-iii
(D) a-iv, b-ii, c-iii, d-i

Eggs of various Helminths

Schistosoma hematobium
(Elongated with terminal spine) Schistosoma mansoni Schistosoma japonicum
(Elongated with lateral spine) (Round with terminal knob)
Taenia solium & Hymenolepis nana Diphyllobothrium latum
Taenia saginatum

Ascaris lumbricoides Ancylostoma duodenale & Strongyloides stercoralis


Necator americanus

Enterobius vermicularis Trichuris trichuria

Non - bile stained eggs : N.americanus, A.duodenale, E.vermicularis, H.nana


Operculated eggs : D.latum, Spirometra, all trematodes except Schistosoma

64. Which of the following is an obligate anaerobe?


(A) Yersinia
(B) Prevotella
(C) Burkholderia
(D) Stenotrophomonas

Aerobic and anaerobic bacteria can be identified by growing them in test tubes of thioglycollate broth.
Obligate Aerobes (1)
I
Need oxygen, can’t ferment or respire anaerobically,
They gather at the top of the tube where O2 concentration is highest.
:
Eg :
Mycobacterium tuberculosis
Pseudomonas aeruginosa
Bacillus, Nocardia, Micrococcus, Bordetella, Brucella
Fransicella, Legionella

Obligate Anaerobes (2)


Poisoned by oxygen, gather at the bottom of the tube where O2 is lowest.
:
Eg :
Actinomyces, Bacteroides, Clostridium
Fusobacterium, Peptostreptococcus, Porphyromonas
Prevotella, Propionibacterium, Veillonella

Facultative Anaerobes (3) -

Can grow with or without oxygen.


: Can metabolise aerobically or anerobicaly.
Gather mostly at top as aerobic respiration generates more ATP.
: Eg : Staphylococcus, E.coli, Salmonella, Listeria, Yersinia
Microaerophiles (4)
Need oxygen for respiration but are poisoned by high concentrations of Oxygen.
: They gather in the upper part of the tube but not at the very top.
Eg : Campylobacter, Helicobacter pylori.

:
Aerotolerant organisms (5)
Do not require oxygen as they metabolise energy anaerobically.
-
They are not poisoned by Oxygen found evenly spread throughout the test tube.
65. A 7yr old non-immunised child presented to the OPD with pseudomembrane over the tonsils. A throat
was taken, stained and examined under the microscope. Choose the correct staining steps.
(i) Albert stain 1
(ii) Albert stain 2
(iii) Iodine
(iv) Acetone
(v) Methylene blue
(vi) Safranin
(vii) Water

(A) i & ii
(B) iii, iv, v, vi
(C) i, ii, iii, iv, v
(D) iii, iv, vii

The given image shows a microscopy of a slide stained with Albert’s stain.

Albert’s Staining
Type of differential stain.
:
Stains metachromatic granules/volutin granules of Corynebacterium diphtheriae (which
contains high molecular weight polymers of polyphosphate)
Principle
Albert stain is made up of two stains (Toluidine blue ‘O’ and Malachite green) which
: are basic dyes.
On applying stain, Toluidine blue ‘O’ stains volutin granules (the most acidic part of
cell) and malachite green stains the cytoplasm blue - green.

Composition Procedure
Albert stain is composed of two reagents. 1) Prepare smear on clean side.
: Albert Stain 1
2) Air dry and heat fix the smear.
Toluidine blue 3) Apply Albert stain 1 and allow to react for
: Malachite green
about 7 mins.
Glacial acetic acid 4) Drain the excess stain.
: Alcohol (95% Ethanol)
5) Flood the smear with Albert stain 2 for 2
I

Albert Stain 2 minutes.


Iodine 6) Wash the slide with water, air dry and
: Potassium Iodide
observe under microscope.
Procedure of Gram staining

66. Identify lipid A in given gram negative bacterial cell wall.


(A) A
(B) B
(C) C
(D) D tttthhrnnnnan B••-B-

D-

Cell wall of gram negative bacteria


Gram negative bacteria has certain special structures in their cell wall when compared to gram
positive bacteria. They are :
Outer membrane
: Periplasmic space
i

Endotoxin/LPS
The gram negative bacteria has an outer membrane just beneath the cell wall.
: Porins and integral proteins are present in outer membrane.
A periplasmic space seperates the outer membrane from the cytoplasmic membrane.
: The periplasmic space contains 2 layers of murein monomers cross - linked by peptides.
Endotoxin
It is a lipopolysaccharide.
: It is embedded in the cell wall of bacteria.
Released only on lysis.
: Heat stable but poorly antigenic.
It has 3 parts
: Lipid A : Embedded in outer membrane
Has the actual endotoxic activity.
Core polysaccharide : Short chain of 6 - 10 carbohydrates.
: O/Somatic Antigen : Outermost part, polysaccharide, most variable part of endotoxin.
Mechanism
Activates alternate complement pathway.

:|
Activates the tissue factor.
Induces cytokine release.

Gram positive cell wall Gram Negative Cell Wall


~ 80nm thick lipid : 1 - 5% 10 - 25 nm thick
Peptidoglycan : 50% Lipid : 30 - 50%
Peptidoglycan : 5 - 10%

50 - 100 layers of murein 2 layers of murein monomers cross linked by


monomers cros linked by peptides.
peptides.

Aromatic or Sulphur containing All aminoacids present.


amino acids are absent.

Teichoic acids present. Teichoic acids absent.

Special structures absent. Special structures present.


- Outer membrane
- Periplasmic space
- Endotoxin/LPS

67. Which of the following cause attachment effacement lesion as shown in the image?
(A) ETEC
(B) EPEC
(C) EIEC
(D) EHEC

E.Coli
E.coli is a gram negative bacteria which is the most common facultative anaerobe present in

i.
human GIT.
It is a normal gut flora.
E.coli can cause diarrhea in humans.
Diarrheogenic E.coli are never normal flora.
Diarrheogenic E.coli
Entero - toxigenic E.coli (ETEC)

÷
Entero - pathogenic E.coli (EPEC)
Entero- Aggregative E.coli (EAEC)
Entero invasive E.coli (EIEC)
Entero - haemorrhagic E.coli (EHEC)

Entero - toxigenic E.coli (ETEC)


Most common cause of traveller’s diarrhea in all age groups.
Transmitted by food and water.
: Infective dose > 109
Produce toxins causing secretory diarrhea.
: Heat labile toxins : Increases cAMP, similar to cholera toxin.
Heat stable toxins : Increases cGMP.

:
Entero - pathogenic E.coli (EPEC)
Most common cause of infantile bacterial diarrhea.
Viruence is conferred by plasmids.
: Adherence factor plasmid : Bundle forming pilus for adherence.
Enterocyte effacement plasmid : Attachment to cup like pedestals with effacement of villi.

:
Cause attachment - effacement effect leading to
secretory diarrhea.

Entero - haemorrhagic E.coli (EHEC)


Verocytogenic E.coli
Most common cause of haemolytic uremic syndrome in the world.
: Infective dose : 10 - 100.
Transmitted from animals (Zoonotic disease)
: Reservoir : GIT of cows.
Virulence is caused by a phage mediated shiga like toxin (verocytotoxin).
:
Causes bloody diarrhea, abdominal cramps and vomitting.
Strain causing mosy severe disease with HUS : E.coli 0157 : H7.
: Diagnosis :
Sorbitol MAC test - To identify 0157 : H7 EHEC
Cytotoxicity on vero cell lines - non 0157 : H7 EHEC
Management is done only for symptoms.
: Antibiotics are contraindicated.
Entero- Aggregative E.coli (EAEC)
Most common cause of persistent diarrhea in the world.
: Causes secretory diarrhea.
Hep2 adherence assay shows ‘Stacked Brick Adherence’ pattern.

:
Entero invasive E.coli (EIEC)
Atypical E.coli (non - motile, anaerogenic)
Causes disease similar to shigella.

:
68. Paul-Bunnell test is based on
(A) Complement fixation
(B) Superantigens
(C) Hapten
(D) Heterophile antibody
Paul Bunnel Test
Paul - Bunnell test was used for the diagnosis of infectious mononucleosis by
Ebstein - Barr Virus (EBV).
In patients with IMN, there is production of heterophile antibodies.
:
Heterophile Antibodies
These are antibodies directed against external antigens which cross react with self
antigens.
Its presence is characterised by broad reactivity with antibodies of other species.
: They can interfere in immunoassay.
In Paul - Bunnell test, heterophile antibodies in the patient’s blood sample is made to react wuth
: sheep, horse or cow red blood cell antigens.
This works on the principle of latex agglutination reaction.
69. Identify virus having the lifecycle given below:
(A) HIV
(B) Hepatitis B
(C) Herpes
(D) Hepatitis C

The given lifecycle is that of Herpes Simplex Virus (HSV).

Steps of HSV Replication Cycle


Binding : Viral glycoprotein B (gB) binds to the cell surface, followed by glycoprotein D (gD)
: binding to one of its receptors.
Entry : After fusion of membranes, the viral capsid and tegument proteins are internalised
in the cytoplasm.
Capsid transport to the nucleus : Once in the cytoplasm, the viral capsid accumulates in the
: nucleus either by simple diffusion or aided by cytoskeletal structures such as microtubules.
Transcription : HSV genes are sequentially transcribed as Alpha, Beta and Gamma genes.
Translation : Viral mRNAs are translated sequentially.
: Replication : Viral genome replication occurs as a rolling cycle.
Capsid assembly : HSV capsids are assembled within the nucleus of infected cell.
: Glycosylation : Glycoproteins are translated and glycosylated in the ER.
Glycoprotein export to the cell surface : Glycoproteins are processed in the trans - Golgi
: network and microvesicular bodies.
Glycoprotein - containing plasma mebrane endocytosis.
Envelopment : Glycoproteins within early endosomes fare with capsids in the cytoplasm.
: Viral release.
Structre of HSV
Enveloped.
: Icosahedral shaped.
ds DNA virus.
: Presence of tegument between the capsid and the envelope.

HIV Replication Cycle


Hepatitis B Replication cycle

Hepatitis C Replication cycle


70. Which of the following is a dematiaceous fungi?
(A) Cladosporium
(B) Aspergillus niger
(C) Mucor
(D) Blastomyces dermatitidis
Classification of Fungi - Morphology
Yeasts
Unicellular state.
:Divide by budding.
Eg : Cryptococcus, Saccharomyces
Yeast - Like
Exists as yeasts but fail to seperate during budding and
:the buds elongate to form pseudohyphae.
Eg : Candida, Malassezia, Trichosporon.

Molds
Exists as hyphae
: Aseptate/Coenocytic molds
Have broad, ribbon - like hyphae which are aseptate.
: Belong to phylum glomerulomycetes.
Eg : Rhizopus, Mucor, Absidia
: Septate Molds
Presence of septations at regular intervals
:
2 types
Phaeoid/Dematiaceous Septate Molds
: Contain melanin in their hyphal walls.
Eg : Madurella, Piedra, Cladosporium, Alternaria,
: Curvularia, Exiphiala.
Hyaline Septate Molds
Contains hyaline in their walls.
: Eg : Aspergillus, Fusarium, Dermatophytes, Penicillium, Acremonium.

Dimorphic Fungi
Exists as yeast at 37°C .
: Hyaline septate molds at 25°C - 28°C.
I
Eg : Histoplasma capsulatum, B;astomyces dermatiditis
Coccidiodes immitis, Paracoccidiodes brasiliensis,
Sporothrix Schenkii, Penicillium marnetti (Taleromyces).
71. All are examples of xenodiagnosis, except?
(A) Blood from a dengue patient is injected into stomach of Aedes mosquito and the mosquito is used for vira
(B) Rabbit ileal loop toxin study
(C) Sandfly is allowed to bite a hamster infected with leishmaniasis
(D) None of the above
Xenodiagnosis
Xenodiagnosis is a diagnostic method used to document the presence of infectious disease

:
microorganisms or pathogens by exposing possibly infected tissue to a vector and then
examining the vector for the presence of the microorganisms or pathogens it may have
ingested.
It is mainly used in the diagnosis of Chaga’s disease, Arboviral infections, Trypanosomiasis.
Pharmacology
72. Elimination rate constant of a drug is 0.05/hr. What is it’s half life?
(A) 6.5hr
(B) 20hr
(C) 13.9hr
(D) 8hr
Half Life
-

Half life of a drug is the time taken to reach 50% concentration of a drug in the system.
Half life of a drug (t1/2) = Apparent volume of distribution (AVD)

:
X 0.693
Clearance
Elimination Rate Constant = Clearance

Apparent Volume of Distribution

t1/2 = 0.693 = 0.693 = 13.9

:
Elimination Rate Constant 0.05

73. Anionic and slightly acidic drugs usually bind to:


(A) Albumin
(B) Alpha acid glycoprotein
(C) Ceruloplasmin
(D) Globulin

Drugs can either be acidic or basic in nature.


Depending on this, they bind to different plasma proteins in the blood.
: Acidic drugs tend to bind to basic plasma proteins like Albumin.
Eg : NSAIDs, Warfarin, Aspirin, Sulfonamides, Penicillin,
: Benzodiazepenes, Anti - epileptics, Anti - psychotics.
Basic drugs bind to acidic plasma proteins like α1 - acid glycoprotein.
Eg : Opiods, Tricyclic antidepressants, amiodarone, lidocaine, Beta blockers.

:
Applied Aspects
In conditions of hypoalbunimea, there is increased free drug (acidic) in plasma leading to
toxicity.
Eg : Nephrotic Syndrome, Cirrhosis of liver, CKD.
: Conditions like rheumatoid arthritis, myocardial infarction, IBD have raised levels of α1 -
acid glycoproteins resulting in decreased levels of free drug (basic) leading to drug failure.
74. Category A, B, C, D, X division of drugs is based on :
(A) Safety in pregnancy
(B) Dose adjustment in renal failure
(C) Therapeutic index and safety
(D) Over the counter use of drug
Category A
9
No demonstrable risk to the foetus.
Eg : Levothyroxine, Folic Acid.

:
Category B
Animal reproduction studies have failed to demonstrate a risk to the foetus and there are no
adequate and well controlled studies in pregnant women.
Eg : Metformin, Amoxicillin, Hydrochlorthiazide

:
Category C
Animal reproduction studies have shown an adverse effect on fetus.
No adequate and well controlled studies in humans, but potential benefits may warrant use
:of the drug in pregnant women despite potential risks.
Eg : Gabapentin, Amlodipine, Ttrazodone

Category D
Positive evidence of human fetal risk, but potential benefits may warrant use of drugs in
:pregnant women despite potential risk.
Eg : Losartan

Category X
Animal studies have demonstrated foetal abnormalities.
: Positive evidence of human fetal risk.
Risks clearly outweigh potential benefits.
: Eg : Atorvastatin, Methotrexate, Finasteride

75. The following table gives the data of AUC of drug A alone and AUC of drug A when combined with
drug B. p value is <0.01. Which of the following statements regarding these drugs is most correct?

DRUG Area Under The Curve (AUC)

A alone 550 ± 150

A plus B 850 ± 150


(A) Drug B decreases the first pass metabolism of drug A
(B) Drug B increases the systemic metabolism of drug A
(C) Drug B decreases the intestinal absorption of drug A
(D) Drug B increases the renal clearance of drug A
AUC (Area under the curve) demonstrates the extent of absorption of a drug.

AUC (i.v)
AUC (oral)

Plasma

\
Concentration
Of Drug

Time
Bioavailability = AUC (oral)

:
X 100
AUV (i.v)
The given data has a p value < 0.01 which is significant.
According to the given data, AUV of drug A is increased when combined with drug B.
: This implies that drug A is absorbed better when combined with drug B.
Among the given options, option (B) gives the best explanation for this.

:
76. Which dose of dopamine act preferably on beta-1 receptors?
(A) Less than 2mcg
(B) 2-10mcg
(C) 10-20mcg
(D) More than 20mcg
Dopamine

Dopamine is a catecholamine and is an important neurotransmitter in the brain.


Outside the CNS, it functions primarily as a local paracrine messenger.
: Dopamine has a dose dependent action on autonomic nervous system.
Dose dependent action
At 0 - 2 mcg/kg/min dosage, dopamine is more active on D1 receptors.
: Action on these receptors cause diuresis.
Hence dopamine is the drug of choice in congestive heart failure with oliguria.
: At 2 - 10 mcg/kg/min, dopamine acts on Beta 1 receptor thereby increasing
cardiac contraction.
At > 10mcg/kg/min, dopamine acts on alpha 1 receptor causing vasoconstriction.
: Hence, it is preferred in management of cardiogenic shock.

77. Resistant hypertension is defined as when BP is not controlled inspite of using 3 classes of
anti-hypertensive drugs. Among these which of the following class of drugs must be the patient on
to meet the definition?
(A) Diuretics
(B) Alpha blockers
(C) Beta blockers
(D) Calcium channel blockers
Resistant Hypertension
Resistant hypertension is defined as blood pressure that remains high despite the use of 3
:different classes of anti - hypertensives, one of which must be a diuretic.
The underlying cause of resistant hypertension is increased production of aldosterone.
The drug of choice for resistant hypertension is spironolactone.

:
Hypertensive Emergency
Hypertension with end organ damage (encephelopathy, retinopathy, CHF, Ecclampsia,
Pulmonary Oedema)
a
Treatment : iv antihypertensives like Hydralazine, Enalaprilat, Labetalol, Esmolol,
Nitroglycerin, Nitroprusside, Nicardipine.
Hypertensive Urgency
BP >= 220/125 mm Hg
: No end organ damage present.
Treatment : Oral Clonidine, Nifedipine, Captopril.

:
78. A man comes to OPD with history of fall. He is hypertensive with history of atrial fibrillation
and is presently on captopril, atenolol, aspirin and amiodarone. He presents with following finding
as shown in the image. What could have caused this?
(A) Lupus pernio
(B) Captopril toxicity
(C) Amiodarone induced skin lesion
(D) Silver toxicity

The given image shows a bluish discolouration of skin of face.


It is a common side effect associated with long term use of Amiodarone.
: Amiodarone is a class III anti - arrythmic drug used in the treatment of atrial fibrillation.
Amiodarone
Class III anti - arrythmic drug.
: Blocks k+, Na+, Ca+, Alpha and Beta receptors (widest spectrum anti - arrhythmic drug)
It is used in the management of ventricular fibrillation, ventricular tachycardia, atrial
: fibrillation, atrial flutter.
It is also the longest acting anti - arrhythmic.
Side Effects
Due to presence of Iodine.
:
Pulmonary fibrosis
Due to Type II pneumocyte damage; Rx : Prednisolone
e

Corneal Microdeposits
e

Whorl like pattern


e

Ceruloderma
Blue/Grey skin
e

Myocarditis
Photosensitivity
: Liver toxicity, Neuropathy
Hypotension (due to alpha receptor blockade)
: Hypothyroidism
79. 50yr old male patient with liver cirrhosis presents with gross ascites and is admitted. Which
of the following diuretics is most suited for mobilising the edema fluid?
(A) Triamterene
(B) Furosemide
(C) Eplenerone
(D) Chlorothiazide
A patient with liver cirrhosis is at risk of developing portal hypertension as result of which there is
an increase in the production of vasodilators in the body.

Increased amounts of vasodilation cause splanchnic arteriolar vasodilation which cause a decrease in
arterial blood pressure and mimics hypovolemia.

This activates the sympathetic nervous system and renin - angiotensin - aldosterone system (RAAS)
causing increased Sodium and Water retention.

This later causes plasma volume expansion and accumulation of fluid in third space resulting in ascites
Therefore to prevent cirrhosis formation and collection of fluid in third space, sodium and water
: retention must be tackled.
Hence, aldosterone antagonists (spironolactone, Eplerenone) are the preferred drugs for
cirrhotic oedema.
80. Most effective drug used for smoking cessation
(A) Vareniciline
(B) Bupropion
(C) Nicotine gum
(D) Rimonabant
Drugs used in smoking cessation

Vareniciline
It is an α4β2 partial agonist.
: It is the most effective monotherapy for smoking cessation.
Side effects : Increases suicidal thoughts and depressive/psychotic

:
symptoms, sleep disturbances.

Bupropion
It is a norepinephrine - dopamine reuptake inhibitor (NDRI) and antagonist of several acetyl
choline receptor.
It is used as an antidepressant and in smoking cessation.
: It has less sexual side effects and weight gain compared to other anti depressants.
Side effects : Headache, Seizures, Tachycardia, Insomnia.

:
Nicotine Replacement Therapy
Patch, Gum, Lozenge, Spray (most effective form)
It is the most commonly used method.

:
81. Which of the following is an opioid anti-tussive?
(A) Diphenoxylate
(B) Levorphanol
(C) Ambroxol
(D) Levopropoxyphene
Levopropoxyphene
Levopropoxyphene is derived from propoxyphene which is an opiod.
Propoxyphene is a racemic mixture of levo - propoxyphene and dextro - propoxyphene.
: Levo - propoxyphene is used as an antitussive while dextro - propoxyphene is used as
an analgesic.

Diphenoxylate
It is a centrally active opiod drug used in the treatment of diarrhea.
: It is often combined with atropine to prevent drug abuse and overdose.
~

It acts by slowing intestinal contractions.


Levorphanol
It is an opiod drug used as an analgesic to treat moderate to severe pain.

:
Ambroxol
It is a mucolytic agent used in bronchopulmonary diseases.
It acts by breaking up phlegm, stimulating mucous production and stimulating release of
:surfactant by Type 2 pneumocytes.
Serves as anti glue factor by reducing adhesion of mucous to the bronchial wall.

82. Which of the following is used as an antidepressant?


(A) Lurasidone
(B) Vilazodone
(C) Asenapine
(D) Blonanserin
Vilazodone
It is used in the treatment of major depressive disorder.
: MOA : It inhibits serotonin reuptake like SSRIs and it is also a partial agonsit at 5HT1A receptors.
Adverse Effects : Nausea, Diarrhea, Headache, Insomnia
: It has less sexual side effects compared to SSRIs.
Lurasidone
It is an antipsychotic used in the treatment of schizophrenia and bipolar disorder.
: It acts as an antagonist at Dopamine and serotonin receptors and is a partial
agonist of 5HT1A receptor.

Asenapine
It is an atypical antipsychotic used to treat schizophrenia and acute mania associated

:
with bipolar disorder.

Blonanserin
It is an atypical antipsychotic used in treatment of Schizophrenia.

83. Which of the following anti-depressants are avoided in a patient with angle closure glaucoma?
(A) Amitryptiline
(B) Sertraline
(C) Mirtazapine
(D) Fluvoxamine
Angle Closure Glaucoma
I

Angle closure glaucoma occurs due to closure of drainage channels of aqueous humor
due to a narrow angle or bunching up of iris over the drainage canals when the pupil
enlarges too much or too quickly.
I

Therefore, anticholinergics are contraindicated in such patients as it will worsen the angle
closure by pupillary dilation.

Among antidepressants, maximum anticholinergic action is seen with tricyclic antidepressants


(TCAs) like Amitryptilline, Nortryptilline, Clomipramine, Imipramine, etc.
I

Hence, among the given options, amitryptilline is the best choice.


84. Which of the following is used in treatment of both seizures as well as neuropathic pain?
(i) Gabapentin
(ii) Lamotrigine
(iii) Carbamazepine
(iv) Pregabalin

(A) i, ii, iii


(B) ii, iii, iv
(C) i, iii, iv
(D) All the above

Among the given options, Gabapentin, Pregabalin and Carbamazepine are used in the treatment
of neuropathic pain and seizures.
Gabapentin and Pregabalin
MOA : Increase GABA release at synapse by binding to presynaptic calcium channels and
:blocking endocytosis of calcium.
They are mainly used for the treatment of partial seiures and peripheral neuropathy.
Other uses include management of post - herpetic neuralgia, bipolar disorder, generalised
:anxiety disorder and as migraine prophylaxis.
Side effects include weight gain and oedema.

Carbazepine
-
MOA : Sodium Channel Blocker
-
It is the drug of choice in the treatment of partial seizures and trigeminal neuralgia.
-
It is also used in the treatment of GTCS, mania and BPD.
-

Side Effects
Hypersensitivity Reactions
-

Stevens - Johnson Syndrome (associated with HLA B - 1502 gene)


: Agranulocytosis
Eosinophilia
S

Aplastic Anaemia
-

Ataxia, Diplopia
-

Splenomegaly
-

Hyponatremia due to increased ADH secretion (SIADH)


-

-
More common with Oxcarbamazepine.
-
Seen in elderly

Lamotrigene
l
MOA : Na+ and Ca2+ channel blocker.
Decrease Glutamate levels.
It is used as the first line drug in the treatment of GTCS, myoclonic seizures and partial seizures.
: Side Effects : Nausea, Vomitting, Ataxia, Diplopia, Stevens Johnson Syndrome

85. Combination of Saquinavir with which one of the following will improve its bioavailability
(A) Ritonavir
(B) Vitamin C
(C) Cimetidine
(D) Ganciclovir

Saquinavir is a protease inhibitor used in anti - retroviral therapy.


Protease Inhibitors
They are used in combination with other anti retroviral drugs to treat HIV 1 and HIV 2.
: They are used as first line drugs in children < 3 years of age and as second line in all
other age groups.
They are metabolised by CYP3A4 (except Nelfinavir -CYP2C19) and hence act as
: inhibitors of CYP3A4.
Ritonavir is the most potent inhibitor of CYP3A4 and is used in combination with other
protease inhibitors to boost their action and bioavailibility.
Eg : Lopinavir, Ritonavir, Saquinavir, Indinavir, Darunavir, etc.
: Side Effects : Diabetes, Dyslipidemia, Obesity, Hepatotoxicity.

In the given question, Ritonavir is the answer as it is a potent enzyme inhibitor of CYP3A4 and
improves the bioavailibility of Saquinavir when used in combination.

86. A patient on phenytoin for seizure disorder was prescribed sucralfate 4 times a day for
peptic ulcer. What should be the minimum duration between consumption of two drugs?
(A) 30min
(B) 60min
(C) 90min
(D) 120min
Sucralfate
Sucralfate is a gastroprotective agent used in the treatment of peptic ulcer disease,
: oesophageal ulcers and as prophylaxis in radiation proctitis and stress ulcers.
MOA : Increase prostaglandin synthesis, protects the ulcers by polymerisation into a viscous
substance, increases gastric pH.
C/I : Renal failure (because of presence of Aluminium), along with antacids and other drugs (will
: impair absorption due to high pH)
For a drug to be absorbed from the GIT, it must be able to cross the lipid barrier between
the GI wall and blood vessel.
Hence it must either be lipid soluble or unionised.
: Phenytoin is a weakly acidic drug and is absorbed from the stomach.
Sucralfate raises the gastric pH, hence phenytoin will be in ionised form and can’t be absorbed
: normally.
So, a minimum gap of 2 hours must be present between consumption of phenytoin and
sucralfate for phenytoin to be effective.
87. What is the mechanism of action of Remdesivir?
(A) Entry inhibitor
(B) Fusion inhibitor
(C) Inhibits DNA dependant RNA polymerase
(D) Inhibits RNA dependent RNA polymerase
Remdesivir
It is a nucleotide analogue that stops COVID - 19 viral replication by inhibiting RNA dependent
: RNA polymerase.
It was first approved for treatment of Ebola virus.
It is the only FDA approved drug used in the treatment of COVID - 19 infection.
: It is used only in moderate and severe COVID - 19 cases that require hospitalisation.
It has to be used within 10 days of disease onset.
: It is C/I in children (<12 years or <40 kg weight) and in mild cases of COVID - 19.
Dosage
Given as iv infusion over 30 - 120 minutes.
: For moderate COVID - 19 infection/ those not requiring ventilator support
Day 1 : 200mg OD
: Day 2 - Day 5 : 100mg OD
If not responding

100mg OD for another 5 days


:For severe COVID - 19 infection/those requiring ventilator support
Day 1 : 200 mg OD
: Day 2 - Day 10 : 100 mg OD.

Side Effects
Hypersensitivity/Anaphylaxis
: Liver toxicity : Discontinue drug if ALT/AST increases by 10 tims upper limit.
L
Renal toxicity : Decreased GFR

88. Which of the following cephalosporins can increase the effect of warfarin resulting in raised INR
and increased risk of bleeding?
(A) Cefoperazone
(B) Cefixime
(C) Ceftazidime
(D) Ceftibiprole
Cephalosporins
Cephalosporins that contain methylthiotetrazole can cause hypothrombinemia leading to

: bleeding manifestations.
Examples :
Cefoperazone
Moxalactam
: Cefomendole
Can cause disulfiram like reaction.

Cefotetan
: Cephalosporins that are safe in renal failure :
Cefoperazone
: Ceftriaxone
Cefpiramide
: Cephalosporins that are used in treatment of pseudomonas infection :
Cefoperazone
: Ceftazidine
Classification of Cephlosporins

Recent Update : 5th generation Cephalosporins - Ceftobiprole, Ceftaroline ( used in T/t of MRSA)
89. What is the mechanism of action of Cyclosporine?
(A) Calcineurin inhibitor
(B) mTOR inhibitor
(C) IL-2 receptor antagonist
(D) TNF-alpha inhibitor
Cylosporine
-

It is a calcineurin inhibitor.
-

It is used as an immunosuppressant medication.


-

It works by decreasing the funtion of lymphocytes by forming a complex with cyclophilin to


block the phosphatase activity of Calcineurin which in turn decreases the production of
inflammatory cytokines by T - lymphocytes.
Uses
Rheumatoid Arthritis
: Psoriasis
Graft v/s Host disease
: During Organ transplant
Atopic dermatitis
: Ulcerative Colitis
Side Effects
Convulsions, fever
: Diarrhea, Vomitting, Gum hyperplasia, Peptic Ulcers
Pancreatits, Kidney and liver dysfunction.
: Gout
90. Which of the following best describes the mechanism of action of organophosphorus
compounds in inhibition of acetylcholine esterase?
(A) Reversible inhibition
(B) Non-competitive inhibition
(C) Uncompetitive inhibition
(D) Irreversible inhibition
Mechanism of Action of OP Compounds
Acetyl Choline Esterase has two sites in it :

Acetyl Choline Esterase

Esteratic Anionic
Site Site
The anioic site accommodates the positive quarternary amine of Ach while the esteratic site
: hydrolyses the Ach into Acetate and Choline.
In case of OP poisoning, Organophosphates bind with PO4 in the esteratic site reversibly but later
the PO4 bond is replaced by a much stringer bond (ageing) making the binding irreversible.
This prevents ACh from binding to Acetyl Choline Esterase.

:
Types of Inhibition
Competitive Inhibition
Here the inhibitor and substrates are structural analogues.
: They bind to same active site on the enzyme.
Binding can be reversible or irreversible.
:Non - Competitive Inhibition
Here the inhibitor and substrates are not structural analogues.
: Inhibitor binds to seperate site on the enzyme (allosteric site)
Binding can be reversible or irreversible.

:
91. A 14 month old child was diagnosed with H. influenza pneumonia and was started on ampicillin
initially which was later changed to ceftriaxone. What is the rationale behind this decision?

(A) Cotrimoxazole is the drug of choice, but the patient is allergic to sulpha drugs.
(B) Ceftriaxone is the better IV drug compared to ampicillin
(C) Some strains of H. influenza have developed modified penicillin binding proteins
(D) H. influenza has developed resistance to penicillin based antibiotics because of beta-lactamase
production.
Among the given options, (B) is the most suited answer.
The drug of choice in H. Influenza infection is ampicillin.
But in recent years, there has been reports of resistance to ampicillin among H .
: Influenzae bacteria.
Mechanism of resistance :
Production of Beta Lactamase.
÷ Altering the penicillin binding target proteins.
In such cases, the most effective drug to treat infection in ceftriaxone.
Drugs like amoxicillin - clavulinic combination, 2nd and 3rd Generation Cephalosporins
can also be used.
Forensic Medicine
92. A patient is brought to OPD following a road traffic accident. A lacerated wound of 3x1cm,
bone deep, is noted over the scalp. Which of the following is the correct mechanism of
lacerated wound?
(A) Stretching of tissue
(B) Soft tissue crushed between weapon and bone
(C) Localised pressure with shearing forces
(D) Pushing/pulling force
Based on the given clinical history, the lacerated wound would most likely be a split laceration.
Laceration

Splitting of epidermis/dermis/subcutaneou tissue caused by a heavy blunt force.
Margins of the wound are irregular, crushing injury.
: On lens examination, crushed tissue bridges and crushed hair follicles are seen.
Types Of Laceration
Stretch laceration : Due to a tangential force on skin.
: Eg : Being run over by a vehicle.
Shearing Laceration : Skin tissue gets avulsed and remains attached by a
small margin of tissue (flaying).
Eg : Being run over by a vehicle at an acute angle.
Cut Laceration : Spindle shaped gaping wound also known as chop/slash wound.
: Eg : Axe Wounds
Split Laceration
Also known as incised looking lacerations usually seen on bony prominences.
:
(Scalp, chin, iliac crest, shin of tibia, eyebrow)
Caused by a perpendicular force that crushes the skin between bone and object,
there is no irregularity of wound margins, hence lens examination is done to confirm.

Examples of Lacerated Wounds


Other Wounds

Abrasion Incised Wound

Puncture Wound Contusion

93. A case of hanging is brought for autopsy. On examination right eye was open and saliva
dribbling from the angle of mouth was noted. Compression of which of the following structures is
the cause for opening of right eye?
(A) Carotid artery
(B) Internal Jugular Vein
(C) Cervical sympathetic chain
(D) Vagus nerve
Hanging
Hanging is a type of asphyxial death in which there is neck compression and body suspension.
:
It is the most common method of suicide in India.
Types of Hanging
Complete Hanging : Complete suspension of body death occurs due to asphyxia.
: Incomplete Hanging : Partial hanging, some part of body is touching the ground.
Cause of death is bilateral venous congestion.
Typical Hanging : Knot of the noose isplaced on the occiput.
: Atypical hanging : Knot is placed at any other place than occiput.

Post Mortem changes in hanging


Asphyxial Triad (+) : Cyanosis, Petechiae, Congestion
: Protrusion of Tongue
Tardieu Spots (purple to black spots on skin due to ruptue of capillaries)
: Semen emission
Ligature Mark
: Most specific finding
V shaped mark seen on neck above thyroid cartilage
: Pressure abrasion/contusion
Hyoid fracture : Abduction fracture
: Amussat Sign : Tears of walls of Carotid Artery
Simon’s Sign : Haemorrhage on the anterior aspect of intervertebral discs.

:
(Lower Thoracic and Lumbar Spine)

Antemortem changes in hanging


Dribbling of saliva
Due to compression of submandibular gland and stimulation of pterygopalatine ganglion.
: Seen on opposite side of knot.
>
La facies Sympathique
1

Eye on the same side of knot remains open and pupil dilated due to compression of
cervical sympathetic ganglion.

94. A person was brought dead to casualty following an explosion. Multiple abrasions and lacerations
were noted on right side of chest and abdomen which were close together. These injuries were caused
by?
(A) Explosion burns
(B) Flying missiles
(C) Blast air pressure changes
(D) Collapse of building
Explosion Injuries
Primary Blast Injury
\

Caused due to blast waves (compression of surrounding air).


Also called barotrauma.
: Organs containing air are commonly injured.
Most sensitive for primary blast injury : Tympanic Membrane
: Most common organ that is damaged : Lung
Most common organ severely dmaged underwater : GI tract (Intestine)

:
Secondary Blast Injuries
Caused by missiles which are produced/generated by the blast.
-

Can cause Abrasions, Contusions, Lacerations (punctate) : Marshall Triad

Tertiary Blast Injury


-

Caused when person is thrown away by the explosion and hitting another object.

Quarternay Blast Injury


-

Caused by burns or flames.

95. A person comes to OPD with an injury to right thigh. On examination a soddened burn injury is
noted with erythema, blisters and line of demarcation which extends down the limb. What is the
most likely cause?
(A) Chemical burn
(B) Dry heat
(C) Electric burn
(D) Moist burn
Based on the given history, the patient appears to have sustained a scald.

Burn Injuries
It is due to application of heat either to internal or external surface of body.

:
Scald/Moist heat burn
Burns produced by hot liquid > 60°C or by steam.
Water at 70°C or more can produce full thicknes scalds in one second.
: On examination, skin is red, blisters seen.
There is no singeing of hair, destructin of skin or blackening of skin.
: Clear line of demarcation is present at site of contact.
Dry heat burn
/
Produced by application of flame or hot object on the skin.
On examination, there is destruction of skin, singeing of hair or blackening of skin.

:
Electrical Burn
Burns sustained during electrocution.
Entry burn mark : Marks the site of electricity aka endogenous burn / Joule burn.
: High voltage sparks can also cause crease burns (flash burns) which can form multiple
punched out lesions called crocodile flash burns.

Lightning Burn
Most commonly seen on shoulder flanks.
:
Burns have typical branching tree pattern (Filigree Burns/Arborescent Burns/
Lichtenberg flower petal burns)
Mechanism : Staining of blood vessels by haemoglobin from denatured RBCs.

:
96. A 25yr old male working in a dye manufacturing company presents to the OPD with pain in the
throat followed by vomiting, dark bloody stools and inflamed conjunctiva. Which of the following
compounds is most likely responsible?
(A) Potassium permanganate
(B) Lead sulphate
(C) Arsenic compound
(D) Thallium sulphate

Based on the given history patient is most likely suffering from acute arsenic toxicity.
Arsenic Poisoning
Arsenic compounds are toxic, arsenic as such is non - toxic.
Patient can present with garlicky odour, nausea, vomitting, burning pain in throat and abdomen,
: conjunctivitis and painful micturition.
On examination :
Red velvety mucosa seen (strawberry mucosa).
: Hyperpigmentation/hyperkeratosis of palms and soles present.
Aldrich Mees line present (white transverse lines in nail).
: Black foot seen.
Arsenic can also cause neuropathy and skin cancers.
:
Antidote for Arsenic poisoning : BAL (Dimercaprol), DMSA, Iron Oxide
I Acute arsenic poisoning can mimic cholera disease.
Acute Arsenic Poisoning Cholera
Vomitting first followed by diarrhea. Diarrhea followed by vomitting.
Vomitus contain blood and mucous. Vomitus is watery.
Purging seen after vomitting. Purging before vomittimg.
Stools are dark, bloody, contain mucous Rice watery stools, no blood seen.
Tenesmus and anal irritation present. No tenesmus.

Thallium Poisoning
I

Triad
1

Alopecia
Painful Neuropathy
:
Skin Rashes
Ideal homicide poison.
: Lateral eyebrow sign is classical.
(Lateral 2/3rd of eyebrow falls off)

Potassium Permanganate (KMnO4) Poisoning


Patient complains of burning pain of lips, mouth, throat and stomach.
: On examination; lips, gums, teeth and tongue are discoloured.
I

Initially purple in colour which turns dark brown later jet black.

Lead Poisoning
-

Plumbism/Saturnism
-

Most Common compound involved is lead acetate.


-

Most Common route : Inhalation


-

Patient presents with influenza like syndrome having fever, chills, myalgia and headache.
-

Other features
Anaemia
-

Basophilic Stippling
-

Colicky Abdominal pain


-

Foot drop/wrist drop and encephalopathy


-

Characteristic blue lines on gums are seen (Burtonian Lines)


-

97. Two friends after consuming alcohol gets into a fight. One of them pushes the other onto the
floor, covers his face preventing him from breathing while sitting on his chest. This is known as:
(A) Smothering
(B) Traumatic asphyxia
(C) Burking
(D) Overlaying
Based on the history given the mechanism of death here is burking.
The person has died due to asphyxia by suffocation.
Suffocation
It is the purest form of asphyxia.
:
Death occurs due to lack of Oxygen.
Types
^
Choking : Asphyxia due to obstruction of trachea, bronchi or bronchiole by food particle during
eating.
^
Gagging : Suffocation due to obstruction of oropharynx by a cloth piece.
^
Smothering : Suffocation due to blockage of both nasal airway and mouth.
It is a combination of choking and gagging.
^
Traumatic Asphyxia : Here there is restriction of movement of chestwall thereby stopping
breathing, seen in incidents of building collapses where the chest
gets fixed.
^
Burking : It is a combination of smothering and traumatic asphyxia.
A person lies on the chest of the victim, preventing breathing and also covers the
nose and mouth (face) with his hands.
^
Overlaying : Here the victim is put under the body weight of another person resulting in
chest fixation and blockage of nasal airway.
Seen in cases of mother lying over her child.
^
Cafe Coronary : Sudden, unexpected, accidental death due to obstruction of pharynx/larynx
by food bolus while eating.
Death mostly occurs due to vasovagal reflex inhibition leading to cardiac
arrest.

98. Identify the laws dealing with insanity


(i) Curren’s
(ii) Durham
(iii) Ashby
(iv) Murphy

(A) i & iii


(B) ii & iv
(C) i & ii
(D) ii & iv
Laws for Defence of Insane

McNaughten’s Rule
Accepted in India under section 84 of IPC.
: “Nothing is an offence done by a person who, at the time of doing it, by the reason of
unsoundness of mind is unable to understand the nature and consequences of the act which he
is doing (or) that what he is doing wrong (or) contrary to the law.”
It is called the right or wrong test.

:
Durhams’s Rule
Not used presently.
Aka “The product of mental illness” test.
: Acts that are the “products” of mental disease or defect excuse criminal liability.

Curren’s Rule
Not used presently.
:
Superior to Durham’s rule.
It states that an accused person will not be criminally responsible, if at the time of committing the

:
act, he did not have the capacity to regulate his conduct to the requirement of law, as a result of
mental disease or defect.

American Law Institute test


Most common followed rule in USA.
It states that a person is not responsible for criminal conduct if at the time of such conduct, as a

:
result of mental disease (or) defect he lacks adequate capacity to either appreciate the
criminality of the conduct (or) to adjust his conduct to the requirements of law.

Irresistible Impulse Test


Not used presently.

99. Match the following in relation to fingerprint changes:


(a) Ridge atrophy (i) Dermatitis
(b) Permanent loss of fingerprints (ii) Acromegaly
(c) Ridge alteration (iii) Radiation
(d) Loss of pattern with ridge atrophy (iv) Scleroderma
(e) Increased distance between ridges (v) Coeliac disease

(A) a-i, b-iii, c-iv, d-v, e-ii


(B) a-iv, b-v, c-ii, d-iii, e-i
(C) a-iii, b-iv, c-i, d-v, e-ii
(D) a-ii, b-iii, c-i, d-iv, e-v
Fingerprinting
Fingerprints are papillary ridges on fingers which are unique.
:
Fingerprints are the best method for criminal identification as it is not even identical
in identical twins (Quetlet Rule).
Study of fingerprints : Dermatoglyphyics
: Fingerprints start developing in foetus at 12 - 16 weeks of IUL and is completed by 24 weeks.

Types
Loop (Most Common)
÷ Whorl
Arch
÷ Composite

Permanent loss of fingerprints occur in leprosy, electorcution (charring) and radiation.


÷ Ridge atrophy is seen in Coeliac disease, dermatitis.
Ridge alteration is seen in acanthosis nigricans scleroderma, eczema.
÷ Increased space between ridges is noted in acromegaly.

100. Police officials during their routine inspection in a nearby forest came across the skeletal
remains of a dead body. They brought them to the hospital for forensic examination and
identification. Only the pelvis was in good shape. Which of the following characteristics is
suggestive of a male pelvis :
(i) inverted ischial tuberosity
(ii) U shaped subpubic angle
(iii) small, deep, narrow greater sciatic notch
(iv) large acetabulum
(v) ischiopubic index 80
(vi) small triangular obturator foramen

(A) i, ii, iii, iv, v


(B) i, iii, iv, vi
(C) iii, iv, v
(D) ii, iii, v

Pelvis is the best bone for sex determination according to Krogman’s table.
Adult sex can be determined from skeleton.
: According to Krogman accuracy in sexing, adult skeletal remians is:
ENTIRE SKELETON : 100%
: PEVIS + SKULL : 98%
PELVIS ALONE : 95%
: SKULL ALONE : 90%
I
LONG BONES ALONE : 80%

Bony Pelvis Male Female


General structure Thick and heavy Thin and light
Greater pelvis (false pelvis) Deep Shallow
Lesser pelvis (true pelvis) Narrow and deep, tapering Wide and shallow, cylindrical
Pelvic inlet (superior pelvic aperture) Heart-shaped, narrow Oval and rounded; Wide
Pelvic outlet (inferior pelvic aporture) Comparatively small Comparatively large
Pubic arch and subpubic angle Narrow (<70%) Wide (>80°)
Obturator foramen Round Oval
Acetabulum Large Small
Greater sciatic notch Narrow(-70°); inverted V Almost 90°
Ischial tuberosity inverted everted
Ischiopubic index 101.05 +/- 16.65 115.99 +/- 18.5
Sciatic notch index 4-5 5-6
Sacral index 95.42 +/- 13.14 111.27+/- 7.66
ENT
101. Identify the hearing aid used for bilateral canal atresia

(A) (B)

(C) (D)

In case of bilateral external auditory canal atresia, the device used is bone - anchored hearing aid.
(BAHA)
Bone Anchored Hearing Aid (BAHA)

Indications for BAHA


When air-conduction (AC) hearing aid cannot be used:
: Canal atresia, congenital or acquired, not amenable to treatment.
Chronic ear discharge, not amenable to treatment.
:
Excessive feedback and discomfort from air-conduction hearing aid.
Conductive or mixed hearing loss, e.g. otosclerosis and tympanosclerosis where surgery is
: contraindicated.
Single-sided hearing loss.
BAHA replaces the pinna, external
auditory canal and middle ear.
It directly stimulates inner ear.
BAHA can be used as early as 3 - 5 years of age.

Components of BAHA
I

Titanium screw
Sends a vibration to cochlea through skull by bone conduction.
: It is integrated with bone.
Abutment
: Attaches speech processor to the titanium screw.
Speech processor
: Acts as transducer.
Captures sound via microphone and passes the vibrations to implant via abutment.

:
Other options

Cochlear Implant
Replaces the organ of Corti.
Acts by stimulating cochlear nerve.
: Indications
B/L severe Sensory Neural hearing loss.
: No benefit from hearing aids despite using them for 3 months.
Stapes Prosthesis
Used in surgeries for replacing Stapes (Stapedectomy/Stapedotomy).
: Commonly used in otosclerosis patients.

Ossicular Reconstruction Prosthesis


I

Used as replacement for ear ossicles in case of ossicular necrosis.

102. Identify the marked structure in the given NCCT scan


(A) Pneumatised Superior Turbinate
(B) Onodi cells
(C) Haller cells
(D) Concha bullosa

The given images shows an NCCT of sinuses (coronal view) with the marker showing a pneumatised
superior turbinate.
Normal NCCT of sinus (coronal view)

Other options

Haller Cell
It is a posterior group ethmoidal air sinus present on the floor of the orbital cavity.
<

Infection here can lead to orbital cellulitis.

:
Concha Bullosa
It refers to a pneumatised middle turbinate.
Onodi Cell
It is the posterior most ethmoidal air cell and is closely related to optic nerve and internal

:
carotid artery which can get injured during sinus surgery.

103. Post total laryngectomy rehabilitation, which of the following is not used.
(A) Polite yawning
(B) Supraglottic prosthesis
(C) Oesophageal speech
(D) Tracheo-oesophageal prosthesis
After total laryngectomy, supraglottic prosthesis can’t be used as it can’t be fixed properly in the
absence of larynx.

Other Options

Polite Yawning
Done in patients who have undergone laryngectomy as part of rehabilitation of olfaction.
I
The patient is told to yawn with mouth closed which induces a negative pressure in the
oral cavity and oropharynx generating a nasal airflow enablng odorous substances to
reach the olfactory epithelium again.
Oesophageal Speech
L

Done as part of speech rehabilitation in patients who have undergone laryngectomy.


Patient is asked to swallow air and use it for speech production.
: This technique is difficult to master for most patients.

Tracheo - oesophgeal prosthesis


This is the best technique for production of voice in a post laryngectomy patient.
: Also known as voice prosthesis.
A valve is placed which on closure during expiration leads to air in hypopharynx which in turn
: is used for speech production.
Valve used is termed as Blom singer valve.
Electrolarynx
It is another modlity used for speech production after laryngectomy.
: It vibrates tracheo - oesophageal segment which can be used for speech.

104. Patient was operated for safe CSOM in right ear. He then presented with complaints of vertigo,
tinnitus & ear fullness. He experienced relief on turning his head to the opposite side. Identify his
condition.
(A) Paget’s disease
(B) Perilymphatic fistula
(C) Labyrinthitis
(D) Schwannoma
According to the given clinical history, patient develops tinnitus, vertigo and ear fullness
: which are signs of labyrinth (inner ear) pathology.
He developed his symptoms just after surgery for CSOM which imples that he could have had
an inner ear injury during the surgery.
Among the given options, perilymphatic fistula is most appropriate as it is common after

:
middle ear surgeries like that of CSOM (mastoidectomy and tympanioplasty) and relief on
turning head to opposite side is more suggestive of fistula formation.
Perilymphatic fistula
Formation of abnormal communication between the perilymph filled inner ear and outside the
inner ear (most commonly middle ear via oval or round window).
It mainly occurs following head trauma, barotrauma, ear infections, ear surgeries, etc.
: Patients often complain of hearing loss, vertigo, nausea, tinnitus and imbalance.
It is best diagnosed by Gadolinium enhanced MRI and clinical examination.
: Treatment is often almost always by surgical exploration.

Other options

Paget’s disease
Also known as osteitis deformans.
: There is excessive bone breakdown resulting in subsequent disorganised new
bone formation.
This results in structurally weak bones.
:
Patients can develop hearing loss, vision loss and other cranial nerve
symptoms due to narrowing of foramina of skull due to excessive bone
formation resulting in nerve compression and palsy.
Labyrinthitis
It refers to inflammation of inner ear.
:
It can be idiopathic or due to a viral infection, bacterial infection, head injury, allergy, etc.
Patient presents with vertigo, nystagmus, nausea, tinnitus and ear ache.
: Treatment depends on the causative agent.

Schwannoma
Aka Acoustic Neuroma.
: Most common tumour of Cerebellopontine angle.
It is slow growing, benign and locally invasive.
:
It can cause tinnitus, unilateral SNHL and can involve other cranial nerves.
(CN5, CN6, CN9, CN10, CN11)

105. A 76yr old female patient presented to the ENT OPD complaining of chronic nasal
discharge with occasional epistaxis. Her CT scan showed the following findings. What could be the
possible diagnosis?
(A) Nasopharyngeal angiofibroma
(B) Inverted papilloma
(C) Esthesioneuroblastoma
(D) Maxillary carcinoma

Based on the history given, the most likely diagnosis is an inverted papilloma.
Points in favour of Inverted Papilloma :
Arising from nose (more specifically, lateral part of nose)
: No evidence of bony erosion or intracramial extension.
I
Clinical symptoms of nasal discharge, epistaxis.
Inverted Papilloma
Most common tumour of nasal cavity.
: Aka Ringertz tumor or Schneiderian Papilloma or Transitional cell Papilloma.
It is caused by HPV virus and arises from lateral wall of nose.
: It is locally invasive and recurrent in nature.
Patient presents with nasal congestion, discharge, epistaxis.
: Management : Surgical Excision of tumour using CO2 laser.
Radiotherapy is contraindicated as it can lead to malignant transformation.

:
Other Options
Nasopharyngeal Angiofibroma
It is a benign tumour of nasopharynx seen in adolescent males.
It arises from sphenopalatine foramen and is locally invasive.
: Mostly presents with recurrent epistaxis.
CECT shows characteristic Hollman Miller Sign/Antral Sign.
: Management is by surgical excision.

Esthesioneuroblastoma
It is a rare cancer of nasal cavity that arises from sensory neuroepithelial cells (olfactory
: epithelium).
It is highly invasive and almost always show intracranial extension.
Patients c/o nasal obstruction, epistaxis, hyposmia, etc
: Preferred treatment is by surgery followed by radiotherpy to prevent recurrence of tumour.

Maxillary Carcinoma
It is a squamous cell carcinoma of maxillary sinus.
: It results in blockage of maxillary sinus resulting in sinus headache, tenderness of sinus,
epistaxis, earache, etc.
It is malignant in nature and can erode bone to spread to infratemporal fossa, pterygopalatine
: fossa leading to symptoms like trismus, facial swelling, proptosis, diplopia, etc.
Management for all stages is surgery followed by radiotherapy.
Ophthalmology
106. Identify the given ophthalmologic finding:
(A) Vossius ring
(B) Weiss ring
(C) Kayser Fleischer ring
(D) Arcus senilis
The given image shows a Vossius ring.
Vossius Ring
-

It is commonly seen after blunt truma to the eye.


-

A circular ring of fainted or strippled opacity is seen on the anterior surface of the lens due to
brown amorphous granules of pigment lying on the capsule.
-

It is due to the impression of the iris on the lens during trauma.

Other Options
Weiss Ring
Weiss ring is a large type of floater usually seen in cases of posterior vitreous detachment.
: It is removed with the help of lasers.
Kayser Fleischer Ring
I

They are dark rings that encircle the iris of eye.


This occurs due to Copper deposition in the Descemet membrane layer of cornea.
: Seen in conditions like Wilson’s disease, Chalcosis, etc.

Arcus Senilis
It is ring in peripheral cornea caused by cholesterol deposits.
: Usually seen in elderly especially those with hyperlipidemia.

107. Identify the following chart


(A) Hess chart
(B) ETDRS chart
(C) Pelli Robson chart
(D) Snellen’s chart

The given image shows a Hess chart.


Hess Chart
U
It is used in assessing diplopia; to asses the degree of paresis, measurement of
deviation, udentify the paralysed muscle and differentiate between mechanical and
neurogenic cause.

Other options

ETDRS Chart
It is a chart used to assess visula acuity in diabetic patients.

:
ETDRS stands for Early Treatment for Diabetic Retinopathy Study.

Pelli - Robson chart


It mesures a patient’s contrast sensitivity by finding the lowest contrast letters he/
she can read correctly.
Snellen’s Chart
\
It is used to assess visual acuity in a person.

108. Identify the site of lesion in the optic pathway based on the given visual field study of a patient
(A) Optic nerve
(B) Optic chiasma
(C) Optic tract
(D) Occipital lobe

The given image shown an example of bitemporal hemianopia due to injury at the optic chiasma.

Optic Pathway
109. Which lobe of brain is involved in hand-eye coordination?
(A) Frontal
(B) Temporal
(C) Parietal
(D) Occipital

Among the options given, parietal lobe is the most appropriate answer.

Hand - Eye Coordination


Hand - eye coordination is the coordinated control of eye movement with hand movement and

:the processing of visual input to guide reaching and grasping along with the use of
proprioception of the hands to guide the eyes.
The neural control of hand - eye coordination involves the eyes, cerebral cortex,
subcortical structures (cerebellum, basal ganglia, brain stem), spinal cord and the
peripheral nervous system.
e
Frontal and parietal lobes are involved, more specifically the posterior parietal cortex and
parieto - occipital junction are believed to play an important role in relating proprioception
and the transformation of motor sensory input to plan and control movement with regard
to visual input.
110. Most common ophthalmological manifestation of COVID-19
(A) Follicular conjunctivitis
(B) Viral keratouveitis
(C) Peudomembrane conjunctivitis
(D) Retinal vein thrombosis
n

Most common ophthalmic manifestation of COVID - 19 is follicular conjunctivitis (88% of


ophthalmic manifestations).
It is the inflammation of conjunctiva that is usually associated with viral infections.
:
Examination shows appearance of follicles in the palpebral conjunctiva.

Other Options

Viral Keratouveitis
Most commonly caused by Herpes Simplex Virus type I.
:
It involves the corneal epithelium (ulcer) and can also involve the stroma (immune
reaction against viral antigens).

Pseudomembrane conjunctivitis
It is a rare case of conjunctivitis characterised by mucopurulent discharge and
:pseudomembrane formation.
It is caused by Corynebacterium, Neisseria, Streptococcus and Adenovirus.

Retinal Vein Thrombosis


It can be associated with mucormycosis infection.
: Infection can trigger coagulation cascade in retinal vessels leading to formtion of
thrombus and occlusion of vessels.
111. Which step of cataract surgery is shown in the image?
(A) Hydrodissection
(B) Lens aspiration
(C) IOL insertion
(D) Capsulorrhexis

The given image shows the capsulorrhexis step of cataract surgery.

Cataract Surgery

2 techniques
-

Intracapsular
-

Cataract/Lens removed with the capsule intact.


-

Only done in case of lens dislocation.


-

Extracapsular
-

Lens is removed but capsule is left behind.


-

Includes
- Extra Capsular Cataract Extraction (ECCE)
- Small Incision Catarct Surgery
- Phacoemulsification
- FLACS (Femtosecond Laser Assisted Catarct Surgery)

Steps of Cataract Surgery

Anaesthesia
General Anaesthesia in children
: Local Anaesthesia (Lignocaine + Bupivacaine + Hyaluronidase +/- Adrenaline)
Retrobulbar Nerve Block
: Peribulbar Nerve Block
Topical Anaesthesia
: Using lignocaine, Paracaine, used only in phacoemulsification.
Scrubbing
4

Using Povidone Iodine solution


- 10% for peri - orbital skin
- 5% for conjunctiva

Incision
Superior limbal incision of 8 - 9 mm length made after peritomy (cutting and
: opening of conjunctiva) in case of ECCE.
Superior or temporal small incision of 6 - 7mm is used in SICS, following which a
sclerocorneal tunnel is made (hallmark of SICS, self healing incision).
Peripheral clear corneal incision of 2.7 - 3.2 mm used in phacoemulsification.

:
Anterior Chamber Entry
After incision, aqueous comes out making cornea flat.
AC in inflated using irrigating solution or by viscoelastics.
: Eg : Balanced Salt Solution plus (BSS + Glutathione), Normal Saline, Ringer Lactate,
HPML, Sodium Hyaluronase.

Anterior Capsulotomy
Anterior Capsule is stained using Trypan blue and is opened up.
: 2 techniques
Can opener technique
: Continuous Curvilinear Capsulorrhexis (CCC)

Removal of cataractous lens


In ECCE, hydrodissection is done.
: Sepereation of lens from capsule by injecting fluid beneath capsule.
Lens is delivered out as a whole.

:
In SICS
Hydrodelineation is done after hydrodissection.
Seperation of nucleus and cortex of lens.
: Nucleus is delivered and cornea is aspirated.

In phacoemulsification
Nucear fragmentation is done using ultrasound energy.
: (Phacoprobe with Titanium needle vibrating at 40,000 Hz)
Nuclear fragment and cortex is aspirated.
IOL Implantation
Rigid IOL for ECCE and SICS.
: Foldable IOL for Phacoemulsification.
I
Viscoelastics are removed, AC is reformed and incision is closed.
SPM
112. Human development index includes
(A) Life expectancy at one year, income, literacy rate
(B) Life expectancy at birth, decent living standard, knowledge
(C) Life expectancy at one year, income, decent living standard
(D) Infant mortality rate, decent living standard, knowledge

Human Development Index

Human Development Index (HDI) is an indicator used in assessing the overall well being of
: a population.
It is a geometric mean.
It has the following components :

:
- Long and healthy life : Life exepctancy index.
- Knowledge
Mean schooling years
Expected years of schools

:
- Decent Standard of living : Income (Gross National Income per capita)
Expressed in terms of purchasing power of parity (PPP) in USD.

Indices Min Max


Life expectancy index 20 years 83.2 years
Mean school years 0 13.2 years
Expected School years 0 20.6 years
Income per year 163 USD 1082111 USD

Index = Actual value - Minimum value


Max value - Minimum value

HDI is a geometric mean of all indices.


Range of HDI : 0 - 1.
: HDI of India : 0.645 (131st position - 2020)
113. A drug reduces the mortality of a disease but doesn’t cure it. Which of the statements
are true in this case?
(A) Prevalence of disease increases
(B) Prevalence of disease decreases
(C) Incidence of disease increase
(D) Incidence of disease decrease

Prevalence
Prevalence of a disease refers to the total number of cases of that disease in a community at a

:
particular point of time.
Prevalence = No : of existing cases (At a point of time)
Total number of population
Incidence of a disease refers to the number of new cases of that disease that arise in a community
for a specified period of time.
Incidence = No : of new cases
(Per unit of time)
No : of people at risk of acquiring the disease

According to question, the new drug helps reduce mortality of a disease but doesn’t cure
: the disease.
So the total no : of cases in the community will keep rising as survival of diseased
people is increased.
There is no change in incidence of the disease as nothing is done to prevent the
: occurence/increase the occurence of the disease.
Therefore, the prevalence of the disease increases while the incidence remains the
same.

114. Which of the following steps are not part of case control study?
(A) Follow up
(B) Matching
(C) Inference of results
(D) Selection of subjects
Follow up is a part of cohort study not case control study.
Case Control Study
Also known as retrospective study.
: It has 3 distinct features
Both exposure and outcome (disease) have occured before the start of study.
: Study proceeds backwards from effect to cause.
L
Uses a control or comparison group to support or refuse an inference.
four basic steps
i.

Selection of cases and controls


Selection of cases
i.

- Defining diagnostic and eligibility criteria of cases.


- Identifying sources of cases.
Selection of controls
i.

i.

Matching
The process by which controls are selected in such a way that they are similar to the cases
i.

with regard to certain selected variables (Eg : Age, smoking, etc)


i.

Measurement of Exposure
To measure the exposure of a particular factor in the cases and controls.
i.

i.

Analysis
To find out :
i.

- Exposure rates among cases and controls to suspected factor.


- Estimation of disease risk associated with exposure (Odd’s Ratio).

115. Which of the following is the most suited to find out the strength of association between exposure and out
(A) Cohort study
(B) Case control
(C) Cross sectional
(D) Ecological study

The best suited study to asssess the strength of association between exposure and outcome is
cohort study.

Cohort Study

Also known as prospective study, longitudinal study or incidence study.


: Proceeds from cause to effect.
Features
Cohorts are identified prior to the appearance of the disease under investigation.
: The study groups so defined are observed over a period of time to determine the
frequency of disease among them.
The study proceeds forward from cause to effect.

:
Indications
To find out strength of association between exposure and disease.
When attrition of study population can be minimised.
: When exposure is rare (rare risk factor), but the incidence of disease is high among exposed.
Other Options

Case control study


Also known as retrospective studies.
: It proceeds from effect to cause.
It helps in studying about rare diseases.

:
Cross - Sectional Study
Also known as prevalence study/snapshot.
Whole/sample of population is taken up and studied.

:
Ecological Study
The unit of study is population.
It is used to study frequency of disease and frequency of risk - related factors across

:
populations.

116. Meta-analysis consists of all the following steps except


(A) Randomisation
(B) Selection
(C) Abstraction
(D) Analysis
Meta - Analysis
Meta - analysis is a method of statistical analysis that pools summary data (Eg : means, relative
risk, etc) from multiple studies for a more precise estimate of the size of an effect.
It also estimates the heterogenecity of effect sizes between studies.
:
It helps improve the power, strength of evidence and generalisability of study findings.
It is limited by quality of individual studies and bias in study selection.
: A good meta - analysis study has 4 basic steps :
Identification
: Selection
Abstraction
:
Analysis
a
Randomisation is not a step in meta - analysis.
117. Recent increase in NCD is an example of which of the following?
(A) Seasonal trend
(B) Endemicity
(C) Secular trend
(D) Cyclical trend
Based on the frequency of disease in a population in relation to time distribution, diseases can have :
- Short term fluctuations (Epidemics)
- Periodic fluctuations (Cycles or Seasonal)
- Long term fluctuations (Secular Trends)
Epidemics
Sudden rise in number of cases which is clearly in excess which is >80% or > 2 SD more
:than expected frequency.
Types
Slow epidemics
: Gradual increase, may rise or fall with time.
Eg : DM, Obesity, Mental health problems.
:
Common Source
Single exposure

: - Lasts for only one incubation period, sudden increase and fall in cases.
- Eg : Food poisoning.
Multiple exposure
- Extends beyond one IP, gradual rise and fall of disease.
- Eg : Contaminated well, food handler with Typhoid.
Propagated
: Spreads from person to person.
Gradual rise in number of cases, sometimes explosive.
: Eg : Dengue, Malaria, HIV/AIDS.

Periodic Fluctations

Seasonal Trend
Frequency varies with season/climate.
: Eg : Increased incidences of RTA, dengue, malaria in monsoon season.

Cyclical Trend
Disease comes in cycles.
: Eg : Influenza every 7 - 10 hours.
Long Term Fluctuations (Secular Trends)
Diseases increase or decrease over long periods of time.
: They are unidirectional (either decrease or increase).
Eg : NCDs have shown secular trend of increase in number of cases.

:
118. External validity of a test implies
(A) Objectivity
(B) Stability
(C) Generalisability
(D) Replaceability
External Validity
It is the validity of applying the conclusions of a scientific study outside the context of
that study.
It represents the extent to which the results of a study can be generalised to and across
other situations, people and times.
It is an important property of any study because general conclusions are almost always a
goal in research.

Internal Validity
It is the validity of conclusions drawn within the context of a particular study.

:
119. Hepatitis-B subunit vaccine is derived from which antigen?
(A) HBeAg
(B) HBsAg
(C) HBcAg
(D) HBV DNA

Hepatitis - B Vaccine
It is a type of recombinant vaccine on Hepatitis B surface antigen (HBsAg) inserted into yeast
(Saccharomyces).
It was the first human vaccine produced by recombinant DNA technology.
: Schedule
At birth : Hep B birth dose
: At 6, 10, 14 weeks : Hep B (Pentavalent vaccine)
I
In adults : 0,1,6 months
120. Match the correct option according to the image given below:

(A) 1-normal, 2-positive skew, 3-negative skew, 4-outliers


(B) 1-positive skew, 2-normal, 3-negative skew, 4-outliers
(C) 1-normal, 2-outliers, 3-negative skew, 4-positive skew
(D) 1-normal, 2-outliers, 3-positive skew, 4-negative skew
The given question shows the distribution of four Box and Whisker Plots.
Box and Whisker Plot

(i)

Normal Distribution
(Bilaterally Symmetrical)

Q1 Q2 Q3

Min Max

(ii)

Normal Distribution
With Outliers

Outliers

Q1 Q2 Q3

Min Max
(iii)

Right Skew/Positive Skew


(Longer quartile is present towards right)

Q1 Q2 Q3

Min Max

(iv)

Left Skew/Negative Skew


(Longer Quartile is present towards left)

Q1 Q2 Q3

Min Max

Normal Distribution Left Right


With Outliers

rest
Normal Skew Skew
Distribution
121. Age adjusted mortality rates are used in
(A) To compare mortality rates of two different populations
(B) To identify the proportion of deaths due to a particular disease
(C) To assess the population distribution according to age
(D) To identify the case fatality rates
Age adjusted mortality rates
I
Age adjusted/Age specific death rates refers top the death rates in a population distributed
according to the age groups.
/
It is useful in comparing mortality rates of two different populations by calculating
standardised mortality rates.
/
Standardisation is of two types :
Direct Standardisation : When age specific death rate and standard population is available.
: Indirect Standardisation : When age - specific death rate is not available.
Anaesthesia
122. Which of the following agents can be used as sole agent for induction and intubation?
(A) Sevoflurane
(B) Isoflurane
(C) Halothane
(D) Desflurane
s

Sevoflurane is the best answer among the given options.


Sevoflurane has the smoothest induction and is the least irritative to respiratory system.
: It is the preferred agent in day - care surgeries.
Other Options
Isoflurane
It releases bronchioles.
: Clinically not used in asthmatics as it has a pungent nature that is irritative to the airways.
Desflurane
It is also avoided in patients with reactive airway diseases because of its pungent nature.
: It produces the maximum inflammatory response in the respiratory system.

Halothane
It produces the maximum bronchodilatation.
: It can cause nodal arrhythmia/junctional bradycardia and is hepatotoxic as well.

123. Emergency cardiopulmonary resuscitation followed by rapid administration of 20% intralipid


solution is done following
(A) local anaesthetic administration
(B) pulseless electrical activity
(C) electrocution
(D) drowning
Emergency CPR and administration of 20% intralipid solution is done following local anesthetic
toxicity (Bupivacaine toxicity).

Bupivacaine
It is a long acting local anaesthetic.
: It is used in spinal anaesthesia especially as part of labour analgesia as it has good differential
blockade property.
Dose : 2 - 3 mg/kg.
: It can cause arrhythmias in toxic doses which is resistant to treatment.
Management of Arrythmia
Cardiopulmonary resuscitation initially followed by administration of 20% intra - lipid solution.
: Initial bolus dose of 1.5ml/kg followed by 0.5 ml/kg/hr maintenance dose.

Other toxicities of LA
Lignocaine : Can cause epilepsy.
: Prilocaine & Benzocaine : Cause Methaemoglobinemia.

124. Maximum rate of O2 delivery by conventional O2 therapy


(A) 5l/min
(B) 10l/min
(C) 15l/min
(D) 20l/min
Conventional O2 therapy refers to simple O2 mask usage.
O2 delivery devices

O2 delivery devices

Variable Performance (Low Flow) Fixed Performance (High Flow)


I
Nasal Prongs 7
Venturi Mask
Simple O2 mask
: O2 mask with reservoir bag.
Nasal Prongs (Nasal Cannula)
Delivers O2 at 2 - 3 ml/ min.
: Flow rate, maximum : 6l/min (~ 45% FiO2)
Hudson’s Simple O2 mask
Delivers O2 at a rate of 5 - 6 l/min.
: Maximum : 10l/min (~61% FiO2)

O2 mask with Reservoir Bag


Can deliver O2 at a rate of 6 - 8 l/min.
: Maximum : 15l/min (~80% - 90% FiO2
possible because of reservoir bag.

Venturi Mask

It deliver a fixed flow of O2 which can be adjusted by chaging the mask values.
: Maximum flow rate : 15l/min (~60% FiO2)
125. An obese ASA-2 patient was undergoing lap cholecystectomy. It was noticed that his ETCO2
suddenly dropped to 8mm of Hg, spO2 became 90%, BP dropped to 80/50mm of Hg and peak airway
pressure was 18mm of Hg. What could be the possible cause?
(i) CO2 embolism
(ii) Severe bronchospasm
(iii) Endobronchial intubation
(iv) Pneumothorax

(A) i, iv
(B) ii, iii
(C) i, ii
(D) i only
The given clinical history gives evidence of hypoxemia, hypotension, sudden drop in EtCO2 with
normal peal airway pressure which is suggestive of CO2 embolism.

Other options

Bronchospasm
÷
In bronchospasm, there is increase in peak airway pressure due to increased
resistance in airways.
÷

ETCO2 remians normal.


÷

Patient can have hypoxmia and hypotension.

Endobronchial Intubation
÷

Here there is a transient fall in ETCO2 following which there is increased ETCO2 value
because of ventilation - perfusion mismatch.
÷
There can be hypoxemia.
÷

Peak airway pressure and BP remians normal.

Pneumothorax
÷
There is decrease in BP and ETCO2.
÷

Hypoxemia is also present but peak airway pressure is raised (~45.55 mm Hg).
Dermatology
126. A child presents with asymptomatic lesions on dorsum of hand and penile shaft as shown below.
What could be the possible diagnosis?
(A) Lichen planus
(B) Scabies
(C) Scrofuloderma
(D) Lichen nitidus
The given image shows lesion suggestive of lichen nitidus.
Lichen Nitidus
I

It is a chronic inflammatory disease of unknown etiology.


Seen in children and young adults.
: Presents as skin coloured shiny papules on the forehead, dorsum of forearms or shaft of penis.
Kobner’s Phenomenon is present (similar lesions at site of trauma).
: Histopathological examination shows characteristic ‘claw clutching the ball’ appearance.
Other Options

Lichen Planus

! I
Chronic Immune mediated inflammatory skin disease.
i. Characterised by purplish, pruritic, polygonal, plain topped papules and plaques
along the linear aspect.

¥1

Scabies
Itchy, contagius, ectoparasitic skin infection caused by Sarcoptes scabiei (itch mite).
Excoriated papules, burrows (linear/wavy tunnel in skin) are seen in web spaces.

Scrofuloderma
It is the cutaneous manifestation of TB disease characterised by multiple draining
a

sinuses/scars commonly on the neck.


127. A 68yr old patient presented with itchy tense skin blisters on the extensor aspect of limbs as
shown below. What could be the diagnosis?
A) Pemphigus vulgaris
(B) Dermatitis herpetiformis
(C) Bullous pemphigoid
(D) Pemphigus foliaceous

The given image shows tense bullae on urticarial red base which are most likely due
to bullous pemphigoid.
Bullous Pemphigoid
It is an immunobullous disorder where antibodies are directed against BPAG - 2 and
: BPAG - 1 present in hemidesmosomes.
There is sub - epidermal split in the skin resulting in formation of tense bullae which
have red, urticarial bases.
Clinically, Nikolsky sign in negative and bulla spread sign reveal round regular

: bordered bullae.
Lab Profile
Tzanck smear shows eosinophils; Acantholytic cells are absent.
Histopathology shows sub - epidermal bullae with eosinophils.
: Direct immunofluorescence study shows a linear pattern of IgG, C3 deposition in
the basement membrane zone.

Rx : Steroids, Azathioprine
Other Options
Pemphigus Vulgaris
Immunobullous disorder characterised by antibodies directed against DSG - 3 and DSG - 1 of
:desmosomes resulting in acantholysis.
There is an intra - epidermal split resulting in formation of flaccid bullae that rupture
spontaneously to form erosions.
Clinically, Nikolsky sign positive and bulla spread sign shows irregular, angulated margin.
: Lab Profile
Tzanck smear shows acantholytic cells.
: Histology shows a suprabasal split with intact stratum basale : Tombstone Appearance.
Immunoflorescence studies : Shows fish net appearance due to deposition of IgG, C3

:
intraepidermally on the desmosomes.

Pemphigus Foliaceous
Milder form of P.vulgaris characterised by subcorneal split and absence of oral lesions.
Dermatitis Herpetiformis
Chronic sub - epidermal immuno bullous disease associated with intense pruritis.
: IgA antibodies are produced which are directed against epidermal transglutaminase resulting
in sub - epidermal split.
It is associated with gluten sensitive enteropathy.
: Clinically, intense pruritic papulo vesicles and grouped excoriation marks are present along
the extensor aspect.
Lab Diagnosis
: No acantholytic cells on Tzanck smear.
Histology shows subepidermal split with papillary tip microabscesses.
: Immunofluorescence study shows granular pattern of IgA deposition in the basement
membrane zone.

128. Which of the following drugs is not used in the treatment of the following condition?
(A) Tazarotene
(B) Retinoic acid
(C) Benzoyl peroxide
(D) Fluocinolone

Fluocinolone is a steroid and is not used in the treatment of acne as steroids can exacerbate acne.
Treatment of Acne
It is based on the grade /severity of disease.
: Grade I : Formation of comedonal acne.
Rx : Topical Retinoids (Tretinoin, Adapelene, Tazarotene)
Grade II : Formation of Papules

: Rx : Topical Retinoids + Antibacterials (Clindamycin, Nadifloxacin)


Grade III : Conversion of papules to pustules
Rx : Topical Retinoids + Oral Antibiotics (Doxycycline, Azithromycin, Minocycline)
+/-
Benzoyl Peroxide
Grade IV : Nodulocystic Acne

:
Rx : Oral Retinoids - Isotretinoin (highly teratogenic, C/I in pregnancy)

129. A patient presented with vesicular lesions over the chest confined to a single location on one side,
he also complains of pain in the same area. What is the diagnosis?
(A) Herpes zoster
(B) HIV
(C) Herpes simplex
(D) Lymphangioma

Herpes Zoster (Shingles)


It arises due to the reactivation of Varicella Zoster virus infection within the
dorsal root ganglion.
It is characterised by a painful, unilateral vesicular eruption, usually in a
: dermatomal distribution.
It is seen people with poor immune function. Eg : Diabetes, Old Age.
Patients can develop a condition called post - herpetic neuralgia which causes
: allodynia ( non - painful stimuli perceived as painful).
Treatment : Amitryptilline, Pregabalin, Gabapentin.
130. A patient from Bihar with history of prolonged fever during childhood presented with nodular,
hypo pigmented lesions in the face and neck. There was no hypoaesthesia or nerve thickening.
Diagnosis?
(A) Tuberculoid leprosy
(B) PKDL
(C) TB
(D) Atopic dermatitis

The given history (h/o prolonged fever in childhood, living in Bihar) and clinical findings are
suggestive of post - Kala Azar dermal leishmaniasis.

Leishmaniasis
r

It is transmitted by sandfly (Phlebotomus Argentipes).


r

It is endemic in Bihar, West Bengal and UP.


r

3 types : Cutaneous, Mucocutaneous, Visceral


Cutaneous
Also known as oriental sore, Baghdad boil.
: Caused by L. tropica comples ( L. tropica, L. major, L. mexicana, L. aethiopica)
Presents as painless cutaneous ulcer with red base (hypersensitive response).

:
Mucocutaneous
Also known as espundia (not reported in India).
Caused by Leishmania Brasiliensis.
: Characterised by nodular ulcerative lesions in nasal and oropharyngeal mucosa.

Post - Kala Azar Dermal Leishmaniasis (PKDL)


It is a complication of Kala Azar folowing complete/incomplete/irregular
:
treatment of Kala Azar.
Presents as macular/papular/nodular skin rash.
Visceral
Caused by L. donovani, L. infantum, L. chagasii)
: Patients presents with prolonged fever, weight loss, pronounced hepatosplenomegaly,
pancytopenia and hypergammaglobulinemia.
Death occurs in 2 years due to 2° bacterial infection if not treated properly.
: Diagnosis is by biopsy [Splenic aspirate (95% sensitive) > Bone marrow aspirate] and
demonstration of LD bodies in macrophages.

Donovan Bodies
Clusters of blue or black staining, bipolar
chromatin condensations in large
mononuclear cells in granulation tissue
infected with Klebsiella
(Calymmatobacterium granulomatis).

Most sensitive method is NAAT.


: Most specific method is rk - 39 antigen ELISA test.
Rx : Amphotericin - B, Oral Miltefosine, Antimony Compunds

:
131. Identify the correct statement about leprosy.
(A) 50% of cases resolve spontaneously
(B) Spontaneous remission is more common in children than adults
(C) Indeterminate leprosy resolves more spontaneously than tuberculoid leprosy
(D) Leprosy heals by itself in most of the patients within 1 year
Leprosy (Hansen’s Disease)
It is a chronic infectious granulomatous disease caused by Mycobacterium leprae.
It mainly affects the skin and nerves (Schwann cells).
: Course of the disease depends on the cell mediated immunity of the host.
Good CMI : No disease
: Average CMI : Intermediate leprosy

Progress
Self Heal Persist
(95%) Determinate
Leprosy
Indeterminate Leprosy
C
Indeterminate leprosy is usually seen in children presenting from endemic
regions (UP, TN, Bihar) whose CMI is not fully developed.
They develop ill defined hypopigmented macules with no nerve involvement.
: 95% of cases resolve spontaneously.

Determinate Leprosy

Classified into 3 based on CMI (Ridley - Jopling Classification)

Fair CMI
Good CMI Poor CMI

Borderline
Limited Disease Disease Disseminated Disease

Tuberculoid Type (TT) Borderline Type Lepromatous Type (LL)

Borderline Borderline Borderline


Tuberculoid (BB) Lepromatous
(BT) (BL)

In the given question, option C is the most correct.


Option A : 95% of cases resolve spontaneously
Option B : Children recover better from leprosy than adults.
Option C : Indeterminate leprosy resolve better than tuberculoid
Option D : Leprosy doesn’t heal by itself in multibacillary type.
Even though option B is correct, Option C is more acurate and factual.
132. A rose gardener developed the following skin changes after a prick injury during work. What
could be the causative agent?
(A) Sporothrix schenkii
(B) Histoplasma capsulatum
(C) Sarcoptes scabei
(D) Blastomyces dermatitidis
The given image is showing nodulo - ulcerative lesions along the lymphatics suggestive of
sporotrichosis.
Sporotrichosis
Also known as Rose Gardener’s disease.
: Caused by Sporothrix Schenkii.
It is endemic to the Himalayan belt.
: Infection arises following traumatic implantation of the fungus (rose prick, injury with
plant material, etc)
Patient develop nodulo - ulcerative lesions along the lymphatics within weeks to months.
: Diagnosis is made by biopsy.
H & E shows cigar shaped yeast arranged in structures called asteroid bodies.
: Culture reveals hyaline septate molds at 25°C - 28°C, dimorphism can be proved by yeast
conversion test.
I

Rx : Itraconazole (continued 2 - 4 weeks after resolution of lesions).


Psychiatry
133. Which of the following statements is true about Korsakoff syndrome?
(A) Triad of acute onset confusion, ophthalmoplegia, ataxia
(B) Loss of identity, memory & intact personality
(C) Impairment of implicit memory & immediate recall
(D) Anterograde amnesia with loss of recent memory
Korsakoff psychosis or Korsakoff syndrome is a long term/chronic manifestation of alcohol toxicity.

Alcohol toxicity

Wernicke’s Encephalopathy
It is an acute manifestation.
: It occurs due to deficiency of Vitamin B1 (Thiamine).
It has a characteristic triad :
: Global confusion
Ophthalmoplegia
: Ataxia
Peripheral Neuropathy can also be present.
: It is due to the involvement of mamillary bodies and peri - aqueductal grey area.
Rx : Thiamine inj 100 mg iv/im for 3 days followed by oral supplementation.

:
Korsakoff Psychosis
It is a chronic manifestation.
Characterised by anterograde amnesia and filling up holes in recent memory with made up facts
: (confabulation).
It is due to the involvement of mamillary bodies and anterior thalamic nuclei.
Prognosis is bad compared to Wernicke’s Encephalopathy.
: Management include Thiamine supplementation and rehabilitation.

134. Which of the following is a true statement regarding OCD?


(A) Depression is a common comorbidity in OCD
(B) Contamination is an uncommon obsession
(C) Atypical antipsychotics are the first line treatment
(D) Prevalence of OCD in general public is 7-10%
Obsessive Compulsive Disorders (OCD)
s
OCD is a thought content disorder in which a person has intrusive thoughts (obsessions) and/or
feels the need to perform certain routines repeatedly (compulsions) to an extend where it
induces distress or impairs one’s general functioning.
Prevalence of OCD in general public is 2% - 3%.
: It is seen in adolescents and young adults mostly in 3rd decade of life.
It is commonly associated with depression, anxiety and distress.
: Patient loses an hour or more in a day due to this condition.
The most common type of obsession is “fear of contamination”.
: Management
Pharmacological treatment

:
- SSRIs : First line of treatment
Eg : Fluoxetine, Fluoxamine, Sertraline, Paroxetine
- Tricyclic Antidepressants : Clomipramine
- 2 trials of SSRIs are to be given before starting TCAs.
Non - Pharmacologic Treatment : Exposure Response Prevention Therapy (ERP)
Radiology
135. Which of the following procedures would be contraindicated in a patient with the following
finding on imaging?
(A) NG tube
(B) Log roll
(C) ICD tube drainage
(D) Epidural anaesthesia

1-

The given image shows herniation of stomach into the left lung cavity following
diaphragmatic injury.
ICD insertion is not required in this case and is contraindicated because it can injure
: the herniated part of the stomach.
Rest of the given options can be done and are not contraindicated.
136. A 30yr old patient was hit by a bull and later presented with complaints of abdominal pain for
the past 3 days and obstipation along with nausea & vomiting for the last 2 days. He was initially
stabilised but now complains of increased abdominal pain and abdomen was found to be rigid on
examination. A radiograph was performed and is given below. What could be the most likely cause
of his condition?
(A) Hollow viscous perforation
(B) Sub diaphragmatic abscess
(C) Gastric volvulus
(D) Splenic rupture

The given clinical history (h/o trauma to abdomen. Abdominal pain, rigidity) and the radiograph
showing air under diaphragm is suggestive of hollow viscous perforation.
Radiological features of Bowel perforation (Pneumoperitoneum)
Free gas under diaphragm
: Rigler sign (double wall sign)
Gas is outlining both sides of the bowel wall, i.e., gas within the body’s lumen and gas within the
: peritoneal cavity.
It is seen with large amounts of pneumoperitoneum (> 1000 ml).
Telltale Triangle sign
: Football sign
Falciform ligament sign
: Inverted “V” sign or lateral umbilical ligament sign.
Cupola sign
: Urachus sign

Football Sign Telltale Triangle Sign Falciform Ligament Sign

Inverted “V” sign Cupola Sign Urachus Sign


Rigler Sign

137. Identify the investigation based on the following image


(A) USG
(B) Fluoroscopy
(C) MRI
(D) Contrast study

The given image shows the ultrasonogram view of thorax region.

138. Which of the following statements about CORADS is true?


(A) Pleural thickening is a typical finding
(B) CORADS 0 - Normal lung
(C) CORADS 5 - Definitive COVID
(D) Lobar consolidation is a typical finding
CORADS is a CT based system that is sued to assess the suspicion of pulmonary involvement in
COVID -19
COVID - 19 Reporting and Data Systems
CT findings
CO-RADS 1 No Normal or non-infectious abnormalities
CO-RADS 2 LOW Abnormalities consistent with
infections other than COVID-19
CO-RADS 3 Indeterminate Unclear whether COVID-19 is present
CO-RADS 4 High Abnormalities suspicious for COVID-19
CO-RADS 5 Very high Typical COVID-19
CO-RADS 6 PCR +
Typical findings seen in COVID-19
Bilateral, peripheral & lower zone predominant airspace opacities (GGOs,

:
consolidations) on X - ray.
Bilateral, sub pleural, peripheral GGOs, airspace consolidations, crazy pavement
patterns on CT.
Reverse halo/Atoll sign: central area of GGO with surrounding consolidation.

G@••*
Reverse Halo Sign
(Atoll Sign)

139. Double bleb sign on USG is due to


(A) Amnion & Chorion
(B) Amnion & Yolk sac
(C) Primary & Secondary Yolk sac
(D) Chorion & Yolk sac

The double bleb sign is an important USG

i
finding confirming pregnancy.
The double bubbles are formed by the
amnion and the yolk sac.
It is seen at 5-6 weeks of IUL.
Important Landmarks in Early Obstetric USG
Intradecidual Sac Sign : 4 - 5 weeks of gestation.
: Double decidual sac sign : 4 - 5 week of gestation.
Double belb sign : 5 - 6 weeks of gestation.
: Foetal pole and Cardiac Activity : 6 weeks of gestation.

(1) (2)

(4)

140. Which of the following modalities are used for the diagnosis of placenta accreta?
(i) USG
(ii) MRI
(iii) CT scan
(iv) Angiography

(A) i only
(B) i & ii
(C) i & iii
(D) i, ii, iii, iv
Placenta accreta is a type of morbidly adherent placenta.
Morbidly Adherent Placenta
<

It arises due to the abnormal invasion of blastocyst during impantation resulting in a placenta
that is adherent to the myometrium.
<

This occurs because of the absence of Decidua basalis and Nitabuch’s layer, a layer of fibrinoid
degeneration which is present between placenta and decidua that limits the penetration of
blastocyst.
Types
Placenta accreta : Superficially attatched to myometrium.
:
Placenta Increta : Invades deep into myometrium.
Placenta percreta : Penetrates myometrium to reach serosa.

:
Risk Factors
Placenta previa in previous pregnancy.
Previous history of Caesarian section.

:
Diagnosis
USG (TVS)
Demonstrates absence of subplacental sonolucent area, heterogenous appearance of

: placenta and presence of placental lakes.


MRI is the gold standard.

Management : Elective CS + Hysterectomy

141. A primigravida at 10 weeks of gestation presented to the OPD with complaints of fresh
bleeding per vaginum. On examination her uterus corresponded to 12 weeks size and her USG
showed the following picture. What could be the possible diagnosis?
(A) Missed abortion
(B) Hydatidiform mole
(C) Endometriosis
(D) Threatened abortion
The given image of a USG shows snowstorm appearance of uterus which is seen in complete
hydatidiform mole.
Hydatidiform Mole
It is a gestational trophoblastic disease that arises due to undue proliferation of
: trophoblasts and hydropic degeneration of chorionic villi.
Patients provide h/o passing grape like vesicles per vaginum and h/o bleeding per vaginum.
USG examination shows a snow storm appearance.
: It is of two types : Complete mole, Partial Mole.
Partial Mole Complete Mole

Karyotype Karyotype
Triploid, Dispermic Diploid, Monospermic (90%)
(69 XXX or 69 XXY) (46 + XY)

Extra genetic material is paternal in origin. Duplication of genetic material of sperms,


hence entire genetic material is paternal
(Androgenesis).

Hydropic degeneration is less. Hydropic degeneration is more marked.

Foetal parts may be present. No foetal parts are present.


5
5
High levels of HCG but < 10 IU/l. Levels of HCG > 10 IU/l.

Theca lutein cysts absent. Theca lutein cysts present.

Immunostaining with p57, Kip -2 positive. P57, Kip - 2 negative.

Management : Suction evacuation or Hysterectomy followed by β - Hcg level estimation.

142. A patient presented with features of jaundice, generalised itching and clay coloured
stools. His ultra sonogram showed a distended gall bladder without evidence of any stones,
his CBD was also dilated. Which of the following is the next best investigation?
(A) MRI
(B) Endoscopic ultrasound
(C) PET scan
(D) Percutaneous transhepatic cholangiography
The given clinical history (jaundice, generalised itching, clay coloured stools) are suggestive
: of obstructive jaundice.
His USG shows evidence of a distended gall bladder and dilated CBD which implies that the
obstruction must be distal to the CBD and most likely at the periampullary region.
Obstruction at the periampullary region can arise due to a periampullary carcinoma,
: obstruction by a dislodged gall stone, sphincter of Oddi dysfuntion, etc.
The best imaging modality to check for obstruction at peri - ampullary region is endoscopic
ultrasound as it offers the benefit of taking biopsy in case of peri - ampullary carcinoma.
Endoscopic Ultrasound
/

It is a medical procedure in which endoscopy is combined with ultrasound to obtain


images of the internal organs.
It also allows characterisation and biopsy of any focal lesions.
: It is highly sensitive for detection of pancreatic cancer, particularly in patients who are
suspected to have a mass or present with jaundice.
Medicine
143. What is the cause of c wave in JVP?
(A) Atrial systole
(B) Isovolumetric ventricular contraction
(C) Atrial diastole
(D) Passive filling of ventricles
JVP (Jugular Venous Pulse)
-

It is the indirectly observed pulse pressure over the venous system via visualisation of the
internal jugular vein.
-

It is useful in the differentiation of different forms of heart and lung disease.

JVP waveform
/
It is biphasic.
Classically it has three upward deflections and two downward deflections.
: a wave : Right atrial contraction.
c wave : Isovolumetric contraction of right ventricle.
: x descent : Pulling of tricuspid valve downward during ventricular ejection.
v wave : Venous filling when tricuspid valve is closed causing rise in venous pressure.
: y descent : Rapid emptying of the atrium into the ventricle following the opening of
tricuspid valve.

Pathological JVP waveform


Large a wave : Pulmonary hypertension, pumonary stenosis.
: Absent a wave : Atrial fibrillation.
Canon a wave : Complete heart block, Ventricular tachycardia.
: Large v wave : Tricuspid regurgiation.
Steep x, Absent y : Cardiac tamponade.
: Steep x, Steep y : Constrictive pericarditis.

144. Drug that can be used in an acute attack of cluster headache?


(A) oral Sumatriptan
(B) sc Sumatriptan
(C) O2 6l/min
(D) O2 8l/min
Cluster Headache
It is a neurologic disorder characterised by recurrent severe headaches on one side of the head,
: typically around the eye.
It is often accompanied by eye watering, nasal congestion or swelling around the eye on the
affected side.
It often lasts for 15 minutes to 3 hours, attacks occur in clusters which typically lasts for weeks
to months and occasionally more than a year.
Risk Factors
Tobacco, Alcohol
Family history
Exposure to alcohol, nitroglycerin and histamine may trigger attacks.
Diagnosis is based on symptoms and history.

Treatment
Acute attacks
100% O2 at 10l/min for 10 - 15 minutes.
: Subcutaneous Sumatriptan.
1
Prophylaxis
Short term : Steroids
: Long term : Verapamil, Lithium

145. An old aged patient with severe MS with atrial fibrillation suffered multiple episodes of TIA.
Which of the following statements are correct regarding prevention of stroke in this patient?
(i) Only aspirin is given in prevention of stroke
(ii) Warfarin is used in prevention of stroke
(iii) Dabigatran is not indicated
(iv) Mitral valve surgery is indicated in this case

(A) i & ii
(B) ii & iii
(C) ii, iii & iv
(D) all are correct
Prevention of Stroke

According to ACC/AHA, guidelines for management of valvular heart disease.


I

Aspirin is an anti - platelet drug and is not recommended in patients with AF


because the reason for clot formation is stasis of blood and activation of
coagulation cascade which is not inhibited by aspirin.
Warfarin is a Vit K antagonist and is recommended in this case.
: There are no adequate studies to support the use of newer oral anticoagulants
(NOACs) like Dabigatran in patients with Rheumatic MS with AF or those with
mechanical heart valves with AF.
I

Mitral valve surgery is the definitive treatment for dealing with severe valvular
heart disease.
146. Based on the given serological test results identify the type/stage of hepatitis infection
in a patient:
HBsAg - negative
HBcAg - negative
HBeAg - negative
HBV DNA - negative
IgM Anti-HBcAg - negative
IgG Anti-HBcAg - positive

(A) Pre core mutant variety


(B) Chronic hepatitis B
(C) Recovered from hepatitis B
(D) Window period
Hepatitis B infection
Caused by Hepatitis B virus which is a partially double stranded DNA virus belonging to the
Hepadnaviridae family.
Route of Transmission
Sexual Transmission (Most Common)

Blood Transfusion
Vertical Transmission : Mother to baby

Accidental Needle - Stick injury


Serum Markers

HBsAg
Surface Antigen

Earliest serological evidence of Hepatitis B infection, first to appear.



HBcAg
Core Antigen, never appears in blood.

Only antibody to it is seen in blood.


HBeAg
Marker for replication/infectivity of virus.

HBxAg
CD95 inhibitor.

Inhibits extrinsic pathway of apoptosis.


Triggers development of HCL.


PCR HBV DNA : Quantify viral load.

Antibodies against Hepatitis B


-
Anti - HBsAg : Protective Antibody
-
Anti - HBcAg : IgM in a/c viral hepatitis, IgG in c/c viral hepatitis.
-
Anti - HBeAg : Reduces replication rate and infectivity.

Chronology of serum markers and antibodies


Stage of disease according to serum markers and antibodies
HBsAg Negative Susceptible
anti-HBc Negative
anti-HBS Negative

HBsAg Negative Immune due to natural infection


anti-HBc Positive
anti-HBs Positive

HBsAg Negative Immune due to hepatitis B vaccination


anti-HBc Negative
anti-HBs Positive

HBsAg Positive Acutely infected


anti-HBc Positive
IgM anti-HBc Positive
anti-HBs Negative

HBsAg Positive Chronically infected


anti-HBc (IgG) Positive
IgM anti-HBc Negative
anti-HBs Negative

HBsAg Negative Interpretation unclear; four possibilities:


anti-HBc (IgG) Positive 1. Resolved infection (most common)
anti-HBs Negative 2. False-positive anti-HBc, thus
susceptible
3. "Low level" chronic infection
4. Resolving acute infection

147. A 45yr old chronic smoker presents with shortness of breath, bilateral pedal edema and right
upper abdominal pain. On examination, his JVP was found to be elevated and liver is palpable 8cm
below right costal margin. Which of the following heart chambers is under functioning?
(A) Right ventricle
(B) Left ventricle
(C) Right atrium
(D) Left atrium
The given signs and symptoms (pedal edema, tender hepatomegaly, raised JVP) is suggestive of
right sided heart failure which is due to the inability of right ventricle to pump blood efficiently.

Signs of heart failure


Left Ventricular Failure Right Ventricular Failure

Decrease in Systolic BP. Raised JVP.


End organ failure due to ischemia. Kussmaul sign (Rise in JVP on inspiration)
Eg : Oliguria Tender hepatomegaly.
B/L fine crepitations. B/L pitting edema.
B/L reduced air entry. Abdomino - Jugular reflux present.

Diagnostic Criteria used : Framingham Criteria


J
Major criteria Minor criteria
J

Acute pulmonary edema


J
Ankle edema
)

Cardiomegaly
)
Dyspnea on exertion
)

Hepatojugular reflex
)
Hepatomegaly
)

Neck vein distension


)
Nocturnal cough
)

Paroxysmal nocturnal dyspnea


)
Pleural effusion
or orthopnea
: Tachycardia (> 120 beats per minute)
Rales
: Third heart sound gallop
Heart failure is diagnosed when two major criteria or one major and two minor

: criteria are met.


Right sided heart failure is often caused by pulmonary heart disease (Cor Pulmonale) which
occurs due to :
COPD
: Pulmonary Hypertension
Interstitial Lung Disease
: ARDS
Obstructive Sleep Apnea
: Sarcoidosis, Cystic Fibrosis
]
Management : O2 therpy, Antibiotics, Expectorants, Diuretics, Anticoagulants, Vasodilators
148. In a patient with grade II pulmonary artery hypertension whose vaso-reactivity test was
negative. Which of the following will be a preferred management of this patient?
(A) Continuous infusion of Epoprostenol
(B) Iloprost 100mg orally
(C) Intravenous Alprostadil
(D) Oral Ambrisentan

Pulmonary Hypertension
Types
Class I : Primary Pulmonary Artery Hypertension
: Class II : Pulmonary Venous Hypertension
Class III : Pulmonary Capillary Hypertension
: Class IV : Chronic Pulmonary Thromboembolic Hypertension
Class V : Miscellaneous (Extrinsic Compression)

:
Class I (Primary Pulmonary Arterial Hypertension)
Defined as mean pulmonary artery pressure (PAH > 25 mm Hg).
-
Grades of Pulmonary Artery Hypertension
Grade I : PAH > 25 mm Hg
:
Grade II : PAH : 25 mm Hg - 40 mm Hg
Grade III : PAH : 41 mm Hg - 55 mm Hg
: Grade IV : PAH > 55 mm Hg
-
Clinically patient is dyspneic with no signs of heart/lung failure.
-
On examination,
‘a’ wave seen in JVP
: Palpable epigastric impulse
-
Parasternal heave
Loud P2, palpable P2
: TR murmur
-
Ascites, Pedal Edema, Hepatomegaly
X - Ray shows large central pulmonary artery, peripheral rapid attenuation of vessels.
: ECG shows RV strain.
-
PFT : FEV1, FVC normal
FVC
= Decreased
DLCO
Treatment
Based on vaso - reactive test.
^

If positive (inhaled Nitric Oxide cause PAH < 10 mm Hg with BP and CO normal),
^

Ca2+ channel blockers are used.


If negative, endothelin antagonists (Bosentan, Ambrisentan) are preferred.
: Drugs like prostacyclin analogues and PDE - 5 inhibitors (Sildenafil) are also used in
advanced cases.
Oral anticoagulants, O2 therapy, diuretics are given as supportive management.

:
149. Which of the following statements is wrong about heart failure?
(A) Non cardiovascular death is more in HFpEF 30-40% as compared to HFrEF 15%
(B) ACE inhibitors leads to angioedema in 1% cases and dry cough in 10-15% cases
(C) 5yr mortality in heart failure is around 50% regardless of EF or type of heart failure
(D) In elderly atrial fibrillation is common and constitutes quarter of stroke cases.
Option A
Heart Failure is of two types:
Heart Failure with preserved Ejection Fraction (HFpEF)
Here EF > 50% is maintained.
: Majority of deaths in this group occurs due to non - cardiovascular causes.
Heart Failure with reduced Ejection Fraction (HFrEF)
\

Here EF < 40%


: Majority of deaths arise due to cardiovascular causes.

Option B
ACE inhibitors cause cough in 15% - 30% cases and angioedema in 0.1% - 1% cases.
:
This is because they inhibit the degradation of Bradykinin which can cause cough and angioedema.

Option C
Overall 5 year mortality rate in heart failure is around 50% regardless of EF and type of

:
heart failure (HFpEF or HFrEF).

Option D
In elderly patients, AF is responsible for more than 1/3rd of stroke cases.
1

AF can result in formation of thrombus in heart which can embolise and cause ischemic stroke.
150. A patient presents to OP with nausea & headache. On evaluation he was found to have a BP of
180/110 and blood tests showed a K+ value of 2mEq/l. His ABG showed results in favour of metabolic
alkalosis. Which syndrome could be responsible for this condition?
(A) Gitelman syndrome
(B) Liddle syndrome
(C) Bartter syndrome
(D) Gordon syndrome
The give history shows development of metabolic alkalosis, hypokalemia and hypertension which
are suggestive of Liddle Syndrome.
liddle’s syndrome
i

It is due to the overactivity of epithelial Na+ Channel in the collecting tubule ( due to gain of
function mutation ).
It is Autosomal Dominant.
:
This causes :
Increased absorption of Na+ and H2O into circulation causing hypertension.
: Increased excretion of K+ and and H2O into tubule resulting in hypokalemia &
metabolic alkalosis.
Hypertension causes feedback RAAS inhibition resulting in low levels of Renin & Aldosterone.
: Low Renin HTN.

Other Options
gitelman Syndrome
I

Autosomal Recessive.
Due to defect in Na-Cl cotransporter and TRPM6 transporter in DCT.
+ -

This results in Salt wasting & Polyuria.


:
Features :
Antenatal history of Polyhydramnios
:
Dehydration, low urine osmolality
Failure to thrive
:
High Renin, Aldosterone
Hypomagnesemia causing reduced PTH levels which result in hypocalcemia ( TETANY )
:
Hypokalemia
I

Metabolic Alkalosis
Bartter syndrome
I

Autosomal Recessive
Defect in Na-K-2Cl cotransporter, CL channel and ROMK (Range of motion K+ channel) in
+ + -

loop of Henle.
Defect in these channels lead to Ca wasting.
: Features include
Salt wasting
: Polyuria
Dehydration
: Polyhydramnios
I

Sensorineural Deafness ( Na - K - 2Cl channel present in the ear )


t t -

High Renin, Aldosterone


:
Hypocalcemia, Metabolic Alkalosis
Hypocalcemia ( Calcium Wasting ) - Renal Rickets
: Nephrocalcinosis ( Renal Stones )
I

Treatment
1

Indomethacin is the D.O.C


Supplement Na , Ca
1- 2T
1

Gordon syndrome
Autosomal dominant disorder
:
Features
Cleft Palate
: Congenital Contractures of hands & feet
Short stature
:
Kyphoscoliosis
-
.

Hip dislocation

151. A patient following head trauma was found to have increased secretion of anti-diuretic hormone.
Which of the following findings is not present in this case ?
(A) Decreased Plasma Osmolality
(B) Decreased Urinary Na+
(C) Increased Urine Osmolality
(D) Decreased Plasma Na+

The given history is suggestive of SIADH.


SIADH
Characterised by excessive unsuppressible release of ADH either from Posterior Pituitary Gland

:or an abnormal non-pituitary source.


Causes of siadh
CNS Causes
Infections ( Meningitis, Encephalitis, Brain Abscess )
: Mass/ Bleed ( SAH, SDH, Trauma, ICSOL )
Hydrocephalus
:
GBS
Multiple Sclerosis
I

CANCERS
Lung Cancers ( Small Lung Cancer, Mesothelioma )
1

Lymphoma
:
Gastro- Intestinal Cancers
I

PULMONARY Cause
Infection ( Pneumonia, Lung Abscess )
1

Asthma
: Cystic Fibrosis

Diagnosis
Made by Lab Parameters

:
i. Decreased serum osmolarity ( < 275 mOsm/Kg )
ii. Increased Urine Osmolarity ( > 100 mOsm/Kg )
iii. Urinary Sodium > 40 mmol/L with normal dietary intake
iv. Normal thyroid & Adrenal function
v. Normal renal function, No hydrocephalus, No Acid Base Disorders
vi. Clinical Euvolemia

Treatment
Manage the underlying cause
Fluid restriction, Salt and Protein supplementation
: Drugs:
VASOPRESSIN - 2 Receptor blockers
: TOLVAPTAN
-
DEMECLOCYCLINE
152. Which of the following statements regarding EEG is true ?
(A) Scalp EEG is diagnostic of frontal lobe epilepsy
(B) 10% of healthy people have epileptiform waves on EEG
(C) EEG is mandatory for diagnosis of epilepsy
(D) In progressive multi-focal Leucopenia encephalopathy generalised epileptiform waves are seen
Option (A) - Scalp EEG is used in the diagnosis of frontal lobe epilepsy, it is not diagnostic for
frontal lobe epilepsy, it is not diagnostic for frontal lobe epilepsy.
Option (B) - 5-10% of healthy people can have epileptiform waves on EEG.
Option (C) - EEG is not mandatory for diagnosis of epilepsy.
Option ( D ) - In progressive multi-focal leucoencephalopathy, diffuse slow waves are seen on EEG.
Electro encephalograph
e

It is a method to record the electrical activity of the brain by attaching electrodes to the scalp.
e

It was first done by Hans Berger.


e

It is used in diagnosis of epilepsy, sleep disorders, brain death etc.


e
It has 5 distinct waveforms.
Gamma
Has highest frequency ( 60/s ).
: Seen during focused attention.
Beta
13 - 30/ sec frequency.
: Seen during attention & wakeful state.
Arise from frontal cortex.
:
Alpha
8 - 12/sec frequency.
Seen during released state ( eyes closed ).
: Produced from parietal cortex.

Theta
4 - 7/ sec frequency.
: Seen during memory conversion ( Short term to long term ) at hippocampus.

Delta
1-3 / sec frequency.
: Slowest wave seen during NREM sleep.
ELectrode Placement in EEG

Normal EEG
Abnormal eeg
3 Hz spikes & Slow wave : Absence seizure
: < 2.5 Hz spikes & slow waves : LENNOX GASTAUT SYNDROME
4-6 Hz Polyspike wave : Juvenile Myoclonic Epilepsy
: Periodic Sharp wave complex : Variant Creutzfeldt - Jakob Disease
Triphasic Waves : Hepatic Encephalopathy

:
153.Which of the following drugs is the only medication effective in preventing disability
progression in a patient with primary progressive multiple sclerosis ?
(A) NATALIZUMAB
(B) OCRELIZUMAB
(C) SIPONIMOD
(D) RITUXIMAB
Multiple sclerosis
It is an autoimmune demyelinating disorder.
Main target in this disease is Myelin Basic Protein ( MBP ).
: Commonly seen in females than males.
Pathophysiology
T cells attack blood vessel endothelium increasing BBB permeability, they cause direct

:damage to myelin by producing various cytokines like TNF-Alpha which damage myelin basic
protein resulting in axonal degeneration.
B cells produce antibody against myelin oligodendrocyte glycoprotein.
Repair occurs following damage in the form of gliosis resulting in formation of

:
periventricular plaques.

Clinical features
Sensory Symptoms
Optic Neuritis
: Motor Symptoms

Sensory Symptoms
Paraesthesia
: Pins and Needle sensation
Pain
:
UHTHOFF PHENOMENON: Neurological symptoms triggered on hot shower
OPTIC NEURITIS
Decreased visual acuity
: Color blindness
Internuclear Ophthalmoplegia
: Periorbital pain that increase on movement of eyeball

motor symptoms
Weakness that increase on activity
: UMN Lesion
LMN Lesion
: Ataxia, Vertigo
Facial Myokymia : Flickering movements of muscle of face
: Detrusor Hyper- Reflexia
Lhermitte’s sign : Neck flexion causes shooting pain in legs

:
Types
Relapsing Multiple Sclerosis
Secondary Progressive
: Primary progressive
Most Severe
: Least Common

Diagnosis
Done by Revised McDonald’s Criteria
: 2 neurologic symptoms lasting > 24 hrs, separated by 4 weeks or more
Work up
: GADOLINIUM ENHANCED MRI
Shows Periventricular plaques ( Dawson’s Finger )
: Investigation of Choice
~

CSF STUDY
Shows mild to moderate pleocytosis and slight increase in proteins
: Electrophoresis shows Oligoclonal band in CSF
Treatment
Acute Attack - I.V METHYL PREDNISOLONE
: Disease modifying drugs
ß - Interferon
: Oral FINGOLIMOD
Oral DIMETHYL FUMARATE
: NATALIZUMAB (Inhibit T Lymphocyte binding to BBB capillaries)
~

OCRELIZUMAB : Used in PPMS management { acts by attacking mature B calls }

154. Identify the structure affected in the most common subtypes of GBS ?
(A) a
(B) d a

(C) b
b
(D) c

:
Guillain barre syndrome
It is a rapid onset muscle weakness caused by immune system damaging the
: peripheral nervous system
It is an Autoimmune disorder where the body mistakenly attacks Myelin Sheath
{ MOLECULAR MIMICRY }
Males are more commonly affected
: Triggers
Campylobacter Jejuni ( MC ) : H/O Acute Dysentry
: CMV
EBV
:
Mycopalsma
Hep A, B
:
HIV
[

Zika Virus
Types
Acute inflammatory demyelinating polyneuropathy ( AIDP ) : Most Common
: Acute motor axonal neuropathy ( AMAN )
Acute motor sensory axonal neuropathy ( AMSAN )
: Miller Fischer Syndrome
Clinical features
Flaccid Paraplegia { Symmetrical }
: Truncated Paralysis { Starts from legs - Face: Ascending Paralysis
Quadriplegia
: Respiratory muscle paralysis { Cause of death }
Neck Floppiness
:
Facial Nerve Palsy
Minimal Sensory Involvement
: Distal Areflexia { Earliest Manifestation }
Bowel & Bladder characteristically spared
: Autonomic involvement: Neurogenic Shock (HR,BP)
Work up
1. Lumbar Puncture : Albuminous - Cytological dissociation
2. Nerve Conduction Velocity : Latency period increased
3. MRI Spine : Demyelination
4. Detection of Anti-bodies
AIDP : Anti - GM1 antibodies
:
AMAN : Anti- Gd1a antibodies
^

MFS : Anti- Gq1b antibodies

Treatment
1. IV Ig
2. Plasmapheresis
Surgery
155. Identify the best method to measure the length of nasogastric tube in Adults
(A) Length from Nose to Xiphoid process to umblicus
(B) Length from Nose to tragus of ear to umblicus
(C) Length from Nose to tragus of ear to xiphoid process
(D) Length from mouth to tragus of ear xiphoid process

Nasogastric tube ( Ryle’s tube ) is used for administration of drugs, oral agents and feeding.
: It can also be used for Nasogastric aspiration.
It is 125cm in length with marks as 45cm, 55cm, 65cm & 75cm.
: The length required is calculated as the distance from nose to tragus of ear to the tip of xiphoid
process.
Technique of insertion
Patient is asked to flex his neck in sitting position.
: The tip of the tube is lubricated with xylophone gel and is directed straight towards the
back of the patient as it moves through the nasal cavity and down into the throat.
When the patient feels gag, they are asked to swallow.
: Once the tube reaches the stomach the tube is fixed to the nose using tape.

156. Most common cause of Bladder Outlet Obstruction in Male child


(A) Anterior Urethral Valve
(B) Posterior Urethral Valve
(C) Ureterocele
(D) Urethral Atresia

The most common cause of Bladder Outlet Obstruction (BOO) in a male child is
posterior urethral valve.
Posterior urethral valve
It is the most severe form of obstructive uropathy.
: Usually affects boys & often presents in the post - natal period.
Young’s classification
-

Type 1
Most Common.
Occurs when two mucosal folds external anteroinferiorly from bottom of
verumontanum and fuse anteriorly at lower level.
-

Type 2
Mucosal folds extend along posterolateral urethral wall from ureteric orifice to
verumontanum.
-

Type 3
Cobb’s Collar.
Circular diaphragm with central opening in membranous urethra.
Located below the verumontanum and occurs due to abnormal canalisation of
urogenital membrane.

Clinical features
Post micturition dribbling.
: Increased frequency of micturition.

Frequent UTI’s.
Hydronephrosis due to urinary pathway obstruction.
:
Uremia.
Diagnosis
r
Antenatal or Postnatal USG : Key Hole Defect
r
Micturating Cystourethrogram

Management
r
Fulguration of Valves ( Definitive ).
Foley’s catheterisation for temporary relief.
: Treat Uremia.

157. Which of the following is not a lower urinary tract symptom?


(A) Incontinence during intercourse
(B) Leaking of urine while coughing
(C) Sudden urge to urinate
(D) Incontinence during sleep
Lower Urinary Tracy Symptom ( LUTS ) are symptoms related to the lower urinary tract : Bladder,
Prostate & Urethra.
LUTS
Voiding Symptoms
Hesitancy
: Poor Stream
Intermittent Flow
: Incomplete emptying
Post Void Dribbling
: Overflow incontinence

Storage Symptoms
Frequency
: Nocturnal
Urgency
: Urge Incontinence

Complex Symptoms
Hematuria
:
Recurrent UTI
Urinary incontinence
:
A/C or C/C urinary retention
158. A patient presents to OP with complaints of hematuria. Urine cytology reveals high
grade transitional cell cancer and CT scan shows a mass in the bladder of size 2x2 cm.
What is the next step in management ?
(A) Radical Cystectomy
(B) Neoadjuvant Chemotherapy
(C) Hemicystectomy with bladder reconstruction
(D) Transurethral resection of Tumor

Bladder cancer
3 types
: Transitional Cell Cancer (Most Common)
Squamous Cell Cancer
: Adenocarcinoma
Clinically presents with gross painless hematuria.
: NAT2 and GSTM1 are consistent germline markers for bladder cancer.
Work up
USG - growth in bladder, Clots
:
Urine Test - Routine & Microscopy
Urine Cytology
: Cystoscope is done to confirm diagnosis
MRI to stage disease

:
Management
For Superficial Bladder Cancer (not invading muscle layer)
Transurethral resection of bladder tumor is done.
Intravesical chemotherapy is done

: THIOTEPA
in case of high grade bladder tumors.

ADRIAMYCIN
: MITOMYCIN -C
]
Intravesical immunotherapy (BCG)
is also done in high grade tumors.
For advanced deep layer bladder cancer {Invading bladder Cancer }
Radical Cystectomy is done followed by radiotherapy.
: In large respectable tumors, neoadjuvant chemotherapy (Systemic) is given ( M-VAC
regimen : METHOTREXATE, VINBLASTINE, DOXORUBICIN, CISPLATIN ) to reduce the size of
tumors following which surgery & radiotherapy is done.

Partial Cystectomy is done in cases where tumor is at the dome of bladder, not involving ureteric
orifice or in case of solitary lesions.
159. Identify the components of Reynolds Pentax
(i) Fever (iv) Malena. (vii) Jaundice
(ii) Hypotension. (v) Vomiting
(iii) Altered Mental Status (vi) Pain
(A) i,ii,iii,vi,vii
(B) i,iii,v,vi,vii
(C) ii,iii,iv,v,vi
(D) iii, iv, v, vi, vii
Reynold’s Pentax
Reynold’s Pentax is a collection of signs and symptoms involved in Ascending Cholangitis
: ( Inflammation of Biliary Tree ).
It includes :
Intermittent fever
:
Intermittent Pain
Intermittent Jaundice
:
Septic Shock
Altered Mental Status

:
160. Which of the following statements are correct regarding HCC ?
(i) Global incidence of HCC is rising
(ii) NASH & NAFLD are risk factors
(iii) LENVATINIB is used in tumors of size < 3.5cm
(iv) TACE is done for multi- modular tumor
(A) i,ii,iv
(B) i,ii,iii,iv
(C) ii,iii
(D) i,ii
Hepatocellular cancer
It is the most common primary malignant tumor of liver.
/

Global incidence of HCC is rising.


Risk factors
Hep B, Hep C.
: D.M, Obesity, Alcohol.
Liver Cirrhosis, NASH, NAFLD.
: Hemochromatosis, Wilson Disease, Alpha-1 antitrypsin deficiency.
THOROTRAST exposure.

:
Clinical features
Hepatosplenomegaly {MC & Earlier Feature}
Abdominal Pain
: Jaundice (Late Sign)

Work up
Triple Phase CT ( IOC )
: USG Abdomen
Staging is done using PET-CT
: Tumor markers
Alpha -Feroprotein (MC)
: PIVKA-11
Glycipan
: Heppar -1
Neurotensin B

:
Management
Localised disease
Small tumors are resected by maintaining functional liver reserve (>25%)
: In case of low functional liver reserve, liver transplant is done
FLR can be in increased by Nimura’s technique or ALPP’s procedure ( Associating Liver
: Partition and Portal vein ligation for Staged Hepatectomy ) in some cases
-

Advanced Disease
-

Only palliative management


SUNITINIB, LENVATINIB, SORATENIB
: TACE { Trans arterial chemo embolisation }
TARE { Trans arterial radio embolisation }
:
Radio frequency ablation { Tumor < 3cm }
Microwave ablation
:
High Frequency Ultrasonic Therapy ( HIFU)
-

Most important Prognostic tumor - Stage of Disease


161. Which of the following is true about Colon Cancer ?
(i) Metformin reduces the risk of Colon Cancer
(ii) Right and Left Colon cancers have different clinical presentation
(iii) Colon cancers frequently metastasise to liver
(iv) Colon cancers can be hereditary
(A) i,ii,iii,iv
(B) i,ii,iv
(C) i,ii
(D) i,iv
colon cancer
Cancer of the Large intestine & Rectum.
Risk factors
n

Alcohol, High Fat Diet, Red Meat.


n
Inflammatory Bowel Disease ( Ulcerative Colitis > Crohn’s ).
n
General Factors / Syndromes.

peutz jegher syndrome


-
STK11 gene defect on Chromosome 19, Autosomal Dominant.
Multiple Hamartomatous Polyps.
: Perioral Melanesia (+).
-
Increased risk for Pancreatic Cancer, Periampullary Cancer, Colorectal Cancer, etc.

Cowden syndrome
-
PTEN gene mutation , Autosomal dominant.
-
Hereditary Non Polyposis Colonic Cancer.
-
Familial Adenomatous Polyposis Coli Syndrome.

Diagnosis
-

Screening
Colonoscopy : Every 10yrs
: Sigmoidoscopy : Every 5 yrs
Fetal Occult Blood Testing : Every Year
: Virtual Colonoscopy
Colonoscopic biopsy is the IOC for diagnosis.
: PET CT is the IOC for staging.
Clinical features -

Depends on the site of Cancer.

:
Right sided Colorectal Cancer Left sided Colorectal Cancer
Ulcerative lesion are more common which Annular growth is seen ( Napkin ring Type ).
bleeds resulting in Iron deficiency anemia.
Late onset alteration of bowel habits. Early onset alteration of bowel habits
( increased frequency, tenesmus ).
Obstruction is a late feature. Obstructive symptoms are early.

MC site of distant metastasis : Liver


MC death in colon cancer : Liver Mets

:
Management
SURGICAL
Tumors in Caecum : Right hemicolectomy
: Tumor in Ascending Colon, Hepatic Flexure , Transverse Colon : Extended right hemicolectomy
Tumor in Splenic Flexure , Descending Colon : Left Hemicolectomy
:
Tumor in Sigmoid Colon
If >5 - 5.5 cm above the anal verge : Low anterior resection

: (Removal of part of sigmoid


colon & rectum)
If < 5cm above the anal verge : Abdominal Perineal Resection
n

Chemotherapy
n

Done in case of LN involvement & neoadjuvant chemotherapy


n

FOLFOX: 5-FLUROURACIL, OXIPLATINUM, FOLINIC ACID


n

FOLFIRI : 5-FLUROURACIL, FOLINIC ACID ,IRINOTECAN


162. What is the rate of flow in the given cannula?
(A) 66ml/min
(B) 86ml/min
(C) 96ml/min
(D) 106ml/min

163. A patient underwent mastectomy and a drain was kept. What type of drain is this?
(A) Closed
(B) Semi-closed
(C) Open
(D) Semi-open

Surgical drian
<

A surgical drain is a tube used to remove pus, blood or other fluids from a wound.
<

The given image shows a Romovac drain which is a closed type of drain.
<

There are 3 types of drains :


Closed drains : Utilise suction action for drainage.
<

Semi - Open drain : Utilise the principle of capillary action.


<

Open drain : Utilise the effect of gravity for drainage.


<
Other example for closed drains include : Jackson - Pratt drain.

164. A patient taken up for appendicectomy was given a grid iron incision initially which was later
converted to Rutherford Morrison incision. What are the structures cut during this conversion?
(i) External oblique
(ii) Internal oblique
(iii) Transversus abdominis
(iv) Rectus abdominis

(A) i, ii, iii


(B) i, ii
(C) ii, iii
(D) i, ii, iii, iv
Incisions used in Appendicectomy
McBurney Incision
Used in emergency appendicectomy.
: Grid - iron incision : A muscle splitting incision involving external oblique and internal oblique.
Rutherford Morrion Incision : It is a muscle cutting incision that cuts through

:
external oblique, internal oblique and transverse abdominis.

Lanz Incision/Skin crease Incision


Used in interval appendicectomy.
165. A patient presents to the OPD with a midline neck swelling which moves with deglutition. He also
has tachycardia and exophthalmos. What are the investigations required in this case?
(i) Thyroid scan
(ii) FNAC
(iii) Thyroid function tests
(iv) USG neck
(v) Anti-thyroid antibodies

(A) i & iii


(B) i, iii, iv, v
(C) i, iv, v
(D) ii, iii, iv
The given clinical history is suggestive of Grave’s disease.
Grave’s Disease
-
It is an autoimmune disorder in which antibodies are produced against the tyroid gland that
stimulate the gland uncontrollably causing hyperthyroidism.
Antibodies Involved : TSH receptor antibodies.
: It is associated with other autoimmune diseases like myasthenia gravis, Addison’s disease,
pernicious anemia, etc.
Clinically Features
-
Features of Hyperthyroidism (Weight loss, Diarrhea, Tachycardia, Heat Intolerance, Irritability)
Pretibial Myxedema.
: Eye Signs
Exophthalmous
: Stellwag’s Sign : Infrequent blinking
Von Graefe’s Sign : Lid lag
: Dalrympte’s Sign : Lid retrction
Joffrey’s Sign : Absence of forehead wrinkling
: Mobius Sign : Loss of accommodation reflex
Investigations
Thyroid function tests : Increase in TSH, T3, T4.
: USG Neck : Diffuse midline neck swelling.
7

Anti - Thyroid Antibodies present.


Management
-

Depends on the age group and type of patient.


Children : Anti - Thyroid drugs
-

Pregnant Woman : Propylthiouracil (Methimazole Contraindicated)


-

Adult > 45 years without goitre : Anti - Thyroid drugs followed by radioactive ablation.
-

Adult with Goitre : Anti - Thyroid drugs followed by surgery (total thyroidectomy)
-

Radioactive ablation is avoided in patients with eye signs.

166. 20yr old patient presents with history of RTA and injury to left forearm. On examination
brachial artery laceration of 3.5cm is seen. What is the preferred technique of repair in this
case?
(A) Segmental resection of vessel with reverse saphenous vein graft
(B) Repair using prosthetic graft
(C) Repair using a vein patch graft
(D) Primary repair
According to the given history, patient has a brachial artery laceration of 3.5 cm length.
: Because of an injury of this length, end to end anastomosis is not possible.
Vessel injuries that cannot be repaired by primary end to end technique will require an
interposition graft.
The most desirable graft is an autologous great saphenous vein harvested from an uninjured leg.
: Great Saphenous vein is preferred because :
Elastic properties
:
Adequate size match
Non - thrombogenic
:
Superior long term patency compared to prosthetic graft.
Orthopaedics
167. A 60yr old post menopausal woman presents to OPD with DEXA scan reports which showed a
score of -2.5. She has a history of Colle’s fracture 6 months back. She is not a smoker and doesn’t
take alcohol. Which of the following should be the first line of treatment for her?
(A) Calcium and Vitamin D3
(B) Alendronate
(C) Hormone replacement therapy
(D) Repeat DEXA scan after 3 years
According to the given history, the patient seems to be suffering from osteoporosis.

Osteoporosis
r

It is a quantitative defect of bone.


r

It is usually seen in elderly, especially in post - menopausal women.


r

Decrease in oestrogen, old age, causes decrease in bone mass and density.
r

It can be primary or secondary.


r

Primary : Seen in post - menopausal women, old age.


r

Secondary : Due to other factors (Steroid usage, chronic disuse,


Hyperparathyroidism, Rheumatoid Arthritis, Astronaut’s weightlessness.
r

Lab findings are all normal.


(Serum Calcium, Parathormone levels, Alkaline Phosphatase, Serum Phosphate)
Clinical Features
H/o pathological fractures (Most Common : Vertebral)
V

Kyphosis
V

Fish mouth appearance / Codfish appearance seen on X - Ray.


V
Diagnosis
/

Bone mineral density estimation using DEXA scan (Investigation of choice).


Comparison with normal populaton :

:
Z - Score T - Score
<2 > -2.5

Diagnostic of Osteoporosis

Treatment
Bisphosphonates (Drug of choice, First line)
Eg : Zoledronate, Risedronate, Alendronate
Denosumab (Monoclonal Antibody that inhibits RANK Ligand)
:
SERMs (Eg : Raloxifene)
Teriparatide (Synthetic PTH) : Given in pulsatile doses to stimulate bone formation.
:
Surgical Methods : Done to repair/reconstruct deformed bones. Eg : Kyphoplasty, Vertebroplasty.

168. Match the following

a. GCT i. Wide excision


b. Osteoid osteoma ii. Radiofrequency Ablation
c. Simple bone cyst iii. Intralesional steroid
d. Osteosarcoma iv. Curettage

(A) a-ii, b-i, c-iii, d-iv


(B) a-i, b-iii, c-iv, d-ii
(C) a-iv, b-ii, c-iii, d-i
(D) a-iii, b-iv, c-ii, d-i

Giant Cell Tumour (GCT)/ Osteoclastoma


It is a locally aggressive bone tumour usually seen in the elderly.
: Usually lacated at the epiphyseometaphyseal junction commonly at lower end of femur.
On examination, egg shell crackling sound on palpation.
:
X - Ray shows soap bubble appearance.
Biopsy shows giant cells that are multinucleated.
: Treatment : Extended Curettage + Bone Grafting
Osteoid Osteoma
It is the most common true benign tumour.
:
Seen in children and young adolescents.
Clinically, patients give h/o night pains not resolved by Salicylates.
: Most Common location : Femoral diaphysis.
X - Ray shows a central nidus and dense sclerosis around it.
: Treatment : Radiofrequency ablation, Excision.

Simple Bone Cyst


It is a benign bone cyst usually seen in the metaphyseal region of proximal humerus.
: X - Ray shows unilocular appearance with fallen - leaf sign.
Needle aspiration reveals a serous straw coloured clear fluid.
: Treatment : Aspiration followed by injection of sclerosants or steroids.

Osteosarcoma
Most common primary malignant tumour of bone.
:
Usually seen in metaphysis of long bones like femur.
It is highly vascular (pulsatile tumour), radioresistant, bone forming tumour.
:
It can be primary in origin or can secondarily arise from diseases like Paget’s disease, fibrous
dysplasia, radiation.
X - Rays shows characteristic sun - burst appearance with elevated periosteum
: (Codman’s Triangle).
Treatment : Neoadjuvant chemotherapy followed by surgery.

GCT Osteoid Osteoma


Simple Bone Cyst Osteosarcoma

169. Injury of nerve resulting in cutting of axons with intact outer nerve sheath is termed as
(A) Axonotmesis
(B) Neuropraxia
(C) Neurotmesis
(D) Axonopraxia
Seddon’s Classification of Nerve Injuries
Neuropraxia
Occurs due to prolonged nerve compression.
No injury to nerve structure.
Healing occurs spontaneously.

Axonotmesis
The axons of nerve is injured but with intact nerve sheath.
Can heal spontaneously but sometimes require surgical repair depending on the injury.

Neurotmesis
Complete transection of nerve occurs.
Almost always require surgical repair.

Seddon and Sunderland Classification of Nerve Injury


170. Arrange the following in increasing order of stability
(i) Ilizarov circle fixator
(ii) Biplanar frame at right angle
(iii) Uniplanar with one rod
(iv) Uniplanar with two rods

(A) i, ii, iii, iv


(B) iii, ii, iv, i
(C) iii, iv, ii, i
(D) iv, iii, i, ii
Order of Stability
Ilizarov Circle Fixator > Biplanar Frame at Right Angle > Uniplanar with 2 rods > Uniplanar with 1 rod

171. A patient was brought to the casualty following an RTA. He was evaluated and was found to have
fracture shaft of femur which needs to be stabilised by tibial traction. Which among the following are
the necessary components for skeletal traction?
(i) Thomas splint
(ii) Bohler stirrup
(iii) Steinmann pin
(iv) K wire
(v) Bohler frame
(vi) Denham’s pin

(A) i, iii, iv, vi


(B) ii, iii, v
(C) ii, v, vi
(D) i, ii, v, vi

Skeletal Traction
I

Skeletal traction is a treatment method to treat broken bones.


1

It is a system where combination of pulleys, pins and weights are used to promote the healing of
fractured bones.
They are usually done in the lower body.
: A pin is placed inside the bone which acts as a base for a pulley system.
I

It uses gradual puling force to realign the broken bones.


Equipments required for tibial Skeletal Traction

Steinmann Pin Bohler Stirrup

Bohler - Brawn Splint Weights to provide traction


172. A patient with tibial fracture put on cast presented to casualty with complaints of pain of the
injured limb. His cast was cut and on examination there is pain on passive flexion of limb, loss of
sensation in first web space and posterior tibial and dorsalis pedis pulses were present. What is the
next best step in management?
(A) Give analgesics
(B) Reapply cast
(C) Doppler study
(D) Measure anterior compartment pressure
Based on the given history, the patient is most likely suffering from compartment syndrome.

Compartment Syndrome
-
-

It is usually seen in cases where a tight cast is applied to a limb following fracture resulting in
raised pressure inside the cast.
-

This can lead to decreased blood supply to the affected limb/part.


-

Ischemia leads to inflammation of the muscles causing them to swell up inside the facial
compartment and compress the supplying blood vessels (veins and arteries) leading to further
ischemia.
Clinical Features Diagnosis
Pain on passive stretching
-

The intra - compartmental pressure is measured.


Pallour
-

> 30 mm Hg is diagnostic of compartment syndrome.


Parasthesia
-

Paralysis
-

Pulselessness

Management
Remove cast.
-

Fasciotomy to relieve pressure.


-

173. Identify the correct sequence of performing Thomas test


(i) Squaring of pelvis
(ii) Flexion of hip
(iii) Overcorrection on normal side
(iv) Check for lumbar lordosis

(A) ii, iii, i


(B) iv, iii, ii
(C) ii, iv, iii
(D) iii, iv, i
Thomas Test
Used to check for flexion contracture of hip.
:
Steps involved
Make the patient lie supine and pass the palm of the hand beneath the patient’s spine to
: identify lumbar lordosis.
The unaffected hip is flexed until the thigh just touches the abdomen to obliterate the
lumbar lordosis.
Clinician then passively ranges the affected hip into extension.
:
Once the pelvis begins to tilt anteriorly, stop the passive range of motion, hold the
affected thigh in position and measure the angle between the affected thigh and table to
reveal the fixed flexion of the hip.
<

Interpretation
<

If iliopsoas is shortened or contracture is present, the lower extremity on the affected side will
be unable to fully extend at the hip causing lumbar lordosis (Positive Thomas Test).
Paediatrics
174. Match the following:
(a) Social smile (i) 1-2 months
(b) Pincer grasp (ii) 5-6 months
(c) Walks 1-2 steps (iii) 9-11 months
(d) Transfer objects (iv) 12-13 months

(A) a-i, b-iii, c-iv, d-ii


(B) a-ii, b-i, c-iii, d-iv
(C) a-iii, b-ii, c-iv, d-i
(D) a-i, b-ii, c-iii, d-iv

Key Gross Motor Developmental Milestones


Age Milestone
3 mo Neck holding
5 mo Rolls over
6 mo Sits in tripod fashion (sitting with own support)
8 mo Sitting without support
9 mo Stands holding on (with support)
12 mo Creeps well; walks but falls; stands without support
15 mo Walks alone; creeps upstairs
18 mo Runs; explores drawers
2 yr Walks up and downstairs (2 feet/step);jumps
3 yr Rides tricycle; alternate feet going upstairs
4 yr Hops on one foot; alternate feet going downstairs

Key Fine Motor Developmental Milestones


Age Milestone
4 mo Bidextrous reach (reaching out for objects with both hands)
6 mo Unidextrous reach (reaching out for objects with one hand); transfers objects
9 mo Immature pincer grasp; probes with forefinger
12 mo Pincer grasp mature
15 mo Imitates scribbling; tower of 2 blocks
18 mo Scribbles; tower of 3 blocks
2 yr Tower of 6 blocks; vertical and circular stroke
3 yr Tower of 9 blocks; copies circle
4 yr Copies cross; bridge with blocks
5 yr Copies triangle; gate with blocks
Key Social and Adaptive Milestones
Age Milestone
2 mo Social smile (smile after being talked to)
3 mo Recognizes mother; anticipates feeds
6 mo Recognizes strangers, stranger anxiety
9 mo Waves "bye bye"
12 mo Comes when called; plays simple ball game
15 mo Jargon
18 mo Copies parents in task (e.g. sweeping)
2 yr Asks for food, drink, toilet; pulls people to show toys
3 yr Shares toys; knows full name and gender
4 yr Plays cooperatively in a group; goes to toilet alone
5 yr Helps in household tasks, dresses and undresses

Key Language Milestones


Age Milestone
1 mo Alerts to sound
3 mo Coos (musical vowel sounds)
4 mo Laugh loud
6 mo Monosyllables (ba, da, pa), ah-goo sounds
9 mo Bisyllables (mama, baba, dada)
12 mo 1-2 words with meaning
18 mo 8-10 word vocabulary
2 yr 2-3 word sentences, uses pronouns" I", "me", "you"
3 yr Asks questions; knows full name and gender
4 yr Says song or poem; tells stories
5 yr Asks meaning of words

175. Which of the following can cause neonatal seizures?


(i) Hyponatremia
(ii) Hypernatremia
(iii) Hypocalcemia
(iv) Hypomagnesemia

(A) i, iii, iv
(B) i, ii, iii, iv
(C) ii, iii
(D) iii, iv
Neonatal Seizures
-

They are usually the clinical manifestation of a serious underlying disease.


They are medical emergencies because they signal a disease process that may produce
: irreversible brain damage.
A seizure is a paroxysmal behaviour caused by hyper - synchronous discharge of a group
of neurons.
-

They can be Subtle, Tonic, Clonic, Myoclonic.


Causes
-
-

Developmental defects
Hypoxic Ischaemic Encephalopathy
: Intracranial Haemorrhage
Metabolic Causes (Hypoglycemia, Hypocalcemia, Hypomagnesemia,
: Hyponatremia, Hypernatremia)
CNS Infections (E. Coli, Klebsiella, Staphylococcus, Candida, HSV)
Inborn errors of metabolism.

:
Investigations
Blood Sugar
Haematocrit
: Bilirubin
Serum Electrolytes
: ABG Examination
CSF Study
:
EEG
CT, MRI
: Metabolic Screening for Inborn errors
(Blood and urine ketones, Blood Ammonia, Anion Gap, Urine Reducing Substances)
Screening for congenital infections, TORCH Infections

:
Management
A/c attack
Ensure airway, breathing and circulation
O2 inhalation, Thermoneutral environment
:
Blood sugar estimation
i.v access and fluid administration
: Manage according to cause of seizure.
176. Most common agent causing neonatal meningitis?
(A) Streptococcus
(B) Hemophilus
(C) Mycoplasma
(D) E. coli
Pediatric Meningitis
Cause
-

0 - 2 months : Group B Streptococci (MC Cause Worldwide), E. Coli, Klebsiella, Staphylococcus


-

2 months - 2 years : Haemophilus Influenzae, Streptococcus pneumonia


-

2 years - 20 years : Meningococcus, Pneumococcus

Clinical features
-

Fever
-

Lethargy
-

Irritability
-

Shrill Cry
-

Seizures
-

Meningeal Signs are arre below 2 years of age.

Complications

Seizures (Most Common)


Raised ICP
:
Subdural Effusion
Hydrocephalus
: Sensorineural deafness
Treatment
i.v Ceftriaxone (Drug of choice).
:
Corticosteroids to reduce inflammation.

177. At what age does a baby attain half of the adult height?
(A) 12-18 months
(B) 28-32 months
(C) 18-24 months
(D) 32-36 months
Height/Length
^

It is measured using infantometer in children < 2 years and by using stadiometer in


children > 2years.
A normal child at birth is 50 cm long.
:
Reaches 75 cm by first year and 87.5 cm by second year.
Thereafter steadily increases by 6 cm per year.
:
A baby reaches half of their adult height by 2 years of age.
Height doubles by 4 years and triples by 12 years of age.

:
178. Arrange the causes of mortality of neonates in descending order (according to 2013 data) as
based on verbal autopsy data for India
(i) Prematurity
(ii) Birth asphyxia
(iii) Sepsis
(iv) Congenital anomalies

(A) i, iii, ii, iv


(B) ii, i, iii, iv
(C) iii, iv, ii, i
(D) i, ii, iii, iv
Causes of mortality in neonates according to WHO data
Prematurity (16%)
Intrapartum events like asphyxia (11%)
:
Sepsis (7%)
Congenital Anomalies (5%)
: Pneumonia (3%)

179. A neonate at 14 weeks of age presents to the OPD with fever, bilateral conjunctivitis and fast
breathing. His chest X-ray showed infiltrates on both sides. Which is the most likely infection present?
(A) Mycoplasma
(B) H. infuenzae
(C) Streptococcus
(D) Chlamydia

The given history is suggestive of ophthalmia neonatorum progressing to pneumonia.


Neonatal Conjunctivitis
-

Also known as ophthalmia neonatorum.


-

It can arise due to various causes (chemical conjunctivitis, bacterial conjunctivitis, viral, etc).
Most common cause is Chlamydia trachomatis infection.
:
It usually occurs with 5 - 14 days of birth.
-

The mother’s birth canal is the source of infection.


Neonates are brought with h/o congestion and papillary reaction of eyes.
:
Conjunctival scrapings show basophilic inclusion bodies which are characteristic of Chlamydia
infection.
-

Complications : If not treated can lead to sepsis resulting in pneumonia, meningitis, etc.
-

Treatment : Erythromycin eye oinment + Syrup 50 ml/kg/day divided into 4 doses +


Treatment of parents.

180. A neonate at 14 weeks of age presents to the OPD with fever, bilateral conjunctivitis and fast
breathing. His chest X-ray showed infiltrates on both sides. Which is the most likely infection
present?
(A) Mycoplasma
(B) H. infuenzae
(C) Streptococcus
(D) Chlamydia
Among the given options, only TAPVC will have equal saturation in all chambers of the heart.
TAPVC (Total Anomalous Pulmonary Venous Connection)
r

In this case. There is anomalous connection of pulmonary veins present.


r

3 types
r

Supracardiac
r

Cardiac
r

Infracardiac

Supracardiac
r

Most common type.


r

Pulmonary veins drain into superior vena cava.


r

Leads to overloading of right aorta, blood reaches left atria through an atrial septal
defect leading to mixing of oxygenated and deoxygenated blood in all chambers.
r

X - ray shows figure of 8/ Snowman appearance.


Cardiac
r

Pulmonary vein drains directly into right atrium.

Infracardiac
r

Pulonary vein joins portal or hepatic vein which in turn drains into right atria.
r

Since the mixing of oxygenated and deoxygenated blood occurs in all chambers of heart, oxygen
saturation is same in all of them.
Obstetrics & Gynaecology
181. Identify the given speculum
(A) Cusco speculum
(B) Siegel speculum
(C) Graves speculum
(D) Sims speculum
It is a self retaining bivalved speculum used for vaginal and cervical examination.

Sim’s Vaginal Speculum Siegel Pneumatic Speculum

Grave’s Speculum
182. Which is the most commonly used emergency contraceptive in India?
(A) Ullipristal 30mg
(B) CuT IUD
(C) Implants
(D) Levonorgestrel 1.5mg
Emrgency Contraception
r

Levonorgestrel
r

Most Commonly used.


r
0.75 mg X 2 tablets, 12 hours apart within 72 hours of intercourse
OR
r
1 mg single dose within 72 hours of intercourse
r

Ullipristal
r
30 mg within 5 days of intercourse.
r

Mifepristone
r
600 mg within 72 hours of intercourse.
r

IUCD
r

Used within 5 days of intercourse.


Best method.

:
183. All are long-term and reversible contraceptives except
(A) Tubal sterilisation
(B) Implanon
(C) IUCD
(D) LNG IUD
Among the given options, tubal sterilisation is an irreversible method of contraception.
Long Acting Reversible Contraceptives
Methods of birth control that provide contraception for an extended period without
requiring user action.
-

They include :
Intrauterine devices - Hormonal, Non - Hormonal
-

Implants
: Injectables
184. Calculate the bishop score of the patient from the given data; Cervix at station -1, 1cm dilated,
30% effaced, posterior position and soft consistency.
(A) 5
(B) 3
(C) 1
(D) 7
Bishop Scoring System

Score Dilation Position of cervix Effacement Station Cervical


(cm) (%) (-3 to +3) Consistency
0 Closed Posterior 0-30 -3 Firm
1 1-2 Mid position 40-50 -2 Medium
2 3-4 Anterior 60-70 -1,0 Soft
3 5-6 — 80 +1.+2 —

It is used as a pre - labour scoring system to assist in predicting whether induction of labour will
required.
If score >= 9 : Cervix is favourable & induction is successful.

: <= 6 : Cervix is not favourable, induction is unlikely to be succesful, ripening of cervix is


needed.
In modified Bishop score, length of cervix is used instead of effacement.
185. A 25yr old female presents to OP with complaints of greenish discharge per vaginum associated
with local pruritus. On local examination cervix has a strawberry appearance. What is the treatment?
(A) Fluconazole
(B) Metronidazole
(C) Cefixime
(D) Azithromycin
The given history is suggestive of trichomoniasis.
Trichomonas Vaginitis
Caused by trichomonas vaginalis.
:
Patients c/o yellowish green discharge, pruritis and dysuria.
On examination, cervix looks inflammed and has characteristic strawberry like appearance.
: (Colpitis Macularis)
Diagnosis is made by saline microscopy of cervical smear which shows flagellated protozoa.
Culture is gold standard for diagnosis and is done on Diamond media.
: Treatment
Metronidazole 500 mg TDS X 7 days OR 2 g oral stat
:
Partner treatment is required.
186. A 20yr old female presents with complaints of discharge per vaginum and local pruritus. A Pap
smear was done which showed the following picture. What is the possible cause of this infection?
(A) Candida
(B) Trichomonas
(C) Gardnerella
(D) Chlamydia

The given image and history is suggestive of bacterial vaginosis infection.


Bacterial Vaginosis
Caused by Gardnerella, Mobilincus or Ureaplasma urealyticum.
: Patients c/o dirty white foul smelling discharge per vaginum.
No h/o itching or pruritis.

:
Diagnosis by Amsel’s criteria
Any 3 out of the 4 criteria
Dirty white foul smelling discharge thinly coating vagina.
: pH of discharge >= 4.5.
Clue cells >= 20% on saline microscopy

: Positive Whitt test (fish like odour on adding 10% KOH to discharge)

Treatment : Metronidazole 500 mg TDS X 7 days (avoided in first trimester)


187. Which of the following is most commonly seen in a patient suffering from Asherman syndrome?
(A) Oligomenorrhea
(B) Hypomenorrhea
(C) Dysmenorrhea
(D) Menorrhagia
Asherman Syndrome
It is an acquired uterine condition that occurs when adhesions form inside the uterus
:and/or the cervix.
It is characterised by variable scarring inside the uterine cavity, where in many cses the
front and back walls of uterus stick to one another.
It can arise following dilation and curettage (D & C) performed after a miscarriage,
:delivery or abortion, pelvic surgeries like Caesarean section, myomectomy, etc.
It can also arise secondary to genital TB, schistosomiasis and pelvic irradiation.
Patients present with menstraul abnormalities (hypomenorrhea), infertility, pain
: during menstruation and ovulation, miscarriages, etc.
Treatment : Hysteroscopic adhesiolysis.

188. What is the dosage of dexamethasone used for fetal lung maturation in pregnancy?
(A) 6mg 4 doses 12hrly
(B) 6mg 4 doses 6hrly
(C) 12mg 2 doses 12hrly
(D) 12mg 2 doses 24hrly
Antenatal steroids are indicated in pregnant women who are at risk of delivering early (<34

: weeks) as they help in maturation of foetal lung and helps prevent respiratory distress
syndrome in neonates.
Steroids are given as :
Dexamethasone : 6 mg X 4 doses (12 hours apart)
: Betamethasone : 12 mg X 2 doses (24 hours apart)

189. As per the IADPSG the fasting glucose cutoff for proposed single step testing with 75g oral
glucose for gestational diabetes is
(A) 90
(B) 95
(C) 92
(D) 100
Iadpsg criteria -

For IADPSG criteria, an OGTT is done in the fasting state using 75g of glucose at 24 - 28
weeks of gestation.
l

GDM is diagnosed if any of the following cut - off is met :


Fasting : >= 92 mg/dl
l

1 hour : >= 180 mg/dl


l

2 hours : >= 153 mg/dl


l

Dipsi criteria
-

The criteria most commonly followed in India is the DIPSI criteria :


No fasting required.
-

75 g Glucose is given orally to the pregnant female whenever she visits the antenatal OPD,
-

irrespective of gestational age.


Sugar levels are checked after 2 hours :
-

>= 120 mg/dl : Glucose intolerance


-

>= 140 mg/dl : GDM


-

>= 200 mg/dl : Overt diabetes

190. Pap smear is done in young women mainly for


(A) Cancer screening
(B) Diagnosis of UTI
(C) Checking for PID
(D) Confirming pregnancy

Pap Smear/ Papanicolaou test


l

It is a method of cervical screening used in detection of precancerous and cancerous


processes in the cervix.
Abnormal findings if present are followed up by more sensitive diagnostic procedures.
: It is usually taken when female is not menstruating.
Procedure
-

A speculum is inserted into the patient’s vagina, allowing access to the cervix.
-

Sample is collected from the outer os of cervix by scrapping it with Ayre’s spatula.
-

An endocervical brush is rotated in the central opening of the cervix.


-

The specimen is smeared on to a glass slide, fixed using 95% ethanol and observed
under microscopy.

Pap smear is started at 21 years of age irrespective of age at first intercourse.


-

It is repeated every 3 years.


Ayre’s Spatula Endocervical Brush

191. Which of the following statements is not true about clomiphene citrate?
(A) Causes endometrial thinning
(B) Can be used with gonadotrophin
(C) Inhibits negative feedback on GnRH
(D) Causes monofollicular development
Clomiphene Citrate
-

It is a selective oestrogen receptor downregulator.


It is used in ovulation induction in females facing infertility.
:
Mechanism : It acts as a partial agonist at pituitary oestrogen receptors thereby increasing GnRH
levels causing LH to rise.
Increased LH levels help in stimulating ovulation.
:
Side Effects
Multiple gestation (as it casues polyfollicular development)
Ovarian cancer
: Ovarian cyst
Thinning of endometrial lining, abnormal uterine bleeding.
: Hepatotoxicity
192. A 17yr old girl presented with complaints of irregular cycles, increased facial hair and change in
voice. USG abdomen revealed bulky ovaries of size 30ml and 40ml. Which of the following are used as
the first line of treatment?
(i) Weight reduction
(ii) OCP
(iii) Spironolactone
(iv) Laparoscopic ovarian drilling

(A) ii, iii, iv


(B) i, ii, iv
(C) i, ii, iii
(D) i, iii, iv
The given history is suggestive of Polycystic Ovarian Syndrome.
Polycystic Ovarian Syndrome (PCOS)

It is the most common endocrine disorder in women of reproductive age group.


:
It is caused by a combination of genetic and environment factors.
Risk Factors
Obesity
:
Lack of physical excercise
Family history
: Junk food

Clinical Features
Irregular or absence of menstrual cycles
:
Excess body hair, facial hair
Acne
:
Infertility
Acanthosis Nigricans
: Metabolic Syndrome (DM, Obesity)

Diagnosis
Rotterdam’s Criteria (Any two of the following)
:
Hyperandrogenism (Hirsutism)
Ovulatory dysfunction (Amenorrhea, Irregular Cycles)
:
USG evidence of PCOS : Either in one or both ovaries
>= 12 follicles in ovary
: Ovary volume >= 10cc
Follicles < 1 cm in size
i
Management
I
-

Weight Loss
I
-

Metformin : To deal with insulin resistance and metabolic syndrome


I
-

OCPs : To regularise menstrual cycles, to treat hirsutism


I
-

Infertility
First Line : Letrozole (DOC), Clomiphene Citrate, Bromocriptine
I
-

Second Line : Human menopausal gonadotropin, Laparoscopic Ovarian drilling


I
-

Third Line : Synthetic GnRH


I
-

193. According to the latest WHO guidelines, which of the following should be done in the
management of 2nd stage of labour?
(A) Delivery in lithotomy position
(B) Hot compress to prevent perineal tear
(C) Routine episiotomy
(D) Manual support to perineum to maintain deflexed head

Stages of Labour
Four Stages of Labour
-

Stage I
Period between onset of true labour pains and the full dilation of cervix, further divided
i into two phases.
Latent : Uptil 5 cm of dilation
-

Active : 6 cm - 10 cm of dilation
-

Stage II
Period bewteen full dilation of cervix and delivery of body.
-

Stage III
Period between delivery of baby and delivery of placenta.
-

Stage IV
1 hour observation period after delivery of placenta.
-
Time Period
First Stage Second Stage Third Stage Fourth Stage
Normal
Duration Latent Phase Passive 1 hour
Nulliparous 12 hours 1 hour Management
Multiparous 8 hours 30 minutes (15 - 25 mins)
Active
Management
(5 - 10 mins)

Prolonged 20 hours 2 hours More than


duration 14 hours 1 hour 30 mins
Nulliparous (+ 1 hour each if
Multiparous epidural given)
Retained
Placenta

WHO Guidelines for 2nd stage of labour


They are practiced to reduce perineal tears during delivery of child.
: They include
No rouitne episiotomy
:
Modified Ritgen maneuver
Manual support to perineum with right hand while

: delivering the head of foetus, flexing head of


foetus with left hand.
Hot compress to the perineum
Advise patient not to bear down while delivering the head.
: Delivery in any position that is safe and comfortable for the
baby and mother.
I

Perineal massage.

194. Dose of carbitocin used in PPH?


(A) 50mcg
(B) 100mcg
(C) 150mcg
(D) 200mcg  
Carbitocin is a synthetic oxytocin used as a uterotonic in the management of 3rd stage of labour.
It is given at a dose of 100 mcg as slow iv to prevent PPH.
Third Stage of Labour
-

Begins after delivery of baby and ends at the delivery of placenta.


Crucial stage in development of PPH.
:
It can be managed passively or actively.
Passive management has greater risk for development of PPH and maternal mortality.
: Active management of Third stage of Labour is the best method to prevent PPH.
Active Management of Third Stage of Labour (AMTSL)

Step 1 : Administer uterotonic agent within 1 min of delivery


-

Oxytocin
Drug recommended by WHO
-

First line to prevent PPH.


: Dose : 10 IU im or iv infusion.
-

Methyl Ergotamine
Dose : 0.2 mg im.
: Contraindicated in organic heart disease, eclampsia, pre - eclampsia,
-

Rh incompatibility and after the delivery of first twin in twin pregnancy.


-

Syntometrine
-

Dose : 5U Oxytocin + 0.5 mg methyl ergometrine


-

Carbitocin
-

Synthetic oxytocin.
-

Dose : 100 mcg, slow iv.


-

Misoprost (PGE1)
-

Dose : 600 mcg (oral)


-

Carboprost (PGF - 2 α)
-

Best drug to prevent and treat PPH.


-

Dose : 250 mcg (im)

Step 2 : Delayed Cord Clamping


<

1 min - 3 mins.
<

Indications for early cord clamping


Birth asphyxia
<

Neonate requiring resuscitation


<

Rh -ve pregnanncy
<

Heart disease in body


<

HIV (+) mothers


<

Diabetic mothers
<
Step 3 : Delivery of Placenta by controlled cord traction
(Modified Brandt Andrews Technique)
Step 4 : Intermittent assessment of uterine tone

195. A 19yr old presents to the OP with primary amenorrhea. On examination breast development,
pubic and axillary hair growth were normal. USG showed absence of uterus. What could be the
diagnosis?
(A) Complete androgen receptor insensitivity
(B) Mullerian agenesis
(C) Turner’s syndrome
(D) Gonadal dysgenesis
According to the given history, the patient has normal secondary sexual characters (Breast
developed, pubic and axillary hair present) but uterus is absent.
This is more suggestive of Mullerian agenesis.
Amenorrhea
-

It refers to absence of menstruation.


-

It can be primary or secondary.


Primary Amenorrhea
-

Has never experienced menarche.


-

Does not attain menarche by 13 years of secondary sexual characters OR


-

15 years of age in the presence of secondary sexual characters.


-

Most Common Cause : Gonadal Dysgenesis

Secondary amenorrhea
-

H/O previous normal menstruation.


-

Not menstruating for > 90 days with previous h/o normal menstruation.

Mullerian Agenesis
Arise due to absence of both mullerian ducts, hence fallopian tube, uterus, cervix and upper
l

vagina are absent.


Ovaries, lower part of vagina present.
1

Patient has normal karyotype and normal secondary sexual characters.


:
Management : Vaginoplasty
Female can’t menstruate or become pregnant but can take part in IVF treatment + Surrogacy.
I
Cryptomenorrhea
-

Normal menstrual cycles with absent bleeding due to imperforate hymen (Most Common),
transverse vaginal septum or vaginal atresia.
-

Results in collection of blood in vagina, cervix and uterus. (Hematocolpos/Hematometra).


-

Examination shows tensed bluish bulging hymen and bulky abdomen.


-

Rx : Cruciste incision on hymen, Resection of hymen.

Androgen Insensitivity Syndrome


Patient has a male genotype (46 + XY) with undescended testes in abdomen.
: Testes has sertoli cells and leydig cells that produce Mullerian inhibiting factor and
Testosterone respectively.
Patient is resistant is resistant to testosterone.
: Wolffian duct regresses, so male internal organs are absent.
Testosterone gets converetd to oestrogen that leads to development of female external
: Mullerian Inhibiting Factor inhibits Mullerian duct development leading to absent uterus,
genitalia and breast.

Fallopian tube, Cervix and Upper Vagina.


Pubic and axillary hair are absent as they are resistant to the action of dihydrotestosterone.
:
Management : Vaginoplasty, Gonadectomy, Oestrogen Replacement Therapy.
Can’t become pregnant or take part in IVF.

:
Gonadal Dysgenesis
46 + XX Genotype but ovaries fail to develop.
Similar to Turner’s Syndrome but with normal height and additional features are absent.
: Management : Oestrogen Replacement Therapy

Swyer Syndrome
Has 46 + XY Genotype but testes fails to develop properly.
-

Testosterone and anti - Mullerian hormone is produced.


:
Hence, Mullerian duct develops (Uterus, Cervix, Upper Vagina and Fallopian Tube present)
External genitalia resembles that of females.
-

Breast development, pubic and axillary hair absent.


: Management : Individuals are treated as females, gonadectomy is done, vaginoplasty and
hormone replacement therapy.

Kallmann Syndrome
Hypogonadotropic hypogonadism (decreased levels of GnRH, FH, LH).
:
Anosmia is a characteristic feature.
Height is normal.
:
Management : Pulsatile GnRH therapy.
Turner’s Syndrome
-

Chromosomal disorder characterised by 45 XO genotype.


-

Ovaries are not fully developed (streak ovaries), uterus, fallopian tube, cervix, upper vagina
present but they are hypoplastic as oestrogen levels are low.
Breast development is absent, external genitalia is female.
: Clinically additional features :
Webbing of neck (+)
Shield shaped chest
Widely spaced nipple
Cubitus valgus, Short stature
Low posterior hair line
Short fourth metacarpal
Heart disease likely (Most Common : Bicuspid Aortic Valve)
Normal IQ
-

Management : Oestrogen Replacement Therapy

Algorithm for evaluation of Primary Amnorrhea

Case A Case B
Secondary Sexual Secondary Sexual
Characteristics Normal Characteristics Absent

Uterus Height of Patient

Present Absent Short Normal


(Cryptomenorrhea) Stature
<

Pure gonadal
(Turner dysgenesis
Karyotype Syndrome) Swyer Syndrome
:
Kallmann Syndrome

46 + XX 46 + XY
Mullerian AIS
Agenesis
196. Which of the following tests can be done to differentiate between testicular failure and
obstructed vas deferens in male azoospermia?
(A) FSH levels
(B) Testosterone levels
(C) Testicular FNAC
(D) LH levels
The best test to differentiate between obstructive and non - obstructive azoospermia is
serum FSH levels.
Azoospermia
-

Absence of sperms in semen.


-

It can be broadly classified into 3 based on its etiology.


Pretesticular
-

Testicular
:
Post - Testicular
Pretesticular Azoospermia
-

Here the defect is in hypothalamus and pituitary.


-

There is no release of gonadotropin releasing hormone (GnRH) resulting in low levels of FSH,
LH, and Testosterone.
-

Spermatogenesis is absent.

Testicular Azoospermia
The defect is in the testes.
-

Gonadotropins levels are raised as testes doesn’t produce testosterone.


-

Hence, FSH and LH are raised but testosterone levels remain low.
-

Spermatogenesis can be present to a small extent.


-

Post - Testicular Azoospermia


Also known as azoospermia.
: There is defect in the pathway of sperms.

Gonadotropin levels are normal (FSH, LH normal), testosterone levels are also normal.
Obstruction present

Before the seminal vesicles After the seminal vesicles


"

Sperms absent ,

Sperms absent
I

Seminal vesicle fluid rich in fructose present \

Seminal vesicle secretions absent


I

Prostate fluid present \

Prostate fluid present


"

Bulbourethral Secretions (+) ,

Bulbourethral secretions present.


Scrotal ultrasound is done to identify the level of obstruction.

197. Milk production in pregnancy is inhibited by


(A) Low LH
(B) Human somatomammotrophin
(C) High estrogen
(D) Low TSH
Lactation
Secretion of milk from mammary glands.
:
Milk production is termed as galactopoiesis.
Preparation of breasts for lactation starts from 18 weeks of pregnancy.

i
Hormonal Influence
Progesterone
Promotes growth of alveoli and mammary lobes, high levels inhibit lactation before
I

birth, fall in progesterone levels after birth is the trigger for milk production.
1

Oestrogen
Promotes growth and differentiation of milk duct system.
:
High levels inhibit lactation.
1

Prolactin
Main hormone involved in milk production.
:
Contributes to growth and differentiation of alveoli and duct structure.
Levels are highest during pregnancy.
I

Human Placental Lactogen (HPL)


Produced by placenta from second month of pregnancy.
: Contributes to breast, nipple and alveolar growth before birth.
I

Oxytocin
Necessary for milk ejection reflex in response to sukling.
1
198. A 65yr old lady with diabetes and hypertension came to the OPD with complaints of bleeding per
vaginum for the past 1 month. What should be the ideal investigation for her?
(A) Colposcopy
(B) Endometrial biopsy
(C) Pap smear
(D) VIA
-

The given clinical history is suggestive of post - menopausal bleeding (PMB).


-

The investigation of choice in PMB is endometrial sampling.


Post - Menopausal Bleeding (PMB)
-

Vaginal bleeding in a women after menopause.


-

Most Common cause : Atrophic Vaginitis.


-

1 in 10 cases can have endometrial cancers, hence all cases of PMB should be evaluated adequately.
Causes
-

Atrophic vaginitis (60-80%)


-

Estrogen treatments (15-25%)


-

Polyp cervical -uterine (12-12%)


-

Endometrial Hyperplasia (5-10%)


-

Endometrial cancer (10%)


-

No cause found (10%)

Evaluation
USG
:
Endometrial Biopsy
Fractional Curettage + Hysteroscopic Biopsy

:
USG
First investigation.
Helps identify cause of bleeding (Polyps, Endometrial hyperplasia, Endometrial cancers, etc)
: If endometrial thickness > 4mm, endometrial biopsy is indicated.

Endometrial Biopsy
-

Investigation of choice in PMB.


-

Done as an OPD procedure, no anaesthesia required.


-

Indications
>= 45 years with abnormal uterine bleeing.
-

< 45 years with AUB in the presence of unopposed oestrogen action.


-

Persistent bleeding irrespective of age.


-

Failure of medical management.


-

If a focal growth is identified, hysteroscopic biopsy is done.


Fractional Curettage + Hysteroscopic Biopsy
-

Gold Standard.
-

Done in OT under GA.


-

Indications
Endometrial biopsy report inadequate.
-

Endometrial biopsy showing hyperplasia with atypia.


-

Cervical Stenosis (+).


-

Persistent bleeding irrespective of biopsy repair.


-

Management
-

In case of endometrial hyperplasia without atypia, treatment is done with progesterone


injection or IUD.
-

In case of hyperplasia with atypia, hysterectomy with removal of ovaries and fallopian
tubes is advised.

199. Match the ovarian tumors with their corresponding tumor markers
(a) Dysgerminoma (i) LDH
(b) Yolk sac tumor (ii) beta-HCG
(c) Granulosa cell tumor (iii) AFP
(d) Choriocarcinoma (iv) Inhibin
(e) Epithelial cell tumor (v) CA-125

(A) a-i, b-iii, c-iv, d-ii, e-v


(B) a-ii, b-iv, c-v, d-iii, e-i
(C) a-iii, b-ii, c-i, d-iv, e-v
(D) a-iv, b-i, c-iii, d-v, e-ii

Tumour Markers of Ovarian Tumours


-

Non - Mucinous Epithelial Tumour - CA125


-

Mucinous Epithelial Tumour - CA 125, CA 19 - 9


-

Yolk Sac Tumour - AFP, Anti - trypsin


-

Choriocarcinoma - hCG
-

Dysgerminoma - LDH, Placental ALP


-

Embryonal Tumour - hCG, AFP


-

Granulosa Cell Tumour - Inhibin


Characteristic Features of Ovarian Tumour
I

Serous Cystadenocarcinoma
Psammoma bodies seen
/

Mucinous Cystadenoma
Associated with Pseudomyxoma peritonei
/

Brenner’s tumour
Coffee bean nuclei + Walthard cell nests seen
: Associated with Pseudo Meig’s syndrome
/

Dysgerminoma
Most common ovarian tumour in pregnancy.
:
Most common ovarian tumour associated with dysgenetic gonads.
Most radiosensitive ovarian tumour.
: Best prognosis.
Yolk Sac Tumour
: Most malignant GCT
Worst prognosis
: Unilateral in all cases,
Schiller duval bodies seen on histological examination.

: Granulosa Cell Tumour


Coffee bean nuclei seen.
: Call exner bodies seen.
200. Which of the following is not included in the quadruple screening test?
(A) Inhibin B
(B) AFP
(C) hCG
(D) Unconjugated estradiol (E3)
Aneuploidy Screening
First Trimester
r
Biochemical Tests
r

Dual test
r

Pregnancy - associated Plasma Protein A (PAPP - A)


r
Human chorionic gonadotropin (hCG)
r
In Down’s Syndrome, hCG is elevated and PAPP - A is normal.
r

Ultrasonogram
r
Nuchal Translucency
Maximum subcutaneous translucent area between skin and neck in cervical region
r

measured in mid - sagittal plane with head in neutral position.


Usually measured at 11 weeks - 13 weeks of gestation.
r

>= 3 mm is abnormal and indicates risk of Trisomy 21, Turner’s Syndrome & CVS
r

anomalies.
r
Biochemical tests + USG togetehr is termed as combined test.

Second Trimester
r
Biochemical Tests : Quadruple test done at 15 - 22 weeks.
Alphafoetoprotein (AFP)
r

Unconjugated Estradiol (E3)


: Human Chorionic Gonadotropin (hCG)
Inhibin A
: In Down’s Syndrome, hCG and inhibin A are increased.
AFP, unconjugated estradiol and hCG together are called triple test/
: Kettering test/ BART test.
Ultrasonogram
Nuchal fold thickness (>= 6 mm : Abnormal)
: Short humerus, femur
Echogenic bowel, Cardiac focus Abnormal
: Cystic hygroma
Absent nasal bone
:If screening tests are positive, cell free foetal DNA testing or Karyotyping is done.
Cell Free Foetal DNA testing
Done >= 10 weeks of gestation.
: Available only for Down’s syndrome, Trisomy 13, 18 and monosomy X.
Best screening test for Down’s Syndrome, but it is expensive.
: Results take 7 - 10 days.

Karyotyping
First Trimester
:
Chorionic Villus Sampling
Best done at 11 - 13 weeks.
: Done under USG guidance.
Most common complication : Foetal loss (1%)
: Most common complication in < 9 weeks : Oromandibular limb defect gestation.
Second Trimester
: Amniocentesis
Best done at 16 - 18 weeks.
: Amniotic fluid (Amniocytes + Fibroblasts) is studied.
I

Foetal loss (0.5%) is a complication.

You might also like